Prefazione `E con grande piacere che vedo apparire la nuova

Transcript

Prefazione `E con grande piacere che vedo apparire la nuova
Prefazione
Introduzione
È con grande piacere che vedo apparire la nuova edizione, aggiornata e completata, del libro I problemi
di Matematica della Scuola Normale .
L’idea di fornire agli studenti liceali e ai loro insegnanti una serie di esempi di problemi di matematica,
molti dei quali di tipo non proprio convenzionale, nacque all’inizio degli anni ottanta, nel corso di una
riunione della Commissione che preparava le prove per il concorso di ammissione alla Scuola Normale. Si
trovarono i testi delle prove assegnate a partire dal 1906, e si pensò che il presentarli con le soluzioni discusse
e commentate avrebbe potuto essere un contributo utile alla didattica e alla diffusione della matematica.
Il successo che il libro ha avuto suggerisce ora di riproporlo in forma più estesa con gli aggiornamenti fino
al 1997. Desidero ringraziare, a nome della Scuola Normale, il professor Franco Conti e il dottor Alessandro
Profeti, che hanno svolto con la loro riconosciuta competenza questo arduo compito, ed esprimo l’augurio
che questo testo accompagni generazioni di giovani nell’apprendimento dei segreti di una tra le più belle
conquiste della mente dell’uomo.
Franco Bassani
Direttore della Scuola Normale Superiore
Il presente volume si propone come continuazione dell’opera I problemi di Matematica della Scuola
Normale Superiore, pubblicato nel 1985, che raccoglieva in maniera sistematica gli esercizi di matematica
assegnati per il concorso di ammissione negli anni 1906 –1984.
Entrano in questa raccolta tutte le prove di esame assegnate dal 1985 fino al 1997 e una selezione degli
esercizi più interessanti (scelta difficile e un poco arbitraria) degli anni precedenti.
I temi presentano una grande varietà, che riflette tanto le preferenze culturali dei docenti che di anno
in anno si avvicendano nella commissione di concorso, quanto l’esigenza, sempre presente, che le prove
non discriminino sulla base di nozioni diligentemente apprese e meccanicamente applicate ma evidenzino
invece l’inventiva dei candidati.
Nei temi assegnati sono egualmente rappresentate l’innovazione e la tradizione. Accanto a problemi di
calcolo delle probabilità e problemi di matematizzazione di situazioni concrete, che non hanno ancora trovato
spazi adeguati nell’insegnamento della matematica, compaiono temi di tipo combinatorio o aritmetico. Uno
spazio abbastanza ampio continua ad essere dedicato ai problemi di geometria piana e solida, che a parere
dei curatori hanno rappresentato e dovrebbero continuare a rappresentare un momento importante per
l’educazione della sensibilità e della intuizione in matematica e che al momento vedono molto ristretto il
loro ruolo nei programmi di matematica della scuola secondaria.
Il libro non è inteso come una raccolta di temi svolti da cui attingere ricette per la soluzione di problemi,
obbiettivo questo difficile da un lato e di dubbia utilità educativa dall’altro; è auspicio dei curatori che
esso venga inteso come una raccolta di domande a cui lo studente in procinto di partecipare all’esame
di ammissione o semplicemente il lettore interessato alla matematica cerca in primo luogo di rispondere
personalmente, per poi confrontare le proprie strategie risolutive con quelle esposte nelle soluzioni. Per
questo motivo è parso utile fornire di alcuni esercizi più soluzioni, basate sulle diverse componenti della
educazione matematica di uno studente di scuola media superiore.
Questa raccolta si rivolge in primo luogo agli studenti e ai docenti della Scuola Secondaria, tenendo
conto che è sempre più sentito fra gli insegnanti il problema di stimolare e indirizzare gli alunni più dotati,
ma anche a tutti coloro che sono interessati a conoscere i metodi di soluzione di problemi matematici anche
difficili e a cimentare la propria inventiva matematica, anche se tale disciplina è lontana dalla loro attività
quotidiana.
Il testo dei problemi è premesso, per invogliare il lettore a cercare da solo le soluzioni, confrontandole
poi con quelle esposte nella seconda parte, che non sono necessariamente le migliori possibili.
Un asterisco contraddistingue gli esercizi che, nell’opinione dei curatori, presentano particolari elementi
di originalità o di difficoltà o di fantasia.
Conclude il volume un glossario in cui vengono richiamate alcune nozioni utili per affrontare i quesiti;
i termini succintamente citati nel glossario compaiono nel testo in carattere evidenziato. Nella stesura
del glossario si è preferito dare dei lemmi una presentazione più agile e semplice, rinunciando ad una
maggiore generalità che avrebbe appesantito l’esposizione.
Infine una osservazione: contrariamente all’uso italiano si è scelto di non distinguere tipograficamente
gli enti geometrici dalle loro grandezze o dalla loro misura, conformemente con una tradizione consolidata
nella letteratura anglosassone.
I curatori
Pisa, aprile 1998
Testi
1
1906.1*
Sono dati, in un piano, un circolo e due punti. Tirare una tangente al circolo in modo che la
somma delle distanze della tangente dai due punti dati sia uguale a una lunghezza data.
1909.1
In un piano sono dati due circoli che si tagliano in due punti. Si domanda di condurre per uno di
questi punti una retta sulla quale le corde staccate dai due circoli stiano fra loro in un rapporto dato.
1909.2*
Dimostrare che se a1 , a2 , . . . , an sono n numeri diversi da zero, fra i quali sussiste la relazione
2
a12 + a22 + · · · + an−
1
a22 + a32 + · · · + an2 =
(a1 a2 + a2 a3 + · · · + an−1 an )2 ,
i numeri stessi sono necessariamente in progressione geometrica.
1911.1
Dividere un dato arco di cerchio in due archi le cui corde siano tra loro in un rapporto dato.
1912.2*
In un dato quadrilatero inscrivere un parallelogrammo il cui centro cada in un punto dato.
1915.2*
Dimostrare che se per un punto interno ad una sfera si conducono tre piani, a due a due
perpendicolari, le somme delle aree dei tre cerchi che essi determinano sulla sfera è costante.
2
I problemi della Scuola Normale
1923.2*
Dati due punti sopra le facce di un diedro, qual è la spezzata minima che li unisce e che ha un
vertice sullo spigolo?
1926.2*
Condurre un piano che seghi un dato angolo tetraedro in un parallelogrammo.
Testi
3
1963.5
Un podista si trova su un punto della Terra, che supponiamo perfettamente sferica. Percorre un
chilometro verso nord, poi uno verso est e infine uno verso sud. Si ritrova al punto di partenza.
Quali sono i punti di partenza che obbediscono a questa condizione?
1967.2*
Sono assegnate tre rette parallele. Esiste un triangolo equilatero con i vertici rispettivamente
sulle tre rette?
1927.2
Determinare il luogo dei centri delle circonferenze ottenute segando una superficie sferica con i
piani passanti per un punto qualunque dello spazio.
1929.2* Verificare che il luogo dei punti M tali che la somma dei quadrati delle loro distanze da tre punti
dati A, B , C sia uguale ad una assegnata costante k 2 è un cerchio.
1939
È dato un rettangolo ABCD. Facendolo ruotare attorno al lato AB si ottiene un primo cilindro
e facendolo ruotare attorno al lato BC si ottiene un secondo cilindro. Si conosce la somma 4 π S 2 delle
superfici laterali dei due cilindri; e pure si conosce la somma π V 3 dei volumi degli stessi. Si domanda di
determinare i lati del rettangolo ABCD.
Qual è la condizione di realtà? Quante soluzioni ha il problema? Fissato V , qual è il massimo valore
possibile per S ?
1943
Determinare un triangolo rettangolo di cui siano noti il perimetro e il raggio del cerchio inscritto.
Una volta fissato il raggio del cerchio inscritto, qual è il valore minimo del perimetro?
1968.3
Sono dati in un piano quattro punti A, B , C , D, in modo che A, B , C e A, B , D siano vertici
di triangoli equilateri distinti. Determinare tutte le circonferenze β che godono della seguente proprietà: i
quattro punti A, B , C , D hanno dalla circonferenza β ugual distanza.
1970.4
Fissato un intero positivo n, determinare il più piccolo intero m tale che, presi comunque m
numeri interi positivi, una almeno delle seguenti eventualità si verifichi:
(a) tra gli m numeri considerati ve ne sono n uguali;
(b) tra gli m numeri considerati ve ne sono n distinti.
1971.4*
Una palla si trova su un biliardo in posizione P . Provare che esiste almeno una direzione
secondo cui si può lanciare la palla in modo che essa non ripassi mai per la posizione P .
Si consideri il biliardo privo di attrito e si supponga che il rimbalzo alle sponde obbedisca alla stessa legge
di riflessione della luce.
1960.3
È più facile, gettando una volta un dado, ottenere 6, oppure, gettandolo tre volte, ottenere tutte e
tre le volte un numero pari?
1973.3*
Un treno parte da Pisa. Il macchinista controlla il cronometro e nota che la lancetta dei secondi
è sullo zero. Dopo aver percorso 8 chilometri, il macchinista controlla di nuovo il cronometro e nota che
la lancetta dei minuti copre esattamente quella delle ore. La velocità media del treno per gli 8 chilometri
percorsi è di 33 chilometri l’ora. A che ora è partito il treno da Pisa?
1961.4*
Con una bilancia a piatti e un certo numero di pesi, si vogliono pesare oggetti di peso inferiore
a 500 grammi, con un errore non superiore ad un grammo. Non si possono mettere pesi nel piatto su cui
poggia l’oggetto. Dire qual è il numero minimo di pesi sufficienti a tale scopo.
1974.5
Dati tre numeri interi a, b, c aventi massimo comun divisore 1, verificare che i numeri della forma
a m2 + b m + c ,
con m intero qualunque, non possono essere tutti divisibili per 14.
Generalizzare il risultato.
1962.5*
Si sostiene talvolta che noi usiamo il sistema decimale di numerazione (per cui, ad esempio,
362 significa 3 · 102 + 6 · 10 + 2) in quanto abbiamo 10 dita.
Un marziano, dopo aver visto scritta l’equazione:
x2 − 16 x + 41 = 0 ,
invitato a scrivere la differenza delle radici scrive 10.
Quante dita hanno i marziani?
(N.B. Per i numeri compresi fra 0 e 6 la scrittura dei marziani coincide con la nostra).
1976.3 Sia n un intero maggiore di 2 e sia ∆ un triangolo rettangolo. Dimostrare che la potenza n-esima
della lunghezza dell’ipotenusa di ∆ è maggiore della somma delle potenze n-esime dei cateti.
4
I problemi della Scuola Normale
Testi
5
1977.1*
Su un tavolo orizzontale vi è una pila di 7 dischi metallici perfettamente uguali, ognuno di
diametro 40 cm.
Dire quale è la distanza massima sul piano orizzontale che può avere il centro del disco più alto dal centro
del disco più basso senza che la pila crolli.
1984.6*
Siano dati una circonferenza γ e un punto P distinto dal centro. Sia P AB un triangolo che, tra
tutti quelli che hanno un vertice in P e i rimanenti due su γ , abbia perimetro massimo.
Dimostrare che le due bisettrici uscenti dai vertici A e B passano per il centro di γ .
(Non si richiede la costruzione geometrica, né la determinazione degli elementi del triangolo.)
1979.1*
Un battello scende lungo un fiume; sia alla partenza sia a ogni stazione intermedia salgono
sul battello tanti passeggeri, ognuno diretto ad una diversa stazione, quante sono le fermate successive.
Sapendo che il numero massimo di passeggeri contemporaneamente presenti sul battello è 380, si determini
il numero delle stazioni.
1985.1
1979.3
Per un punto P passano tre superfici sferiche distinte tra loro. Si considerino le affermazioni
seguenti:
(a) nessuna retta passante per P è tangente a tutte e tre le sfere;
(b) nessuna sfera è tangente a un’altra;
(c) esiste un altro punto Q comune alle tre superfici sferiche.
Dire, per ogni coppia di affermazioni, se esse sono incompatibili, se sono equivalenti o se una delle due
implica l’altra.
1980.6 Un’autostrada ha n caselli a distanze successive di p chilometri. Si è osservato che ogni macchina
entra con uguale probabilità da ogni casello e esce con uguale probabilità da un altro casello. Trovare la
lunghezza del percorso medio di ogni macchina.
1981.6*
Dato un tetraedro avente 5 dei 6 spigoli di lunghezza minore o uguale a 2, provare che il suo
volume è minore o uguale a 1. In quale caso il volume è uguale a 1?
1982.5*
Dato un piano α e due punti P , Q nello stesso semispazio si considerino le sfere passanti per i
punti P , Q e tangenti al piano α.
Si richiede di determinare il luogo dei punti di tangenza.
1983.5*
Due amici si sono iscritti alla prima classe di un liceo. Tale liceo ha due sezioni, le cui prime
classi hanno rispettivamente n e m studenti, con n e m compresi tra 20 e 30.
Sapendo che la probabilità che i due amici si trovino nella stessa classe è 1/2, dire quanti sono gli studenti
delle due classi.
L’eguaglianza
p ! + q ! + r ! = s!
è soddisfatta per p = q = r = 2 e s = 3.
Dire se esistono altri numeri interi positivi per cui tale eguaglianza è vera.
(Si ricorda che n! indica il fattoriale di n, cioè il prodotto n · (n − 1) · . . . · 3 · 2 · 1 dei primi n numeri interi.)
1985.2*
Si considerino nel piano due circonferenze γ e γ 0 di eguale raggio. Determinare il luogo dei
punti medi P dei segmenti AA0 con A in γ e A0 in γ 0 .
1985.3
Fra i triangoli equilateri contenuti in un quadrato assegnato, determinare quelli di area massima.
1985.4
Fissati due punti P e Q su due lati consecutivi di un dato rettangolo, si determinino sugli altri
due lati due punti R e S tali che il quadrilatero P QRS abbia area massima.
1985.5
Per la costruzione di un certo ponte si prevede che il costo di ogni arcata sarà di 18 s2 miliardi
di lire, ove s è la distanza in chilometri tra i due piloni di sostegno di quell’arcata, mentre il costo di ogni
pilone sarà di mezzo miliardo.
Se il ponte deve essere lungo 3 chilometri quale sarà il costo minimo dell’opera?
3 km
s1
s2
sn
6
1985.6
I problemi della Scuola Normale
Con una bilancia a piatti e disponendo di infiniti pesi campione
p, p1 , q1 , p2 , q2 , . . . , pn , qn , . . .
Testi
7
◦
1986.6 Si consideri un biliardo di forma triangolare, come in figura, con α = 30 e si supponga di lanciare
una bilia dal punto A.
da
1, λ, 1/λ, λ2 , 1/λ2 , . . . , λn , 1/λn , . . .
α
grammi rispettivamente, dove λ è un numero reale maggiore di 1, si vogliono pesare tutti gli oggetti con
una precisione arbitrariamente grande. Per quali valori di λ ciò è possibile?
( N.B. Si dispone di un solo esemplare di ogni peso campione e non si possono mettere pesi campione sul
piatto che contiene l’oggetto da pesare ).
1986.1
Si determinino gli interi positivi k tali che il polinomio
A
x5 + x4 + x3 + k x2 + x + 1
sia prodotto di polinomi a coefficienti interi di grado minore di 5.
1986.2
Si dimostri che il sistema di equazioni
(
y ex − e−y = x y ex
B
Si provi che la bilia, qualunque sia la direzione iniziale, effettua solo un numero finito di rimbalzi prima di
battere sulla sponda AB .
Si determini anche il numero massimo di tali rimbalzi.
Si studi infine il caso in cui α è un angolo generico.
(Si supponga che il biliardo sia privo di attrito, che la palla sia puntiforme e che il rimbalzo sulle sponde
obbedisca alla legge di riflessione della luce). 1
x ey − e−x = ey
ha una sola soluzione.
1987.1
Siano assegnati due numeri reali positivi non nulli r e p, con r < p. Tra tutti i quadrilateri
convessi di perimetro p, aventi la somma delle lunghezze di una coppia di lati consecutivi uguale ad r, si
determini quello di area massima.
1986.3
Si dimostri che la composizione di due omotetie nello spazio, con poli P e Q distinti, è ancora
una omotetia di polo R, allineato con P e Q, oppure una traslazione parallela a P Q.
1987.2
b minore di 60◦ . Si
1986.4
Sia ABC un triangolo isoscele di base BC con l’angolo al vertice B AC
costruisca un altro triangolo P QR, di base QR, circoscritto e simile ad ABC , tale che il punto A appartenga
al segmento QR e si abbia QA = 2 · AR.
Siano p, q , r tre numeri reali tali che il polinomio
A(x) = x3 + p x2 + q x + r
abbia tre radici reali.
Determinare tre numeri reali a, b, c, espressi in funzione di p, q , r soltanto, in modo che il polinomio
B (x) = x3 + a x2 + b x + c
abbia per radici i quadrati delle radici di A.
1986.5
(a) Siano α, β , γ , δ quattro angoli minori di 180◦ . Si dimostri che
sin α + sin β + sin γ + sin δ ≤ 4 sin
α+β+γ+δ
.
4
(b) Utilizzando la relazione precedente,
si dimostri che la somma dei seni degli angoli interni di un triangolo
√
è sempre minore o eguale a 3 3/2.
1987.3
Sia dato un segmento AB nel piano. Si consideri il luogo L dei punti del piano che vedono il
segmento AB sotto un angolo di 60◦ . Si scelga P in L e si scelgano due punti C e D rispettivamente interni
ai lati BP e AP del triangolo ABP , in modo che AD = BC .
Si costruisca il triangolo equilatero CDQ di base CD, esterno al quadrilatero ABCD.
Si studi, al variare di P in L e di C , D secondo le condizioni indicate sopra, il luogo dei punti del piano
descritto dal punto Q.
1
Il testo originale del problema è leggermente diverso (N. d. C.).
8
I problemi della Scuola Normale
1987.4
Un punto (x, y ) del piano si dirà razionale se x e y sono numeri razionali. Data una qualunque
circonferenza del piano cartesiano avente centro razionale, si provi che se essa contiene un punto razionale,
allora contiene infiniti punti razionali.
1987.5
Nella figura è rappresentato lo sviluppo delle facce di un tetraedro regolare nello spazio.
M
A
Q
N
9
1988.3
Si considerino i numeri naturali 1, 11, 111, . . . , e in generale si indichi con αn il numero che si
ottiene giustapponendo n cifre uguali a 1.
(a) Si provi che se αn è un numero primo allora n è primo.
(b) Si provi che, assegnato comunque un numero naturale r, non divisibile né per 2 né per 5, si può trovare
un αn multiplo di r.
(c) Si scriva un algoritmo o programma per calcolatore (in un qualunque linguaggio di programmazione)
che, a partire da r, calcoli il minimo n per cui vale la (b).
C
R
P
Testi
O
B
Siano P , Q, R tre punti distinti del tetraedro corrispondenti, nello sviluppo, rispettivamente ad un punto
interno al segmento M N , un punto interno al segmento M O, un punto interno al triangolo M OC .
Sia α il piano contenente P , Q, R. Si determini, nello sviluppo piano, l’intersezione fra il piano α e le facce
del tetraedro.
1987.6
Tizio si trova nella sua abitazione e deve prendere un treno che parte dalla stazione esattamente
tra mezz’ora. Sotto la sua abitazione c’è la fermata degli autobus della linea A che lo portano alla stazione
in 20 minuti. A 5 minuti di cammino vi è una fermata delle linee B e C che lo possono portare alla stazione
in 18 minuti.
Tizio non conosce l’orario di passaggio degli autobus, ma sa che su ognuna delle linee gli autobus passano
ogni quarto d’ora.
Quale strategia conviene a Tizio per aver maggiore probabilità di prendere il treno?
1988.1
Siano A, B , C , D quattro punti distinti assegnati nello spazio. Determinare una condizione
necessaria e sufficiente affinché ogni superficie sferica che passa per A e B intersechi ogni superficie sferica
che passa per C e D.
1988.2
Un ragazzo ha concluso la terza media, sa che sei bravo e ti chiede:
(a) cosa è un poligono piano,
(b) cosa è l’area di un poligono piano.
Esponi le definizioni richieste con rigore, chiarezza e concisione (in un massimo di 12 righe).
Volendo dare le stesse definizioni ad un livello scolastico più avanzato, quali precisazioni occorrerebbe
fare? (assiomi e definizioni precedenti, enunciati di eventuali teoremi, . . .).
1988.4
Sia assegnato su un piano un numero n arbitrario di triangoli con la proprietà che tre qualsiasi
di essi abbiano almeno un punto in comune. Si dimostri che tutti i triangoli assegnati contengono uno
stesso punto. Come occorre modificare l’ipotesi perché la stessa conclusione valga per un numero finito di
triangoli nello spazio?
1988.5 Sia p(x) = a x4 + b x3 + c x2 + d x + e un polinomio con coefficienti razionali. Si supponga che, per
ogni intero m maggiore di un certo m0 , il numero p(m) sia intero. Si dimostri che allora 24 a è un numero
intero.
Si generalizzi questo risultato a polinomi p(x) con coefficienti razionali di grado qualsiasi.
1988.6
Un laboratorio deve organizzare il trasferimento di 10 m3 di scorie radioattive liquide. Occorre
ordinare un numero n di contenitori, identici, che possano contenere tali scorie e garantire un trasporto
sicuro.
Si stima che il costo di ciascuno di tali contenitori sia 16 V 2 milioni di lire, ove V è il volume (in m3 ) di
scorie che ciascuno di essi può contenere; il costo di riempimento di ciascun contenitore risulta essere di
un milione di lire, indipendentemente dalla sua capienza.
(a) Quanti contenitori e di quale volume dovrà ordinare il laboratorio per spendere il meno possibile?
(b) Al momento di effettuare l’ordine si viene a sapere che la ditta fornitrice pratica sul prezzo dei contenitori
ordinati un piccolo sconto del k per cento, con k intero, se il loro numero uguaglia o supera le 50 unità
(il costo del riempimento rimane inalterato). Qual è il più piccolo k per il quale risulta conveniente
modificare l’ordine, e perché?
1989.1
Per 0 ≤ x ≤ π/2 e n = 0, 1, 2, . . . , si ponga:
Fn (x) = 1 − sin x + sin2 x − . . . + (−1)n sinn x.
Provare che:
(a) per 0 ≤ α ≤ 1/2, l’equazione F2k = α non ha soluzioni qualunque sia k naturale;
(b) per 1/2 < α ≤ 1, esiste un numero naturale k ∗ tale che, per ogni k > k ∗ , l’equazione F2k = α ha almeno
due soluzioni.
1989.2
Sia S una superficie sferica di centro O. Per ogni P ∈ S , sia fP : S → S l’applicazione che ad
ogni Q ∈ S associa il punto fP (Q) simmetrico di Q rispetto all’asse OP . Dimostrare che:
(a) per ogni P ∈ S l’applicazione fP è composizione di due simmetrie rispetto a piani;
(b) per ogni P , X , Y ∈ S la distanza tra X e Y è uguale alla distanza tra fP (X ) e fP (Y ).
(c) per ogni P , Q, X ∈ S si ha che fP (fQ (X )) = ffP (Q) (fP (X )).
10
1989.3
I problemi della Scuola Normale
Testi
11
1990.2
Sia P un poligono semplice (cioè tale che da ogni vertice escono esattamente due lati) non
necessariamente convesso, con almeno 4 lati. Supponiamo che P abbia al più un vertice concavo.
E’ vero che esistono due vertici non consecutivi con la proprietà che il segmento che li congiunge è contenuto
in P ? In caso affermativo dimostrarlo, altrimenti trovare un controesempio.
Trovare le soluzioni reali del sistema:

2
2

2y + x − x − y = 0
z − x + y − y (x + z ) = 0


− 2 y + z − y2 − z 2 = 0
1990.3
1989.4 Consideriamo la legge che ad ogni punto P = (x, y ) del piano cartesiano fa corrispondere il punto
f (P ) dello stesso piano, definito da:
P
se OP ≤ 1
x
y f (P ) =
,
se OP ≥ 1
OP OP
dove O = (0, 0) e OP indica la distanza da O a P .
Provare che per ogni coppia di punti P e Q la distanza fra f (P ) e f (Q) non supera la distanza fra P e Q.
Dato il sistema

x1 + x2 + . . . + x100 = 5050




2
2

x

 2 − x1 = 3


 · · · ·

x2k − x2k−1 = 2k − 1





· · · · · ·



 2
x100 − x299 = 199
trovare tutte le soluzioni x1 , x2 , . . . , x100 , con xk ≥ 0, k = 1, 2, . . . , 100.
1989.5
Sia S la superficie di un prisma a base ottagonale regolare inscritto in un cilindro√circolare retto
di raggio R e altezza h = 3R sin(π/8). Siano A e B due punti come in figura, con OA = R/ 2.
P
O
A
N
M
B
Determinare la lunghezza del minimo percorso su S tra A e B .
1989.6
Sia f (x) una funzione a valori reali definita sulla semiretta reale {x ≥ 0}. Supponiamo che:
(a) f (x) sia derivabile con derivata f 0 continua;
(b) f (0) = 0;
(c) per ogni x ≥ 1 risulti
1990.4
Sia dato il polinomio F (x) = xn + an−1 xn−1 + . . . + a0 con coefficienti ai interi. Supponiamo che
esistano quattro interi distinti a, b, c, d tali che F (a) = F (b) = F (c) = F (d) = 7.
Dimostrare che non esiste nessun numero intero k tale che F (k ) = 12.
1990.5
Trovare il più piccolo numero α > 1 tale che risulti:
α + sin x
≤ ey−x
α + sin y
per ogni x, y con x ≤ y
1990.6 Si consideri un rettangolo R di misure 6 × 4 metri. Nel punto di mezzo O di un lato è incernierato
un braccio articolato, della lunghezza totale di due metri, formato da due segmenti OA e AB (vedi figura).
Il braccio può muoversi solo all’interno di R. Più precisamente il segmento OA può ruotare intorno al
punto fisso O e, per ogni posizione assunta, il segmento AB può ruotare intorno al punto A; naturalmente,
durante il movimento, il braccio deve restare in R.
R
0 < f (x) ≤ xf 0 (x).
Provare che l’equazione f (x) = k ha almeno una soluzione x ≥ 0, per ogni k ≥ 0 .
B
A
O
1990.1
Considerare nello spazio euclideo nove punti distinti a coordinate intere. Dimostrare che ne
esistono due tali che il segmento che li congiunge contiene almeno un punto interno (cioè distinto dagli
estremi) a coordinate intere.
E’ possibile scegliere le lunghezze dei segmenti in modo che ogni punto di R a distanza minore o uguale a
due metri da O sia raggiunto da B ?
12
1991.1
I problemi della Scuola Normale
Provare che per ogni numero intero n ≥ 2 si ha
√
n+1
n
n! <
2
e che (n + 1)/2 non è mai un multiplo intero di
√
n
n!
Testi
13
1991.6
Ci si propone di congiungere con una strada due località A e B che distano 4 km, fra le quali si
trova una zona Z costituita da terreno pietroso e avente la forma di un cerchio con centro nel punto medio
di AB e raggio di 1 km.
(a) Sapendo che, a parità di lunghezza, il costo di costruzione della strada nella zona pietrosa è λ volte (λ
numero reale maggiore di 1) quello relativo alla zona circostante, determinare due punti P , Q sul bordo
di Z in modo che il percorso AP QB (vedi figura) sia il più economico possibile.
Q
P
1991.2
Fra tutti i quadrilateri convessi inscritti in un quadrato, in modo che ogni lato del quadrato
contenga almeno un vertice del quadrilatero, si determinino quelli aventi minimo e massimo perimetro.
Z
A
1991.3 Trovare il più piccolo numero intero N0 ≥ 1 con la proprietà che N0 + 1 e 2N0 + 1 siano entrambi
quadrati perfetti.
Mostrare poi che ogni intero N con questa proprietà è multiplo di N0 .
1991.4
Su un treno, inizialmente senza passeggeri e formato da n carrozze, salgono k viaggiatori
disponendosi in modo casuale e indipendente l’uno dall’altro. Qual è la probabilità che solo tre carrozze
siano occupate da almeno un viaggiatore?
O
B
(b) Discutere poi il caso più generale in cui si considerano percorsi formati, oltre che da tratti rettilinei,
anche da eventuali tratti curvilinei contenuti nel bordo di Z (dove il costo unitario di costruzione si può
considerare lo stesso che nella zona esterna a Z).
1992.1
E’ assegnata una legge che a ogni coppia di interi x, y associa un intero x y in modo che
x (y + z ) = y x + z x
per tutti gli interi x, y , z . Si dimostri che
1991.5
x y = x y (1 1).
Costruire un polinomio (a coefficienti reali)
2
P (x, y ) = a x + b x y + c y
2
verificante le proprietà:
(i) P (x, y ) = 0 soltanto per x = y = 0 ;
(ii) se x e y sono numeri interi allora anche P (x, y ) è intero.
Determinare poi il massimo della quantità
1992.2
Nel piano due quadrati ABCD e A0 B 0 C 0 D0 sono disposti come in figura.
C
B
∆ = b2 − 4 a c
B0
al variare di P nell’insieme dei polinomi soddisfacenti le proprietà precedenti.
D
A
A0
C0
D0
Si dimostri che la retta passante per A e perpendicolare a DD0 incontra il segmento BB 0 nel punto medio.
1992.3
Verificare che la somma delle quarte potenze di due numeri reali di assegnato prodotto p > 0
a) decresce se decresce il valore assoluto della differenza dei due numeri;
b) raggiunge il valore minimo quando i due numeri sono uguali.
14
1992.4
I problemi della Scuola Normale
Mostrare che, per ogni intero positivo fissato k , esiste almeno un intero n tale che
100 ≤ nk + n ≤ 101 + k nk−1 .
1992.5 Sia F (x) = a3 x3 + a2 x2 + a1 x + a0 un polinomio di terzo grado con coefficienti interi. Si dimostri
che
i) se p/q , con p, q interi primi tra loro e q 6= 0, è una radice del polinomio, per ogni intero m il numero
F (m) è divisibile per p − m q ;
ii) se esistono due interi x1 e x2 tali che F (x1 ) = 1, F (x2 ) = −1, e che |x1 − x2 | > 2, allora F (x) non ha
radici razionali.
1992.6
1) Dimostrare che, presi comunque tre vertici di un cubo, il triangolo da essi individuato è rettangolo
oppure equilatero.
2) Calcolare la probabilità che tre distinti vertici del cubo, scelti a caso, individuino un triangolo rettangolo
(la probabilità è il rapporto tra il numero di casi favorevoli e il numero di casi possibili).
3) Si escludano tre vertici del cubo, e si considerino “ammissibili” i restanti cinque. Si indichi con P
la probabilità che tre vertici ammissibili del cubo, scelti a caso, individuino un triangolo rettangolo.
Stabilire quanti valori può assumere P al variare dei tre vertici esclusi all’inizio.
1993.1
Nel piano cartesiano riferito a coordinate ortogonali, un quadrato striscia in guisa che due suoi
vertici consecutivi appartengano rispettivamente all’asse delle ascisse non negative e a quello delle ordinate
non negative.
Descrivere analiticamente la traiettoria di un punto P interno al quadrato e rigidamente ancorato ad esso.
Caratterizzare le posizioni di P nel quadrato per le quali la traiettoria appartiene ad una circonferenza,
oppure è un segmento di retta.
D
P
1993.2
In un piano cartesiano un oggetto puntiforme parte dal punto (0, 2 n) (con n intero positivo) e
scende fino all’asse delle ascisse compiendo 2 n passi, con la seguente regola: se prima di compiere un
passo si trova nel punto di coordinate intere (k, l), può recarsi o in (k − 1, l − 1) o in (k + 1, l − 1) con uguale
probabilità.
Le mosse eseguite nei diversi passi sono indipendenti.
Si indichi con pn (k ) la probabilità che dopo 2 n passi l’oggetto si trovi nel punto (k, 0).
i) Calcolare pn (k ).
ii) Mostrare che 2 pn (2) ≥ 1/(2 n + 1).
1993.3
Dati tre numeri interi p > 2, q > 2, r > 2 si consideri un parallelepipedo di legno tale che
i tre spigoli uscenti da un vertice abbiano lunghezza p, q , r. Dopo aver dipinto la superficie esterna del
parallelepipedo, questo viene tagliato, mediante sezioni parallele alle facce, in cubetti aventi spigoli di
lunghezza 1. Ovviamente alcuni dei cubetti sono parzialmente colorati, mentre altri non sono colorati
affatto.
Si dimostri che esiste solo un numero finito di terne (p, q, r) per ciascuna delle quali il numero dei cubetti
parzialmente colorati è uguale al numero di quelli che non sono colorati affatto.
1993.4
Sia data una circonferenza Γ. Un arco circolare γ congiunge due punti distinti di Γ ed è interno
al cerchio C racchiuso da Γ. Dimostrare che, se le due regioni in cui γ divide C hanno aree uguali, la
lunghezza di γ supera il diametro di Γ.
1993.5
Sia n un intero positivo pari. Mostrare che si possono trovare (in modo non necessariamente
unico) n numeri reali a1 > a2 > . . . > an > 0 tali che
(
A
a1 + a2 + . . . + an = 2 n
(a1 − a2 ) + (a3 − a4 ) + . . . + (an−1 − an ) = n .
a1 − an ≤ n .
B
1993.6
O
15
Mostrare poi che questo non è possibile (per n > 2) se si impone la condizione ulteriore
C
y
Testi
x
Per ogni intero positivo n si indichi con θ(n) il numero reale, compreso fra 0 e 2 π , tale che
θ(n) = 3 n (mod 2 π )
cioè tale che θ(n) − 3 n sia un multiplo intero di 2 π .
i) Mostrare che 0 ≤ θ(n) ≤ π/2 per infiniti valori di n (si può usare il fatto che 3.14 < π < 3.15).
ii) Mostrare che θ(n) 6= θ(m) se n 6= m.
1994.1
Sia X un insieme di n elementi, dove n è un numero pari e sia k un intero positivo. Diciamo che
una funzione f da X in X ha molteplicità k se per ogni a ∈ X l’insieme {x | f (x) = f (a)} ha k elementi.
Sono di più le funzioni di molteplicità 1 o quelle di molteplicità 2?
16
I problemi della Scuola Normale
Testi
1994.2
Vi sono 4 città collegate a due a due da 6 strade che non si intersecano (cioè ogni coppia di città
è collegata da una sola strada). Tutte le strade sono aperte al traffico con la stessa probabilità p = 1/2.
Determinare la probabilità che in un determinato istante partendo da una qualsiasi città si possa arrivare ad
ogni altra città.
17
1995.2
Dati quattro punti distinti nel piano dimostrare che è sempre possibile sceglierne tre che determinino un angolo inferiore o uguale a 45◦ . In generale dati n punti dimostrare che se ne possono scegliere
3 che determinino un angolo inferiore o uguale a 180◦/n.
1995.3
Dimostrare che se a, b, c sono tre interi consecutivi allora a3 + b3 + c3 è multiplo di 9.
1994.3 Mostrare che 41 non può essere espresso come differenza di una potenza di 2 e di una potenza di
3, cioè che non può sussistere nessuna delle due uguaglianze seguenti:
41 = 2n − 3m ,
41 = 3n − 2m
1995.4
Sia f : R → R una funzione non negativa, con concavità rivolta verso il basso, derivabile e tale
che f 0 (0) > 0 e f (x) = f (2 − x) per ogni x ∈ R.
Dimostrare che
con n, m interi positivi.
1994.4
Sia P un punto interno ad un triangolo equilatero. Per ogni retta passante per P siano X e Y i
due punti di intersezione tra la retta e i lati del triangolo. Determinare, per ogni punto P , la retta o le rette
che rendono minimo il prodotto
PX · PY
[1]
Siano a1 , a2 , . . . , an numeri reali e siano b1 , b2 , . . . , bn definiti da
bi = max (i · j − aj )
1≤j≤n
f (x) dx ≤ 2f (1) −
[f (1) − f (0)]2
f 0 (0)
Provare che se la condizione f (x) = f (2 − x) non è verificata, tale diseguaglianza può non valere.
Bk = (x, y ) ∈ R2 : |x| + |y| ≤ k
contenuti in A. Determinare k in funzione di a in modo che Bk abbia area massima tra gli insiemi di C .
1995.6
Siano a, b numeri reali non negativi tali che b2 + b6 ≤ a2 − a6 . Dimostrare che allora risulta:
1) a ≤ 1;
2) b < 2/3.
per ogni i = 1, 2, . . . , n.
Allo stesso modo si costruiscono c1 , c2 , . . . , cn a partire da b1 , b2 , . . . , bn e poi d1 , d2 , . . . , dn a partire da c1 ,
c2 , . . . , cn ; si dimostri che
ci ≤ a i
di = bi
0
2
1995.5
Dato A = (x, y ) ∈ R2 : |x|2a + |y|2a ≤ 1 ove a è un numero reale positivo, si consideri al
variare di k ≥ 0 la classe C degli insiemi
1994.5 Consideriamo un triangolo e dividiamo i suoi lati in n parti uguali mediante n − 1 punti su ciascun
lato. Congiungiamo ogni vertice con i punti cosı̀ ottenuti sul lato opposto. Si dimostri che se n è primo
maggiore di 2 allora non esistono punti appartenenti simultaneamente a tre dei segmenti cosı̀ costruiti.
1994.6
Z
per ogni i = 1, 2, . . . , n,
per ogni i = 1, 2, . . . , n.
1996.1 Dato un quadrato ABCD di lato unitario, determinare la massima costante α e la minima costante
β per cui si ha
α ≤ PA + PB + PC + PD ≤ β
per ogni punto P contenuto nel quadrato.
1995.1
Siano C1 , C2 due circonferenze di centri e raggi rispettivi O1 , O2 e r1 , r2 . Dato un punto P
esterno alle due circonferenze si considerino le tangenti per P alle due circonferenze e siano M1 , N1 e M2 ,
N2 i rispettivi punti di contatto.
Si determini il luogo dei punti tali che P M1 2 + P M2 2 = 1.
Si determinino i punti P per cui P M1 2 + P M2 2 è minima.
1996.2
Il prezzo di mercato P di una certa merce dipende dalla quantità totale Q venduta secondo la
legge P = a − b Q, dove a e b sono due assegnati valori positivi.
Sul mercato operano solo due produttori, in concorrenza fra loro.
A regime, cioè quando nessuno dei due ha interesse a cambiare la quantità di merce da lui venduta, i due
produttori vendono rispettivamente le quantità X e Y di merce. Supponendo che la produzione avvenga a
costo zero, determinare X e Y .
18
I problemi della Scuola Normale
Testi
19
1996.3
Sia P un poliedro e siano F il numero delle facce, S il numero degli spigoli e V il numero
dei vertici di P. Sapendo che per il poliedro P, per la nota formula di Eulero, vale la relazione
F − S + V = 2,
(a) provare che P ha qualche faccia con meno di 6 lati;
(b) detto k il numero delle facce con meno di 6 lati, determinare il minimo valore possibile per k .
1997.1
Un cubo è appoggiato su un piano. Un bambino lo muove n volte, facendolo rotolare (senza
strisciare) ogni volta su uno dei lati della faccia su cui è appoggiato. Si suppone che la prima mossa sia
casuale e che, ad ogni mossa successiva, il bambino scelga casualmente di far rotolare il cubo su uno dei
due lati contigui al lato scelto in precedenza (vedi Figura: dopo aver rotolato sul lato a, il cubo rotolerà sul
lato b oppure sul lato c).
1996.4
Sia f (t) una funzione iniettiva definita sui numeri reali positivi. Dati x > 0 e y > 0, chiamiamo
f -Media di x e y l’unico numero z tale che
a
f (z ) =
f (x) + f (y )
c
b
.
2
√
Mostrare che la media geometrica x y e quella armonica 2 x y/(x + y ) sono delle f -Medie.
Fra le funzioni convesse f , individuare quelle per le quali la f -Media risulta minore o uguale della media
aritmetica.
1996.5 Dato un triangolo nel piano euclideo si indichi con I il centro della circonferenza in esso inscritta
e con Γ la circonferenza passante per I e per due qualunque dei vertici del triangolo.
Provare che il centro di Γ si trova sulla circonferenza circoscritta al triangolo.
1996.6 A partire da un cerchio C1 tracciare successivamente: un triangolo equilatero P1 inscritto in C1 , il
cerchio C2 inscritto in P1 , un quadrato P2 inscritto in C2 , il cerchio C3 inscritto in P2 , un pentagono regolare
P3 inscritto in C3 , e cosı̀ via, ottenendo in tal modo una successione
C1 ⊃ P1 ⊃ C2 ⊃ P2 ⊃ . . . Cn ⊃ Pn ⊃ Cn+1 ⊃ Pn+1 . . .
di cerchi e poligoni regolari concentrici, dove Pn ha n + 2 lati.
Mostrare che l’intersezione di tutti i cerchi Cn è un cerchio di raggio positivo. Il candidato può ricorrere
alla diseguaglianza, valida per ogni intero k ≥ 1:
1
1
1
1
+
+
+... < .
(k + 1)2 (k + 2)2 (k + 3)2
k
a) Dimostrare che se il cubo è tornato nella posizione iniziale (non necessariamente appoggiato sulla stessa
faccia) allora n è divisibile per 4.
b) Calcolare la probabilità p(n) che il cubo sia tornato nella posizione iniziale.
c) Dimostrare che
5
1
1
1
−
≤ p(4 k ) ≤
−
,
16 k 16 k 2
3 k 12 k 2
k ≥ 1.
1997.2
Si dice che un punto P esterno ad una circonferenza C “vede” la circonferenza sotto un angolo
α se l’angolo (contenente C ) compreso fra le tangenti a C condotte da P è uguale a α.
a) Data una circonferenza C e un angolo A di ampiezza α > 0, costruire il luogo dei punti del piano che
vedono C sotto l’angolo α.
b) Date due circonferenze C e C 0 esterne l’una all’altra, di centri O, O0 e raggi r, r0 rispettivamente,
costruire il luogo L dei punti del piano che vedono le due circonferenze sotto lo stesso angolo.
c) Dire (in termini dei dati) in che intervallo varia l’angolo di visuale al variare di P in L e quali sono i
punti in L dove tale angolo è minimo e massimo.
1997.3
Si determini, al variare dei parametri α e β interi pari e positivi, il numero di soluzioni reali
dell’equazione
(α + β ) x sin(π x) = x2 + α β .
20
I problemi della Scuola Normale
Anno 1906
21
1997.4
Sia dato un insieme finito Ω di punti distinti del piano tra loro collegati da un certo numero di
percorsi elementari congiungenti coppie di vertici distinti come esemplificato nel disegno seguente:
Dati due punti A e B di Ω un cammino che parte da A e termina in B è una successione di vertici
v0 , v1 , v2 , . . . , vn di punti di Ω tali che v0 = A, vn = B, e tale che vi e vi+1 sono congiunti da un percorso
elementare; in questo caso si dice che n è la lunghezza del cammino.
I punti, i percorsi elementari e i cammini soddisfano le seguenti proprietà:
i) i percorsi elementari non si incontrano fuori dai punti di Ω;
ii) dati due qualsiasi punti A, B ∈ Ω, esiste almeno un cammino che parte da A e termina in B ;
iii) c’è un particolare punto X ∈ Ω per il quale esiste un cammino che parte e termina in X avente lunghezza
dispari.
Si dimostri allora che esiste N intero positivo tale che, scelti due qualsiasi punti A e B di Ω, esiste un
cammino di lunghezza N che parte da A e termina in B .
1906.1*
Sono dati, in un piano, un circolo e due punti. Tirare una tangente al circolo in modo che la
somma delle distanze della tangente dai due punti dati sia uguale a una lunghezza data.
RISOLUZIONE
Indichiamo con A, B i due punti dati, con γ il cerchio e con d la lunghezza assegnata.
Procediamo nel modo seguente: dapprima caratterizziamo le rette r per cui risulta
dist(A, r) + dist(B, r) = d ;
[1]
successivamente vediamo se fra queste rette vi sono delle tangenti al cerchio γ .
E’ utile a questo punto, detto C il punto medio di AB , costruire il cerchio γ 0 di centro C e raggio d/2 (fig. 1).
r
γ0
A
1997.5
In ognuna delle tre figure seguenti è disegnato un poligono P . Si immagini che, in ognuno dei
tre casi, il poligono P sia costituito di materiale elastico e flessibile. Si chiede di disegnare o descrivere
sinteticamente la figura S che, in ognuno dei tre casi, si ottiene facendo combaciare le frecce dei lati che
hanno la stessa lettera, e di illustrare come questi lati appaiono in S .
c
a
d
P
a
b
γ
a
P
c
B
b
b
b
C
d
b
a
c
P
a
c
Figura 1
b
a
1997.6 √ Si determinino tutti gli interi positivi n che sono divisibili per tutti gli interi positivi minori o
uguali a n.
√
(Suggerimento: considerare il minimo comune multiplo dei numeri minori o uguali a n).
Se r è una retta per cui vale la [1], si presentano due possibilità: r non interseca AB oppure r interseca AB .
Caso (a)
Se r non interseca AB , per il teorema di Talete la distanza di C da r è pari alla semisomma
delle distanze di A e B da r, cioè dist(C, r) = d/2. Di conseguenza, la retta r risulta tangente al cerchio
0
γ . Cosı̀ un primo gruppo di soluzioni del problema si otterrà considerando tutte le tangenti comuni ai due
cerchi γ e γ 0 che non intersecano AB . È noto che, a seconda della posizione reciproca dei due cerchi, vi
possono essere 4, 3, 2, 1 oppure nessuna tangente comune.
Caso (b) Se r è una retta verificante la [1] che interseca AB , tracciamo le due rette r0 e r00 , parallele a r,
che passano rispettivamente per A e per B (fig. 2). Si ha allora
dist(r0 , r00 ) = dist(A, r) + dist(r, B ) = d .
0
Dunque r e r00 sono entrambe tangenti a γ 0 (in particolare A e B sono necessariamente esterni a tale
cerchio).
Costruito il rombo AHBK circoscritto a γ 0 , un altro gruppo di soluzioni del nostro problema sarà allora
costituito da quelle rette, tangenti a γ e parallele a uno dei lati del rombo AHBK , che intersecano il rombo
stesso. Evidentemente tali rette non possono essere più di quattro.
22
I problemi della Scuola Normale
Anno 1909
23
1909.1
In un piano sono dati due circoli che si tagliano in due punti. Si domanda di condurre per uno
di questi punti una retta sulla quale le corde staccate dai due circoli stiano fra loro in un rapporto dato.
H
r 00
C
r
γ0
A
Indichiamo con O1 e O2 i centri dei due cerchi, con P e P 0 le loro due intersezioni e con k
il rapporto assegnato. Sia r una retta passante per P che verifica la proprietà richiesta, tale cioè che, dette
A1 e A2 le sue ulteriori intersezioni con i due cerchi (fig. 1), risulti A1 P = k · A2 P .
Detti M1 e M2 i punti medi delle corde A1 P e A2 P , si ha allora
RISOLUZIONE
B
γ
K
r0
M1 P = k · M2 P .
Osserviamo che i segmenti M1 O1 e M2 O2 sono fra loro paralleli perché entrambi ortogonali a r. Condotta
per P la perpendicolare s a r, se A1 e A2 non coincidono con P 0 la retta per O1 e O2 interseca s in un punto
Q, e per il teorema di Talete
O1 Q = k · O2 Q .
[1]
Figura 2
Dal momento che i due gruppi di rette indicate comprendono tutte le possibili soluzioni del problema,
possiamo concludere che vi sono al più otto soluzioni.
Qualora sia AB < d, il problema si semplifica notevolmente, in quanto il caso (b) non si può verificare, e
le uniche soluzioni possibili sono le tangenti comuni a γ e γ 0 che non intersecano AB .
A1
P
M1
C
O1
γ0
γ0
A
A2
M2
H
Q
K
Figura 3
O2
P0
B
γ8
r
Figura 1
Viceversa, se Q è un punto della retta per O1 e O2 che verifica la [1], è sufficiente condurre per P la retta
perpendicolare al segmento P Q; le corde staccate dai due cerchi su tale retta stanno fra loro nel rapporto k .
Osserviamo che, se k 6= 1, vi sono due punti sulla retta per O1 e O2 che verificano la [1]; pertanto vi sono
due rette per P che risolvono il problema (fig. 1 e fig. 2).
OSSERVAZIONE
Non è facile assegnare condizioni esplicite su A, B , γ e d che assicurino la risolubilità del
problema o che ne indichino, più in generale, il numero di soluzioni. Si può però costruire, per ogni intero n
compreso fra 0 e 8, un esempio in cui il problema ha esattamente n soluzioni. I casi estremi (zero soluzioni
per il cerchio γ0 , otto soluzioni per il cerchio γ8 ) sono indicati nella Figura 3.
r
P
O1
O2
P0
Figura 2
Q
24
I problemi della Scuola Normale
0
0
Se k = 1, vi sono ancora due soluzioni: una è la retta per P e P (quindi A1 = A2 = P ), l’altra è la
perpendicolare in P al segmento che congiunge P con il punto medio di O1 O2 (vedi Figura 3).
1909.2*
Dimostrare che se a1 , a2 , . . . , an sono n numeri diversi da zero, fra i quali sussiste la relazione
2
a12 + a22 + · · · + an−
1
a22 + a32 + · · · + an2 =
i numeri stessi sono necessariamente in progressione geometrica.
P
P
r
O2
P0
25
(a1 a2 + a2 a3 + · · · + an−1 an )2 ,
r
O1
Anno 1911
O1
Q
RISOLUZIONE
L’espressione
∆ = (a1 a2 + a2 a3 + · · · + an−1 an )2 −
2
a12 + a22 + · · · + an−
1
O2
a22 + a32 + · · · + an2
è il discriminante (ridotto) dell’equazione di secondo grado:
2
2
a12 + a22 + · · · + an−
1 x −
2 (a1 a2 + a2 a3 + · · · + an−1 an ) x +
a22 + a32 + · · · + an2 = 0 .
Figura 3
[1]
Riordinando i termini l’equazione [1] si può porre nella forma
o anche
a12 x2 − 2 a1 a2 x + a22 + a22 x2 − 2 a2 a3 x + a32 + · · · +
2
2
an−1 x − 2 an−1 an x + an y 2 = 0
(a1 x − a2 )2 + (a2 x − a3 )2 + · · · + (an−1 x − an )2 = 0 .
Poiché il discriminante è nullo, la [1] ha un’unica soluzione x0 ; quindi risulta:
(a1 x0 − a2 )2 + (a2 x0 − a3 )2 + · · · + (an−1 x0 − an )2 = 0 .
Una somma di termini non negativi, però, si annulla solo se ciascun termine è nullo; pertanto
a2 = x0 a1 ,
a3 = x0 a2 ,
···
an = x0 an−1 ;
cioè i numeri a1 , a2 , . . . , an costituiscono una progressione geometrica di ragione x0 .
1911.1
Dividere un dato arco di cerchio in due archi le cui corde siano tra loro in un rapporto dato.
Il problema si può risolvere utilizzando il teorema della bisettrice: la bisettrice
condotta da un vertice di un triangolo divide il lato opposto in parti proporzionali ai lati adiacenti.
_
Sia AB l’arco assegnato e sia P il punto che divide il segmento AB in due parti AP e P B che stanno fra
_
loro nel rapporto dato. Indichiamo con M il punto medio di AB e con Q il punto opposto a M rispetto al
_
centro del cerchio (fig. 1). Il punto C , ottenuto intersecando l’arco AB con la retta per P e Q, risolve il
_
b , dal momento che i due archi AQ
problema: infatti la retta per C e Q è la bisettrice dell’angolo ACB
e
_
b
b
QB sono uguali per costruzione, e che quindi anche i rispettivi angoli alla circonferenza ACQ e QCB sono
uguali.
RISOLUZIONE
Dato un segmento AB , il luogo dei punti P tali che AP e P B stanno in un rapporto dato è
una circonferenza, detta circonferenza di Apollonio, di diametro P1 P2 , dove P1 e P2 sono i due
punti della retta per A e B , l’uno interno ad AB e l’altro esterno, che hanno distanze da A e B nel rapporto
dato.
_
Pertanto il punto C richiesto si può ottenere anche intersecando l’arco AB con la circonferenza di Apollonio
relativa ad AB e al rapporto dato (fig. 2).
OSSERVAZIONE
26
I problemi della Scuola Normale
C M
A
B
C
P
P2
A
P1
B
Anno 1912
27
Supponiamo dapprima che i lati AB e CD non siano paralleli; indichiamo con r, s le due semirette,
concorrenti in O, che contengono AB e CD (fig. 2). Per ogni punto P interno ad ABCD passa una e una
sola retta t che interseca r e s in due punti, H e L, simmetrici rispetto a P ; per poter risolvere il problema
assegnato occorre che la retta t tagli r e s all’interno dei segmenti AB e CD. La costruzione della retta t,
precedentemente descritta, mostra che ciò si verifica se e soltanto se il punto O0 , simmetrico di O rispetto a
P , cade all’interno del parallelogrammo delimitato dalle parallele a s passanti per A, B e dalle parallele a r
per C , D (vedi fig. 2, ove tale parallelogrammo è ombreggiato).
Q
C
L
Figure 1, 2
D
O
Z
D0
P
A
H
1912.2*
In un dato quadrilatero inscrivere un parallelogrammo il cui centro cada in un punto dato.
Supporremo che il quadrilatero assegnato ABCD risulti convesso e che il punto P sia interno
ad esso. Diremo che un parallelogrammo è inscritto in un quadrilatero se ha un vertice su ogni lato del
quadrilatero.
Sia HKLM un parallelogrammo che risolve il problema, e dunque ha il centro in P e i vertici H , K , L,
M rispettivamente sui lati AB , BC , CD, DA. Poiché le diagonali di un parallelogrammo si bisecano, per
individuare i punti H e L occorre tracciare per P una retta che intersechi AB e CD in due punti equidistanti
da P .
Se i lati AB e CD sono paralleli, il problema non ha soluzione, a meno che P non giaccia sulla retta parallela
e equidistante da AB e CD, nel qual caso ogni retta per P che intersechi i lati AB e CD determina punti H
e L accettabili.
Se AB e CD non sono paralleli, i loro prolungamenti si incontreranno in un punto O (fig. 1). Detto O0 il
simmetrico di O rispetto a P , i punti H e L sono necessariamente l’intersezione di AB con la parallela a
CD passante per O0 e l’intersezione di CD con la parallela ad AB passante per O0 .
Allo stesso modo si caratterizzano i punti K e M .
s
W
O0
B0
t
B
RISOLUZIONE
r
Figura 2
Ciò equivale, come è facile verificare, a dire che il punto P è interno al parallelogrammo S1 = D0 ZB 0 W ,
ove D0 e B 0 sono i punti medi dei lati AD e BC , mentre Z e W sono i punti medi delle diagonali AC e BD.
Nel caso speciale in cui i lati AB e CD sono paralleli, S1 si riduce al segmento B 0 D0 , e la condizione di
esistenza di H e L è ancora che P appartenga a S1 .
In modo analogo si può vedere che l’esistenza di due punti K e M su BC e AD, simmetrici rispetto a P , è
equivalente all’appartenenza di P al parallelogrammo S2 = A0 ZC 0 W , ove A0 e C 0 sono i punti medi di AB
e CD.
In conclusione, condizione necessaria e sufficiente affinché il problema dato abbia soluzione è che il punto
P appartenga al parallelogrammo S ottenuto intersecando S1 e S2 (vedi fig. 3, in cui tale parallelogrammo è
ombreggiato).
C
C0
C
D
L
O0
D
M
O
A
Z
D0
B0
K
P
W
A
H
A0
B
B
Figura 3
Figura 1
DISCUSSIONE La costruzione descritta sopra consente, quando è possibile, di determinare il parallelogrammo di centro P inscritto in ABCD; occorre ora determinare in quali casi tale parallelogrammo esiste
effettivamente.
Si noti che S contiene sempre almeno il segmento ZW , e degenera in tale segmento quando ABCD è un
trapezio (se poi ABCD è un parallelogrammo, ZW si riduce ad un punto).
All’infuori dei casi degeneri, la soluzione è unica.
28
I problemi della Scuola Normale
Anno 1926
29
1915.2*
Dimostrare che se per un punto interno ad una sfera si conducono tre piani, a due a due
perpendicolari, le somme delle aree dei tre cerchi che essi determinano sulla sfera è costante.
1923.2*
Dati due punti sopra le facce di un diedro, qual è la spezzata minima che li unisce e che ha un
vertice sullo spigolo?
Indichiamo con O e r il centro e il raggio della sfera; sia γ un cerchio giacente√
su di essa e
sia C il centro di γ . Usando il teorema di Pitagora si vede che il raggio di γ è uguale a r2 − OC 2 ,
e quindi l’area di γ è uguale a π (r2 − OC 2 ).
Sia ora P un punto interno alla sfera e siano α1 , α2 , α3 tre piani passanti per P e a due a due perpendicolari.
Indichiamo con γ1 , γ2 , γ3 i tre cerchi ottenuti intersecando la sfera con α1 , α2 , α3 e con C1 , C2 , C3 i rispettivi
centri. Si ha allora:
RISOLUZIONE
Siano α e β i due semipiani che costituiscono il diedro, A e B i punti dati, con A appartenente
ad α e B a β , r lo spigolo del diedro e M il vertice della spezzata che appartiene allo spigolo (fig. 1). La
spezzata minima richiesta deve consistere necessariamente dei due soli segmenti AM e M B ; si deve quindi
determinare M su r in modo che la somma AM + M B sia minima.
Sia β 0 il semipiano opposto a β e si consideri la rotazione che manda il semipiano α su β 0 ; essa manda il
punto A in un punto A0 di β 0 e, per ogni punto M di r, il segmento AM nel segmento A0 M . Risulta quindi
AM = A0 M e dunque AM + M B = A0 M + M B ed è chiaro per la disuguaglianza triangolare
che quest’ultima somma raggiunge il suo minimo valore quando A0 , M e B sono allineati.
Dunque la spezzata cercata è quella di vertici A, N , B , ove N è il punto di intersezione di r con il segmento
A0 B .
RISOLUZIONE
area(γ1 ) + area(γ2 ) + area(γ3 ) =
π 3 r2 − OC1 2 + OC2 2 + OC3 2 .
[1]
Supponiamo dapprima che i tre punti C1 , C2 , C3 siano distinti da O. Dato che il piano passante per O, C2 ,
C3 è parallelo al piano α1 (e analogamente per α2 , α3 ), il parallelepipedo di spigoli OC1 , OC2 , OC3 è un
parallelepipedo rettangolo avente come vertice opposto ad O il punto P . Si ha dunque:
OP 2 = OC1 2 + OC2 2 + OC3 2 .
B
[2]
M
N
A0
r
α2
α3
β0
γ2
γ3
C3
C2
P
A
β
α
Figura 1
O
C1
γ1
α1
1926.2*
Condurre un piano che seghi un dato angolo tetraedro in un parallelogrammo.
Siano O il vertice e r1 , r2 , r3 , r4 gli spigoli dell’angolo dato. Sia αij il piano passante per ri
e rj , i, j = 1, 2, 3, 4. Si può supporre che α13 lasci r2 e r4 in semispazi distinti. Sia s l’intersezione dei piani
α13 e α24 : proviamo che per ogni punto Q di s esiste un piano α che lo contiene e tale che l’intersezione
con l’angolo dato sia un parallelogrammo.
RISOLUZIONE
Figura 1
È facile verificare che la [2] è ancora valida quando qualcuno dei tre punti C1 , C2 , C3 coincide con O.
Dalla [1] e dalla [2] segue:
area(γ1 ) + area(γ2 ) + area(γ3 ) = π 3 r2 − OP 2 .
Questa uguaglianza mostra che la somma delle aree dei tre cerchi γ1 , γ2 , γ3 dipende solo da P , e non dalla
scelta della terna di piani α1 , α2 , α3 .
Per questo, basta costruire una retta r di α24 contenente Q, una retta t di α13 contenente Q in modo che Q
sia il punto medio dei segmenti staccati su r e t rispettivamente dalle rette r1 e r3 e dalle rette r2 e r4 (fig. 1):
il piano per r e t avrà allora la proprietà richiesta (infatti un quadrilatero le cui diagonali si intersecano nei
loro punti medi è un parallelogrammo).
Siamo ricondotti cosı̀ al seguente problema: date due rette r0 e r00 uscenti da un punto O e un punto Q fuori
di esse, costruire una retta s passante per Q e tale che Q sia il punto medio del segmento staccato da r0 e r00
su s.
Per risolvere questo problema (si veda anche 1912.2) basta costruire il punto R simmetrico di O rispetto a
Q (fig. 2); le parallele condotte da R alle due rette determinano assieme ad esse un parallelogrammo di cui
una diagonale è OR e l’altra giace sulla retta cercata.
30
I problemi della Scuola Normale
s
r1
r2
31
1929.2*
Verificare che il luogo dei punti M tali che la somma dei quadrati delle loro distanze da tre
punti dati A, B , C sia uguale ad una assegnata costante k 2 è un cerchio.
r3
r4
Q
t
Anno 1929
s
r 00
s
R
Q
r0
O
O
RISOLUZIONE
Facendo uso di un sistema di coordinate cartesiane ortogonali la verifica è immediata.
Un altro modo di risolvere il problema parte dall’osservazione che per valori di k troppo piccoli nessun
punto verifica la condizione richiesta ed il luogo è quindi vuoto. Cerchiamo di determinare k0 , il più piccolo
valore di k per cui ciò non accade, e supponiamo che O sia un punto del piano per cui OA2 + OB 2 + OC 2 = k02 ;
per ogni altro punto M risulterà M A2 + M B 2 + M C 2 ≥ OA2 + OB 2 + OC 2 . Se per P e Q generici punti del
−−
→
piano si indica con P Q il vettore applicato in P con estremo libero Q abbiamo, per ogni punto M :
−−→ −−→ −→
M A = M O + OA ,
−−→ −−→ −−→
M B = M O + OB ,
−−→ −−→ −−→
M C = M O + OC ,
da cui seguono le uguaglianze:
Figura 1
−−→ −→
M A2 = M O2 + OA2 + 2 M O · OA ,
−−→ −−→
M B 2 = M O2 + OB 2 + 2 M O · OB ,
→
−
−→ −−
M C 2 = M O2 + OC 2 + 2 M O · OC
1927.2
Determinare il luogo dei centri delle circonferenze ottenute segando una superficie sferica con
i piani passanti per un punto qualunque dello spazio.
Siano O il centro della sfera data, P il punto assegnato (che supporremo distinto da O) e P0
il punto medio del segmento OP (fig. 1).
Se Q è un punto del luogo cercato (dunque Q è centro del cerchio sezione della sfera con un piano α passante
per P ), il segmento OQ è perpendicolare al piano α. Inoltre il triangolo rettangolo P QO è inscritto nel
cerchio di centro P0 e raggio OP0 che giace nel piano passante per O e P e perpendicolare ad α. Ne segue
che P0 Q = P0 P , e allora ogni punto Q del luogo cercato è interno alla sfera data e appartiene alla superficie
sferica S di centro P0 e raggio P0 P .
Viceversa, se Q è interno alla sfera data e appartiene a S, il triangolo P QO è rettangolo in Q e quindi il
punto Q è il centro del cerchio ottenuto tagliando la sfera data con il piano passante per P e Q e ortogonale
a OQ.
RISOLUZIONE
Dunque il luogo cercato è costituito dalla parte della superfie sferica S di centro P0 e raggio P0 P che è
interna alla sfera data. Si noti che se il punto P è interno alla sfera data, detto luogo è costituito dall’intera
superficie sferica S.
( il punto · indica il prodotto scalare di due vettori ), e quindi
M A2 + M B 2 + M C 2 − OA2 + OB 2 + OC 2 =
−
−
→
−
−
→
−
→
−
−
→
3 M O 2 + 2 M O · ( OA + OB + OC ) .
Il secondo membro della [1] è sempre maggiore o uguale a 0 per qualunque scelta di M se scegliamo come
−→ −−→ −−→
O il punto individuato dalla equazione OA + OB + OC = 0, che è quindi il baricentro del triangolo
ABC . Si ha dunque k02 = OA2 + OB 2 + OC 2 ; per ogni k maggiore o uguale a k0 il luogo richiesto può essere
descritto da
1 2
M O2 =
k − OA2 + OB 2 + OC 2 ,
3
ed è quindi il cerchio di centro in O e raggio r dato da
r=
OSSERVAZIONE
O
P0
α
S
Figura 1
r
1 2
k − OA2 + OB 2 + OC 2 .
3
La proprietà enunciata vale anche nel caso in cui si ha un numero finito di punti A1 , A2 ,
. . . , An in luogo dei tre punti A, B , C . La formula [1] diviene allora
M A1 2 + · · · + M An 2 − OA1 2 + · · · + OAn 2 =
−−→ −−→
−−→
n M O2 + 2 M O · ( OA1 + · · · + OAn ) ,
Q
P
[1]
e ancora il punto O che rende minima la quantità M A1 2 + · · · + M An 2 è quello individuato dalla condizione
−−→
−−→
OA1 + · · · + OAn = 0, cioè il baricentro del sistema di punti A1 , . . . , An .
32
I problemi della Scuola Normale
Anno 1943
33
2
3
2
2
1939
È dato un rettangolo ABCD. Facendolo ruotare attorno al lato AB si ottiene un primo cilindro
e facendolo ruotare attorno al lato BC si ottiene un secondo cilindro. Si conosce la somma 4 π S 2 delle
superfici laterali dei due cilindri; e pure si conosce la somma π V 3 dei volumi degli stessi. Si domanda di
determinare i lati del rettangolo ABCD.
Qual è la condizione di realtà? Quante soluzioni ha il problema? Fissato V , qual è il massimo valore
possibile per S ?
Si ha allora 4 π S = 4 π x y e π V = π (x y + y x), cosicché il problema si riduce a risolvere il sistema
simmetrico:
RISOLUZIONE
Indichiamo con x e y rispettivamente il lato minore e quello maggiore di ABCD; le superfici
laterali dei due cilindri (fig. 1) sono uguali a 2 π x y e 2 π y x, mentre i corrispondenti volumi sono dati da
2
2
π x y e π y x.
con la condizione 0 < x ≤ y .
Le soluzioni del sistema [1] sono le radici positive dell’equazione di secondo grado

2
xy = S
3
x +y = V
S2
t2 −
C
D
[1]
V3
t + S2 = 0
S2
[2]
e tale equazione ha radici reali se e soltanto se (V 3 /S 2 )2 − 4 S 2 ≥ 0, cioè V 3 /S 2 ≥ 2 S , ovvero se risulta
V ≥
√
3
2S .
[3]
x
B
y
A
Se la condizione [3] è soddisfatta, le radici della [2] sono positive, essendo positivi la loro somma e il loro
prodotto.
Pertanto si può concludere che, se V ≥
√
3
2 S , il problema ammette la sola soluzione
p
x=
V 3 − V 6 − 4 S6 ,
2 S2
1 3 p 6
y=
V + V − 4 S6 ,
2 S2
1 Figura 1
Dalla condizione [3] si vede poi facilmente che, fissato V , il massimo valore possibile di S per cui il
problema ha soluzioni è
V
Smax = √
.
3
2
In corrispondenza di tale valore di S , il rettangolo ABCD è un quadrato.
1943
Determinare un triangolo rettangolo di cui siano noti il perimetro e il raggio del cerchio inscritto.
Una volta fissato il raggio del cerchio inscritto, qual è il valore minimo del perimetro?
RISOLUZIONE
Indichiamo con r il raggio del cerchio inscritto e con 2 p il perimetro del triangolo cercato e
consideriamo due semirette m e n fra loro perpendicolari uscenti da un punto A. Siano M e P due punti su
m, N e Q due punti su n tali che
AN = AM = r ,
AP = AQ = p ;
conduciamo le circonferenze C e C0 tangenti a m e n rispettivamente nei punti M , N e P , Q; sia γ il minore
degli archi che i punti P , Q individuano su C0 (fig. 1).
Se t è una tangente comune a C e a γ , essa individua sulle semirette m e n due punti B e C tali che il
triangolo ABC risolve il problema. Infatti C è la circonferenza inscritta in tale triangolo e, dall’uguaglianza
dei segmenti BR, BP e CR, CQ (ove R è il punto di tangenza fra t e γ ), si ha che il perimetro di ABC è
2 p.
Il problema pertanto avrà soluzione se l’arco γ è esterno alla circonferenza C oppure è tangente ad essa;
poiché i centri di C e C0 stanno sulla bisettrice di m
d
n, ciò avviene se e solamente se
r+r
√
√
2 ≤ p 2 −p,
√
che equivale a p ≥ r (3 + 2 2).
34
I problemi della Scuola Normale
√
Il valore minimo del perimetro 2 p è dunque 2 r (3 + 2 2); in tal caso C e γ risultano tangenti fra di loro e
il triangolo ABC è isoscele.
Si noti che tale costruzione si può effettuare per ogni scelta dell’angolo in A e quindi consente di costruire
un triangolo, dati un angolo, il perimetro e il raggio del cerchio inscritto.
n
Q
1961.4*
Con una bilancia a piatti e un certo numero di pesi, si vogliono pesare oggetti di peso inferiore
a 500 grammi, con un errore non superiore ad un grammo. Non si possono mettere pesi nel piatto su cui
poggia l’oggetto. Dire qual è il numero minimo di pesi sufficienti a tale scopo.
Poiché ci accontentiamo di un errore non superiore ad un grammo, è sufficiente scegliere i
pesi in modo da poter esprimere, con loro combinazioni, tutti i pesi corrispondenti ai numeri pari da 2 a
498.
È facile convincersi che a tale scopo sono sufficienti 8 pesi; per esempio i pesi da
RISOLUZIONE
Inoltre 8 è il numero minimo: infatti, disponendo di k pesi, se ne possono scegliere h (h ≤ k ) in
diversi; quindi con k pesi si possono ottenere al più
C
A M
35
2, 4, 8, 16, 32, 64, 128, 256 grammi.
γ
N
Anno 1962
R
C
B
P
m
Figura 1
p( k ) =
k X
k
h
k
h
modi
h=1
valori distinti. Ricordando la formula del binomio di Newton, abbiamo
k X
k
= (1 + 1)k = 2k
h
h=0
1960.3
È più facile, gettando una volta un dado, ottenere 6, oppure, gettandolo tre volte, ottenere tutte
e tre le volte un numero pari?
Supporremo che a ogni lancio ciascuna faccia del dado abbia la stessa probabilità di uscire.
La probabilità che in un lancio esca un determinato numero è quindi 1/6, perché vi sono 6 risultati
possibili equiprobabili. Dunque la probabilità che al primo lancio esca il numero 6 è 1/6, mentre la
probabilità che esca un numero pari è 1/2 in quanto a tale risultato si perviene in 3 modi distinti sui 6
risultati possibili. Poiché è ragionevole supporre che risultati di lanci successivi siano tra loro indipendenti,
si ottiene che la probabilità che in tre lanci esca sempre un numero pari è
RISOLUZIONE
1 1 1 1
· · = .
2 2 2 8
È quindi più probabile ottenere 6 (o una qualsivoglia determinata faccia del dado) in un solo lancio che
ottenere un numero pari per 3 lanci consecutivi.
e quindi p(k ) = 2k − 1.
Se k ≤ 7 si ha p(k ) ≤ 127; siccome gli oggetti da pesare hanno peso compreso fra 0 e 500 grammi e l’errore
massimo consentito (per difetto o per eccesso) è di un grammo, qualora bastassero 7 pesi occorrerebbe
poter ricoprire un intervallo di lunghezza 500 con non più di 127 intervalli di lunghezza al più 2. Ciò è
ovviamente impossibile, in quanto 2 · 127 < 500.
1962.5*
Si sostiene talvolta che noi usiamo il sistema decimale di numerazione (per cui, ad esempio,
362 significa 3 · 102 + 6 · 10 + 2) in quanto abbiamo 10 dita.
Un marziano, dopo aver visto scritta l’equazione:
x2 − 16 x + 41 = 0 ,
invitato a scrivere la differenza delle radici scrive 10.
Quante dita hanno i marziani?
(N.B. Per i numeri compresi fra 0 e 6 la scrittura dei marziani coincide con la nostra).
RISOLUZIONE
Supponiamo che i marziani abbiano N dita e quindi usino il sistema di numerazione in base
N . Possiamo subito osservare che N > 6, in quanto i simboli 0, 1, 2, 3, 4, 5 e 6 hanno lo stesso significato
per noi e per i marziani. Inoltre i simboli 10, 16 e 41 che per noi indicano rispettivamente il numero delle
nostre dita, il numero delle nostre dita aumentato di 6, e quattro volte il numero delle nostre dita aumentato
di 1 indicheranno per i marziani rispettivamente i numeri
N , N + 6, 4N + 1 .
D’altra parte l’algoritmo risolutivo delle equazioni di secondo grado (come tutte le regole algebriche) è
indipendente dal sistema di numerazione prescelto; pertanto le due radici dell’equazione
x2 + b x + c = 0
hanno in ogni caso differenza uguale a
δ=
p
b2 − 4 c .
Nel nostro caso dovrà dunque essere N =
p
(N + 6)2 − 4 (4 N + 1), da cui si ricava
36
I problemi della Scuola Normale
N = 8.
I marziani hanno dunque 8 dita (anche se, invitati a scrivere il numero delle loro dita, scriverebbero “10”).
1963.5
Un podista si trova su un punto della Terra, che supponiamo perfettamente sferica. Percorre
un chilometro verso nord, poi uno verso est e infine uno verso sud. Si ritrova al punto di partenza.
Quali sono i punti di partenza che obbediscono a questa condizione?
RISOLUZIONE
Si osserva subito che il polo sud obbedisce alla condizione richiesta.
Sia P un altro di tali punti. Per prima cosa, esso deve distare più di un chilometro dal polo nord e, in
particolare, non può essere il polo nord.
Indichiamo con A il punto un chilometro a nord di P , con B il punto un chilometro ad est di A, cosicché il
percorso del podista avviene lungo la “spezzata” P ABP .
I tratti P A e BP giacciono sul meridiano passante per P (fig. 1), hanno uguale lunghezza e sono percorsi
in verso opposto; pertanto A coincide con B e quindi il percorso AB è un cammino chiuso, consistente in
un intero parallelo (percorso una o più volte).
Si conclude allora che, oltre al polo sud, i soli punti che verificano la condizione richiesta sono i punti che
giacciono sui paralleli Cp tali che il parallelo un chilometro a nord di Cp abbia lunghezza di 1/p chilometri,
per p = 1, 2, . . . .
Anno 1967
37
1967.2*
Sono assegnate tre rette parallele. Esiste un triangolo equilatero con i vertici rispettivamente
sulle tre rette?
RISOLUZIONE
Indichiamo le tre rette con a, b, c, supponendo che a sia interna alla striscia delimitata
dalle altre due. Per dare risposta (affermativa) al quesito, utilizzeremo l’osservazione che un triangolo è
equilatero se e solo se una rotazione di 60◦ gradi attorno ad un vertice trasforma uno degli altri due vertici
nel terzo.
Fissiamo dunque, in modo arbitrario, un punto B sulla retta b ed eseguiamo una rotazione di 60◦ attorno
a B , indicando con a0 la trasformata della retta a e con C l’intersezione fra a0 e c. Dato che C appartiene
alla retta a0 , esso è il trasformato, tramite la rotazione considerata, di un punto A appartenente ad a. Per
determinare A basta intersecare a con il cerchio di centro B e raggio BC , scegliendo delle due intersezioni
quella che sta dalla stessa parte di B rispetto ad a0 (fig. 1). Il triangolo ABC risolve il problema.
Poiché le rotazioni di 60◦ che si possono eseguire attorno a B sono due, a seconda del verso scelto, i triangoli
equilateri con un vertice in B , un vertice sulla retta a e il terzo vertice sulla retta c sono due. Essi sono
simmetrici fra loro rispetto alla perpendicolare alla retta b in B . Nel caso speciale in cui a è la bisettrice
della striscia (b, c), i due triangoli hanno il lato BC in comune e perpendicolare alle tre rette.
C
A=B
P
c
A
a
b
B
a0
Figura 1
Un secondo modo di costruire il triangolo cercato è il seguente. Fissato un punto B di b, si
tracci da B una delle due rette, formanti con le tre parallele un angolo di 30◦ (fig. 2). Sia poi D l’intersezione
fra r e c, e sia A l’intersezione di a con l’asse del segmento BD. Il cerchio γ avente centro in A e raggio
b = 60◦ (in
uguale a AB interseca la retta b, oltre che in D, in un secondo punto C . Poiché AB = AC e B AC
b e B DC
b sono rispettivamente l’angolo al centro e l’angolo alla circonferenza relativi al cerchio
quanto B AC
_
γ e all’arco BC ) si può concludere che ABC è un triangolo equilatero.
OSSERVAZIONE
Figura 1
38
I problemi della Scuola Normale
γ
c
D
a
A
b
B
39
1968.3
Sono dati in un piano quattro punti A, B , C , D, in modo che A, B , C e A, B , D siano vertici
di triangoli equilateri distinti. Determinare tutte le circonferenze β che godono della seguente proprietà: i
quattro punti A, B , C , D hanno dalla circonferenza β ugual distanza.
r
C
Anno 1968
Il quadrilatero ADBC è costituito dai due triangoli equilateri ABC e ABD simmetrici
rispetto alla retta per A e B .
Sia β una circonferenza equidistante dai punti A, B , C , D e siano O e r il suo centro e il suo raggio.
Osserviamo intanto che β non può passare per nessuno dei quattro punti, perché in tal caso dovrebbe passare
per tutti, ma il quadrilatero ACBD non è inscrittibile in una circonferenza.
I punti A, B , C , D non possono neppure trovarsi tutti da una stessa parte rispetto alla circonferenza β (tutti
interni o tutti esterni ad essa): ne seguirebbe infatti che essi sono equidistanti dal centro di β .
Si danno allora due casi:
(1) tre punti sono da una parte di β e uno dall’altra;
(2) due punti sono interni a β e due sono esterni.
RISOLUZIONE
La distanza di un punto P da β è r − OP se P è interno a β , OP − r se P è esterno.
Nel caso (1), supponendo ad esempio che A, B , C stiano dalla stessa parte, si deve avere:
Figura 2
AO − r = BO − r = CO − r = r − DO ;
quindi O è il circocentro del triangolo ABC e β ha raggio
AO + DO
√
3
AB .
2
2
Le circonferenze di questo tipo sono dunque quattro, a seconda che il punto isolato sia D, C , B oppure A;
oltre alla circonferenza sopra descritta si ha quindi la sua simmetrica rispetto ad AB , la circonferenza che
ha centro in B (circocentro del triangolo ACD) e raggio r = AB/2 e infine la simmetrica di quest’ultima
rispetto alla retta per C e D.
r=
=
Nel caso (2), i due punti interni a β devono essere equidistanti dal suo centro O, e cosı̀ pure i due punti
esterni; dunque O appartiene all’asse dei due punti interni e all’asse dei due punti esterni. Poiché gli assi
dei segmenti AD e BC sono paralleli e distinti, e lo stesso accade per gli assi dei segmenti AC e BD, la
sola possibilità è che O sia il punto di incontro degli assi di AB e CD, cioè il centro del rombo ACBD
(fig. 1). Sarà allora:
AO − r = BO − r = r − CO = r − DO ;
da cui
r=
AO + CO
2
=
√
3+1
AB .
4
C
A
O
D
Figura 1
B
40
I problemi della Scuola Normale
Abbiamo cosı̀ trovato cinque circonferenze distinte che hanno la proprietà richiesta; dall’analisi fatta segue
chiaramente che non ve ne sono altre.
Anno 1970
41
1970.4
Fissato un intero positivo n, determinare il più piccolo intero m tale che, presi comunque m
numeri interi positivi, una almeno delle seguenti eventualità si verifichi:
(a) tra gli m numeri considerati ve ne sono n uguali;
(b) tra gli m numeri considerati ve ne sono n distinti.
RISOLUZIONE
Consideriamo un insieme S di numeri che non verifica nessuna delle due proprietà. In S vi
sono allora non più di (n − 1) tipi diversi di numeri, altrimenti (b) sarebbe soddisfatta, e di ciascun tipo ve
ne sono non più di (n − 1), in caso contrario sarebbe verificata la (a). Dunque S non può contenere più di
(n − 1) · (n − 1) = (n − 1)2 elementi.
Osserviamo però che deve risultare
m > (n − 1)2 ;
è facile infatti costruire un insieme di (n − 1)2 numeri che non contiene n numeri uguali né contiene n
numeri diversi fra loro: basta prendere (n − 1) classi ciascuna contenente (n − 1) numeri uguali fra loro, in
modo che elementi di classi diverse siano diversi. Ad esempio:
1, 1, . . . , 1 , 2, 2, . . . , 2 , . . . , (n − 1), (n − 1), . . . , (n − 1) .
|
{z
}
|
{z
}
(n − 1) volte (n − 1) volte
|
{z
(n − 1) volte
}
Dunque, il più piccolo m per cui è certamente verificata una delle eventualità (a), (b) è dato da (n − 1)2 + 1.
42
I problemi della Scuola Normale
Anno 1971
1971.4*
Una palla si trova su un biliardo in posizione P . Provare che esiste almeno una direzione
secondo cui si può lanciare la palla in modo che essa non ripassi mai per la posizione P .
Si consideri il biliardo privo di attrito e si supponga che il rimbalzo alle sponde obbedisca alla stessa legge
di riflessione della luce.
Supponiamo il biliardo rettangolare, con lati di lunghezza a e b; introduciamo un sistema di
assi cartesiani ortogonali aventi origine O nel centro del biliardo e assi paralleli ai lati.
Indichiamo con P (t) la posizione della palla all’istante t, con P = P (0) la posizione iniziale, con A(t) e
B (t) le proiezioni di P (t) rispettivamente sull’asse x e sull’asse y e infine con νx e νy le componenti della
velocità iniziale lungo le direzioni degli assi (fig. 1).
RISOLUZIONE
con α irrazionale;
in tal caso, infatti, la [1] non è soddisfatta per nessuna scelta degli interi √
m e n, e dunque la palla non ritorna
mai nella posizione iniziale (ad esempio si può prendere νx = a, νy = b 2).
Caso generale Siano ora x0 e y0 le coordinate del punto iniziale P . Il punto A(t) ritornerà in A(0), di
ascissa x0 , dopo aver effettuato o un numero pari o un numero dispari di rimbalzi sulle sponde verticali. La
prima eventualità si verifica negli istanti
t=
2na
νx
,
n = 1, 2, . . . ;
la seconda negli istanti
y
t=
B(t)
43
b
νy
=α
νx
a
νy
P(t)
A(t)
a
2
νx
− x0
=
(2 n + 1) a − 2 x0
νx
,
n = 0, 1, 2, . . . ;
Analogamente il punto B (t) ritorna nella posizione iniziale B (0) negli istanti
νx
t=
O
2na + 2
x
2mb
νy
,
m = 1, 2, . . .
e
t=
(2 m + 1) b − 2 y0
νy
,
m = 0, 1, 2, . . . .
Pertanto la palla ritorna nella posizione iniziale P se e soltanto se esistono due interi m ed n che verificano
una delle condizioni seguenti
n a νy = m b νx ,
[1]
2 n a νy = [(2 m + 1) b − 2 y0 ] νx ,
[2]
[(2 n + 1) a − 2 x0 ] νy = 2 m b νx ,
[3]
[(2 n + 1) a − 2 x0 ] νy = [(2 m + 1) b − 2 y0 ] νx .
[4]
Figura 1
Se νx = 0, oppure νy = 0, la palla ripassa certamente per P . A meno di invertire la direzione di uno o di
entrambi gli assi, possiamo supporre
νx > 0,
νy > 0 .
Osserviamo che, durante il moto, la componente della velocità lungo l’asse x si mantiene costante in modulo
e cambia segno a ogni rimbalzo lungo i lati verticali del biliardo; allo stesso modo, la componente lungo
l’asse y è costante in modulo e cambia segno a ogni rimbalzo lungo i lati orizzontali.
Caso semplice Trattiamo dapprima il caso in cui la posizione iniziale P è l’origine, dunque A(0) = B (0) =
O.
Il punto A(t) coinciderà nuovamente con O se e soltanto se tale punto ha percorso una distanza che è
multipla di a, il che avviene negli istanti
t=
na
,
νx
n = 1, 2, . . . ;
2 n a νy = [(2 m + 1) b − 2 y0 ] νx ,
analogamente B (t) coinciderà con B (0) = O negli istanti
t=
mb
,
νy
2 n0 a νy = [(2 m0 + 1) b − 2 y0 ] νx ,
m = 1, 2, . . . .
da cui, sottraendo membro a membro,
Pertanto la palla ritorna nella posizione iniziale se e soltanto se esistono degli interi m, n tali che
n a νy = m b νx .
Scegliamo la velocità iniziale, come nel caso precedente, in modo che νy /νx non sia razionalmente dipendente da b/a e mostriamo che, con tale scelta, la palla passa al più tre sole volte da P . Ne seguirà la tesi, in
quanto la direzione dell’ultimo passaggio in P ovviamente non dà luogo a ulteriori passaggi per P .
Per la scelta effettuata di νy /νx , la [1] non si può verificare.
Se esistessero due distinte coppie di interi n, m e n0 , m0 per le quali è soddisfatta la condizione [2], si
avrebbe
[1]
La condizione [1] equivale a dire che i numeri νy /νx e b/a sono “razionalmente dipendenti”, cioè vi è un
numero razionale m/n che moltiplicato per il primo dà il secondo.
Osservando ora che assegnare la direzione iniziale equivale ad assegnare il rapporto νy /νx , da quanto detto
si ricava che esistono infinite direzioni iniziali che verificano la condizione richiesta: basta che sia
2 (n − n0 ) a νy = 2 (m − m0 ) b νx
e dunque νy /νx e b/a sarebbero razionalmente dipendenti.
Allo stesso modo si prova che non vi possono essere due coppie distinte n, m e n0 , m0 che verificano la
condizione [3], né che verificano la condizione [4].
Dunque [2], [3], [4] possono, ciascuna, essere soddisfatte al più una volta e dunque la palla ritornerà nella
posizione iniziale al più 3 volte.
44
I problemi della Scuola Normale
Si può facilmente verificare che se le condizioni [2] e [3] sono entrambe soddisfatte allora
è soddisfatta anche la [4]. Inoltre, se si verifica la [4] per una certa coppia di interi n e m, allora la traiettoria
della palla passa per un vertice del rettangolo. Le traiettorie che non sono “razionalmente dipendenti” dai
lati del biliardo (cioè con νy /νx e b/a non razionalmente dipendenti) e che non passano per i vertici possono
al più verificare, e un sola volta, una delle condizioni [2] o [3].
OSSERVAZIONE 1
Un altro metodo per risolvere il problema, più conciso ma che fa uso di nozioni non
elementari sulla cardinalità degli insiemi, è il seguente.
Si osserva anzitutto che ad ogni rimbalzo sulle pareti, poiché l’angolo di incidenza è uguale all’angolo di
riflessione, il percorso riflesso è simmetrico del prolungamento del percorso incidente rispetto al lato su cui
avviene il rimbalzo (fig. 2).
Dunque se si costruiscono le immagini simmetriche del biliardo rispetto a tutti i suoi lati, e successivamente si
costruiscono i simmetrici di tali immagini, infinite volte, attorno ai loro lati, si ottiene un “tappezzamento” del
piano con rettangoli delle stesse dimensioni del biliardo e in ciascuno di essi risulta individuata l’immagine
del punto di partenza (fig. 3).
Per l’osservazione precedente, il percorso è rappresentato da una semiretta t uscente dal punto iniziale P ;
pertanto la palla tornerà in P se e soltanto se la semiretta t passa per una delle immagini di P stesso.
Ma l’insieme delle immagini simmetriche di P è in corrispondenza biunivoca con l’insieme dei numeri
naturali, cioè è un insieme numerabile; mentre l’insieme delle semirette uscenti da P non è numerabile
(esso è in corrispondenza biunivoca, ad esempio, con l’insieme dei punti della circonferenza di raggio
unitario e centro P , dunque con l’insieme dei numeri reali compresi fra 0 e 2 π , il quale non è numerabile).
Dunque esiste almeno una (anzi, infinite) semiretta uscente da P che non passa per alcuna delle immagini
simmetriche di P ; il corrispondente percorso della palla sul biliardo non ritorna dunque mai in P .
OSSERVAZIONE 2
Anno 1974
45
1973.3*
Un treno parte da Pisa. Il macchinista controlla il cronometro e nota che la lancetta dei
secondi è sullo zero. Dopo aver percorso 8 chilometri, il macchinista controlla di nuovo il cronometro e
nota che la lancetta dei minuti copre esattamente quella delle ore. La velocità media del treno per gli 8
chilometri percorsi è di 33 chilometri l’ora. A che ora è partito il treno da Pisa?
RISOLUZIONE
Osserviamo subito che le informazioni in nostro possesso non ci permetteranno di stabilire
se il treno è partito prima o dopo mezzogiorno. Supponiamo quindi che il treno sia partito il mattino, e
precisamente p minuti dopo la mezzanotte, con p intero compreso fra 1 e 12 · 60 = 720. Per percorrere 8
chilometri alla velocità media di 33 km/h, il treno impiega 8 · 60/33 minuti; perciò, quando il macchinista
guarda nuovamente il cronometro, sono trascorsi p + 160/11 minuti dalla mezzanotte.
D’altra parte, la lancetta dei minuti si sovrappone a quella delle ore 11 volte ogni mezza giornata, cioè
una volta ogni 12 · 60/11 minuti. Allora, nell’istante in cui il macchinista guarda per la seconda volta il
cronometro, sono trascorsi k · 720/11 minuti da mezzanotte, con k intero.
Deve essere quindi, per p, k interi e 1 ≤ p ≤ 720:
160
720
k =p+
,
11
11
da cui si ricava
80 (9 k − 2) = 11 p .
[1]
Da 1 ≤ p ≤ 720 segue
11 ≤ 720 k − 160 ≤ 720 · 11 ,
e quindi, dovendo essere k intero, necessariamente
1 ≤ k ≤ 11 .
t
P
P
[2]
D’altra parte, per la [1], il numero 9 k − 2 deve essere divisibile per 11; ma l’unico numero k che verifica
la [2] e tale che 9 k − 2 sia divisibile per 11 è k = 10, da cui, per la [1], p = 640. Quindi il treno è partito
alle 10 : 40.
1974.5
forma
Dati tre numeri interi a, b, c aventi massimo comun divisore 1, verificare che i numeri della
a m2 + b m + c ,
Figure 2, 3
Di fronte alla proprietà geometrica espressa dall’esercizio in esame, si possono studiare
interessanti proprietà di tipo statistico del moto ideale di una palla di biliardo. Ad esempio si può dimostrare
che ogni traiettoria che partendo da P non vi ritorna passa infinite volte in un intorno di raggio arbitrariamente
piccolo di ogni punto del biliardo, e in particolare di P . Vengono per questo dette traiettorie quasi-periodiche
(o ergodiche); si può dimostrare che le traiettorie quasi-periodiche sono “quasi tutte”, in un senso che, per
essere precisato matematicamente, richiede un considerevole apparato, ma che è intuitivamente chiaro in
termini statistici, se si tiene presente l’osservazione 2; le traiettorie che partono da P e vi ritornano formano
un insieme numerabile, quelle che non vi tornano formano un insieme infinito più che numerabile e sono
quindi “molte di più”.
OSSERVAZIONE 3
con m intero qualunque, non possono essere tutti divisibili per 14.
Generalizzare il risultato.
RISOLUZIONE
Dimostriamo direttamente che il risultato vale non solo per il numero 14 ma per qualunque
intero N ≥ 3. Ragioniamo per assurdo, distinguendo due casi.
Caso (a) Se N ha un divisore dispari d > 1, allora scegliendo m = 0 si ottiene che c è divisibile per d
e quindi anche a m2 + b m lo è, per qualunque m. In particolare (m = 1 e m = −1) i numeri a + b e a − b
risultano divisibili per d, cosı̀ come la loro somma 2 a e la loro differenza 2 b. Ma allora, essendo d dispari,
anche a e b hanno d per fattore, il che è impossibile.
Caso (b) Se N è una potenza di 2 non inferiore a 4, cioè N = 2k con k ≥ 2, si può ragionare come per il
caso precedente e si ottiene che tutti e tre i numeri a, b, c sono divisibili per 2k−1 .
Si osservi che per N = 2 il risultato è falso: ad esempio, se a = b = 1 e c = 2, si ha che m2 + m +2 = m (m +1)+2
è sempre pari.
46
I problemi della Scuola Normale
Anno 1977
47
Bk+1 Bk
Ck+1
1976.3
Sia n un intero maggiore di 2 e sia ∆ un triangolo rettangolo. Dimostrare che la potenza
n-esima della lunghezza dell’ipotenusa di ∆ è maggiore della somma delle potenze n-esime dei cateti.
Siano x, y , z rispettivamente le lunghezze dei cateti e dell’ipotenusa; quindi
x
y
< 1,
< 1.
z
z
Pertanto, se n > 2, si ha:
x n x 2
y n y 2
<
,
<
z
z
z
z
RISOLUZIONE
e quindi
xn + y n x n y n x 2 y 2 x2 + y 2
=
= 1.
+
<
+
=
zn
z
z
z
z
z2
Figura 1
Osserviamo che è C1 ≡ B1 , e che la distanza tra le proiezioni dei centri del primo e del settimo disco è
dunque B1 C7 . Ma la lunghezza di B1 C7 è non superiore a quella della spezzata B1 B2 B3 B4 B5 B6 C7 , quindi
1977.1*
Su un tavolo orizzontale vi è una pila di 7 dischi metallici perfettamente uguali, ognuno di
diametro 40 cm.
Dire quale è la distanza massima sul piano orizzontale che può avere il centro del disco più alto dal centro
del disco più basso senza che la pila crolli.
Indichiamo con Ck la proiezione, sul piano del tavolo, del centro del k -esimo disco (dall’alto),
e con Bk la proiezione del baricentro del gruppo dei primi k dischi (sempre dall’alto).
RISOLUZIONE
Perché la pila stia in equilibrio occorre che il baricentro dei primi k dischi stia sopra il (k + 1)-esimo disco,
e cioè, dato che il raggio di ogni disco è 20 cm:
Bk Ck+1 ≤ 20
con k = 1, 2, . . . , 6 .
[1]
1
1
= 49
C1 C7 ≤ 20 1 + + · · · +
2
Mostriamo che è possibile costruire una pila per cui C1 C7 = 49: se i punti C1 , C2 , . . . , C7 sono allineati
(in quest’ordine), e se C1 C2 = 20, C2 C3 = 20/2, . . . , C6 C7 = 20/6, è facile verificare che le [1] e le [2]
diventano tutte delle uguaglianze, e che C1 C7 = 49.
Si può generalizzare facilmente il risultato a k dischi di raggio r: per essi risulta
1
C1 Ck ≤ r 1 + + · · · +
2
Noti i punti Bk e Ck+1 il punto Bk+1 si ricava cosı̀: esso sta sul segmento Bk Ck+1 (fig. 1) ed è
Bk Bk+1 =
1
k+1
Bk Ck+1 .
Allora le condizioni per l’equilibrio della pila si traducono in

1


B1 B2 ≤
20


2




1


20
 B2 B3 ≤

3
...





1


B5 B6 ≤
20


6



B6 C7 ≤ 20
6
per ogni pila di dischi in equilibrio (fig. 2).
[2]
Figura 2
1
k−1
.
48
I problemi della Scuola Normale
1979.1*
Un battello scende lungo un fiume; sia alla partenza sia a ogni stazione intermedia salgono
sul battello tanti passeggeri, ognuno diretto ad una diversa stazione, quante sono le fermate successive.
Sapendo che il numero massimo di passeggeri contemporaneamente presenti sul battello è 380, si determini
il numero delle stazioni.
Sia n il numero delle stazioni, compresa la stazione di partenza. Il battello parte con n − 1
passeggeri a bordo: alla prima stazione ne scende uno e ne salgono n − 2; sul battello vi sono ora 2 (n − 2)
passeggeri. Alla stazione seguente ne scendono 2 e ne salgono n − 3, e il numero dei passeggeri passa a
3 (n − 3). Per ricorrenza si deduce che il battello riparte dalla k -esima stazione con k (n − k ) passeggeri.
Ora osserviamo che il prodotto p q di due interi positivi aventi somma assegnata n è massimo quando risulta:
RISOLUZIONE
p=
p=
n
2
,
q=
n+1
2
,
q=
n
se n è pari;
2
n−1
2
Nel nostro caso si ha k (n − k ) = 380; poiché 380 non è della forma n2 /4 per alcun n, mentre invece si ha
380 = 19 · 20, si conclude che
e n − k = 19
(o viceversa) e quindi si ha n = 39.
1979.3
Per un punto P passano tre superfici sferiche distinte tra loro. Si considerino le affermazioni
seguenti:
(a) nessuna retta passante per P è tangente a tutte e tre le sfere;
(b) nessuna sfera è tangente a un’altra;
(c) esiste un altro punto Q comune alle tre superfici sferiche.
Dire, per ogni coppia di affermazioni, se esse sono incompatibili, se sono equivalenti o se una delle due
implica l’altra.
RISOLUZIONE
49
Da (b) non segue (c). Vediamo un controesempio: l’intersezione di S1 e S2 è una circonferenza γ per P , e
possiamo scegliere S3 in modo che sia tangente a γ in P ; allora le tre sfere hanno in comune il solo punto P
(e dunque non verificano l’ipotesi (c)), ma nessuna di esse è tangente a nessun’altra (e dunque vale l’ipotesi
(b)).
Invece, da (c) segue immediatamente (b). Ragionando per assurdo, se due sfere sono tangenti la loro
intersezione è costituita da un solo punto, e quindi le tre sfere hanno in comune il solo punto P , per cui
l’ipotesi (c) non è verificata.
Abbiamo cosı̀ dimostrato che:
dall’affermazione (a) segue la (b) ma dalla (b) non segue la (a);
dall’affermazione (a) segue la (c) ma dalla (c) non segue la (a);
dall’affermazione (c) segue la (b) ma dalla (b) non segue la (c).
se n è dispari.
Infatti, posto p = n/2 − δ , q = n/2 + δ , l’espressione p q = n2 /4 − δ 2 è tanto più grande quanto più δ è piccolo.
k = 20
Anno 1980
Siano S1 , S2 , S3 le tre sfere passanti per P e siano α1 , α2 , α3 i rispettivi piani tangenti in P .
Proviamo che (a) implica (b).
Supponiamo infatti, per assurdo, che valga (a) e non valga (b) e siano, ad esempio, S1 e S2 tangenti in P .
Allora α1 = α2 e l’intersezione di α1 , α2 , α3 contiene almeno una retta per P , che risulta tangente alle tre
sfere, contro l’ipotesi.
Da (b) però non segue (a): basta per questo considerare tre sfere per P tali che i tre piani α1 , α2 , α3 che pur
essendo distinti (e quindi nessuna sfera è tangente ad un’altra) abbiano in comune una retta r, che quindi è
tangente alle tre sfere.
Dunque (a) implica (b) (in particolare (a) e (b) sono compatibili), ma le due affermazioni non sono
equivalenti.
Da (a) segue (c). Supponiamo infatti che valga (a); per quanto già dimostrato, vale anche (b), e di
conseguenza Sia α il piano di γ ; α è distinto da α3 , altrimenti la retta tangente a γ in P sarebbe tangente
alle tre sfere. Pertanto l’intersezione di α con S3 è un cerchio γ1 passante per P , che non è tangente a γ
(altrimenti la retta tangente a γ e a γ1 sarebbe tangente alle tre sfere) e quindi γ1 ha con γ , oltre a P , un altro
punto in comune, Q, che appartiene alle tre sfere. Ciò prova (c).
Da (c) non segue però (a). In un piano π prendiamo una circonferenza γ , e sia P un punto di γ ; se scegliamo
tre sfere distinte S1 , S2 e S3 contenenti γ , queste verificano la condizione (c) perché hanno in comune infiniti
punti distinti da P , ma non verificano (a) perché la retta tangente a γ in P è tangente a tutte e tre le sfere.
1980.6
Un’autostrada ha n caselli a distanze successive di p chilometri. Si è osservato che ogni
macchina entra con uguale probabilità da ogni casello e esce con uguale probabilità da un altro casello.
Trovare la lunghezza del percorso medio di ogni macchina.
Per ragioni di simmetria basterà tener conto delle sole vetture che percorrono l’autostrada in
una direzione; poiché ogni percorso fra gli n caselli è equiprobabile, il percorso medio è la media aritmetica
delle lunghezze di tutti i percorsi possibili.
Dal primo casello sono possibili (n − 1) percorsi, dal secondo (n − 2) e cosı̀ via; dunque il numero dei
percorsi possibili è
RISOLUZIONE
1 + 2 + · · · (n − 2) + (n − 1) =
n (n − 1)
2
,
e la media aritmetica delle loro lunghezze è
pn =
1 + [1 + 2] + · · · + [1 + 2 + · · · + (n − 1)]
p.
n (n − 1)/2
Osserviamo che la precedente relazione si può porre nella forma
pn =
1+
1 + · · · + [1 + 2 + · · · + (n − 2)] n − 2
(n − 1) (n − 2)/2
n
p
da cui segue che
pn = p + pn−1
n−2
.
n
[1]
Poiché, ovviamente, si ha p2 = p, dalla [1] si ricava p3 = p + p/3 = 4 p/3, p4 = p + 2 p/3 = 5 p/3. Proviamo
per induzione che è
pn =
n+1
3
p.
[2]
Supponiamo dunque che per un certo n0 si abbia pn0 = (n0 + 1) p/3; dalla [1] si ottiene:
pn0 +1 = p + pn0
n −1
n0 + 2
n0 − 1
=p+ 0
p=
p,
n0 + 1
3
3
dunque la [2] è soddisfatta per n0 + 1, e quindi per tutti gli interi n ≥ 2.
Il percorso medio di ogni vettura è dunque di (n + 1) p/3 chilometri.
50
I problemi della Scuola Normale
Anno 1982
1981.6*
Dato un tetraedro avente 5 dei 6 spigoli di lunghezza minore o uguale a 2, provare che il suo
volume è minore o uguale a 1. In quale caso il volume è uguale a 1?
51
Dalla discussione precedente è poi chiaro che il volume è uguale
√ a 1 se e solo se ABC e ABD sono triangoli
equilateri di lato 2 situati in piani ortogonali (quindi CD = 6) (fig. 2).
RISOLUZIONE
Siano A, B , C , D i vertici del tetraedro e supponiamo che gli spigoli diversi da CD abbiano
lunghezze non superiori a 2.
Detta S la superficie del triangolo ABC e h l’altezza del tetraedro relativa al vertice D, si ha:
volume(ABCD) =
1
S h.
3
[1]
Sia H la proiezione di D sulla retta per A e B e sia h0 = DH (fig. 1); ovviamente h0 è maggiore o uguale
alla distanza di D dal piano di ABC , dunque si ha:
h ≤ h0 ,
[2]
1982.5*
Dato un piano α e due punti P , Q nello stesso semispazio si considerino le sfere passanti per
i punti P , Q e tangenti al piano α.
Si richiede di determinare il luogo dei punti di tangenza.
Sia S una sfera passante per i punti P , Q e tangente al piano α; indichiamo con T il punto
di tangenza. Conduciamo la retta r per P , Q; si possono presentare due casi.
RISOLUZIONE
Caso (a)
La retta r non è parallela al piano α.
e il segno di uguaglianza vale solo quando le facce ABD e ABC sono ortogonali.
Poniamo ora AB = a; poichè nel triangolo ABD i lati AD e BD hanno lunghezza minore o uguale a 2, si
ha:
h0 ≤
r
S
4−
a2
4
.
[3]
r
Infatti, se ad esempio AH ≤ BH e dunque BH ≥ a/2,
2
h0 = BD 2 − BH 2 ≤ 4 −
a2
P
.
4
Nella [3] il segno di uguaglianza si ha solo quando BD = AD = 2.
p
Allo stesso modo si vede che l’altezza del triangolo ABC relativa al lato AB è minore o uguale a 4 − a2 /4;
dunque per la sua area si ha
S ≤
a
2
r
4−
a2
4
,
T
α
[4]
a
a2
Figura 1
a
4−
=
(4 − a) (4 + a) =
6
4
24
1
[4 − (2 − a)2 ] (4 + a) .
24
Ovviamente se 0 ≤ a ≤ 2, l’espressione [4 − (2 − a)2 ] (4 + a) raggiunge il suo massimo per a = 2; dunque
Indichiamo allora con L il punto di intersezione fra r e α; L è distinto da T , in quanto per tre punti in linea
retta non passa alcuna sfera, dunque T è esterno alla retta r.
Consideriamo il piano per r e T (fig. 1); detto piano taglia la sfera in un cerchio C e il piano α lungo una
retta s. C passa per i punti P , Q ed è tangente in T alla retta s; dunque, per il teorema della secante
e della tangente, risulta:
LT 2 = LP · LQ .
volume(ABCD) ≤ 1 .
D
D
h
H
Caso (b)
K
B
Figure 1, 2
C
[1]
Viceversa, ogni punto T del piano α che verifica la [1] è punto di tangenza con α di una sfera che passa per
T , P , Q e ha centro sulla perpendicolare ad α in T . (Per costruire il centro di tale sfera basta intersecare il
piano β perpendicolare al segmento P Q nel suo punto medio con la perpendicolare ad α in T .)
Si conclude quindi che il luogo cercato dei punti di tangenza è l’insieme dei punti di α che verificano la [1],
e quindi è un cerchio di centro L e raggio medio proporzionale fra LP e LQ.
h0
A
Q
L
valendo il segno di uguaglianza solo quando BC = AC = 2.
Dalle [1], [2], [3], [4] si ricava allora:
volume(ABCD) ≤
C
s
A
C
H
B
La retta r è parallela al piano α.
52
I problemi della Scuola Normale
Anno 1984
53
2
2
2
2 [m + n − (m + n)] = (m + n) − (m + n) ,
m2 + n 2 − 2 m n − ( m + n ) = 0 .
L’equazione [1] è dunque equivalente a
( m − n) 2 = m + n ,
S
[2]
con m e n fra 20 e 30. Dalla [2] si deduce che m + n è un quadrato perfetto; necessariamente, allora, si
ha che m + n = 49 che è il solo quadrato perfetto compreso fra 40 e 60. Supponendo per esempio m > n,
otteniamo
Q
P
T
(
β
m + n = 49
m−n=7
e risolvendo si ha
α
m = 28 ,
n = 21 .
In conclusione le due classi hanno 28 e 21 studenti.
Figura 2
In questo caso il piano β perpendicolare al segmento P Q nel suo punto medio è perpendicolare ad α e
quindi contiene, oltre al centro della sfera S, anche il punto di tangenza T (fig. 2).
Viceversa, ogni punto della retta comune a α e β è punto di tangenza con α di una sfera che passa per T ,
P , Q e ha centro sulla perpendicolare ad α in T . (Per costruire il centro di tale sfera questa volta occorre
intersecare la perpendicolare ad α in T con il piano perpendicolare al segmento P T nel suo punto medio.)
Nel caso (b), pertanto, si può concludere che il luogo cercato è la retta che si ottiene intersecando α con il
piano β perpendicolare al segmento P Q nel suo punto medio.
1983.5*
Due amici si sono iscritti alla prima classe di un liceo. Tale liceo ha due sezioni, le cui prime
classi hanno rispettivamente n e m studenti, con n e m compresi tra 20 e 30.
Sapendo che la probabilità che i due amici si trovino nella stessa classe è 1/2, dire quanti sono gli studenti
delle due classi.
1984.6*
Siano dati una circonferenza γ e un punto P distinto dal centro. Sia P AB un triangolo che,
tra tutti quelli che hanno un vertice in P e i rimanenti due su γ , abbia perimetro massimo.
Dimostrare che le due bisettrici uscenti dai vertici A e B passano per il centro di γ .
(Non si richiede la costruzione geometrica, né la determinazione degli elementi del triangolo.)
RISOLUZIONE
Sia P AB un triangolo che, fra tutti quelli che hanno un vertice in P e gli altri due su γ , ha
b è perpendicolare alla tangente a γ in A e dunque
perimetro massimo. Proviamo che la bisettrice di P AB
passa per il centro della circonferenza data.
Ragioniamo per assurdo e supponiamo che la perpendicolare p a detta bisettrice non sia tangente alla
circonferenza. La retta p ha quindi in comune con γ oltre ad A un altro punto C (fig. 1). Proveremo che il
triangolo P CB ha perimetro maggiore di quello di P AB . Sia infatti P 0 il simmetrico di P rispetto a p; i
b ) e risulta
punti P 0 , A, B sono allineati (in quanto p è la perpendicolare alla bisettrice di P AB
P A = P 0A ,
RISOLUZIONE
Indichiamo con A e B i due amici, e sia M la classe con m studenti e N la classe con n
studenti.
Vi sono m casi, su un totale di m + n, nei quali A viene iscritto in M ; sapendo poi che A sta in M , in m − 1
casi su m + n − 1, anche B viene iscritto in M . Perciò la probabilità che A e B siano entrambi in M è
P C = P 0C .
P
γ
p
m (m − 1)
.
(m + n) (m + n − 1)
O
Analogamente, la probabilità che i due amici risultino iscritti entrambi in N è espressa da
C
P0
n (n − 1)
.
(m + n) (m + n − 1)
La probabilità che A e B siano nella stessa classe è espressa dalla somma delle probabilità sopra calcolate.
Si ottiene cosı̀ l’equazione
m (m − 1) + n (n − 1)
1
= ,
(m + n) (m + n − 1)
2
A
B
[1]
Figura 1
ossia
Dalla disuguaglianza triangolare applicata al triangolo P 0 CB si ha
54
I problemi della Scuola Normale
0
0
P B < P C + CB
Anno 1985
1985.1
55
L’eguaglianza
p ! + q ! + r ! = s!
e dunque
P A + AB = P 0 A + AB = P 0 B < P 0 C + CB = P C + CB .
Abbiamo cosı̀ costruito un triangolo P CB che ha perimetro maggiore di P AB ; dunque P AB non ha
perimetro massimo, contro l’ipotesi. Ovviamente lo stesso ragionamento si può ripetere per il vertice B .
è soddisfatta per p = q = r = 2 e s = 3.
Dire se esistono altri numeri interi positivi per cui tale eguaglianza è vera.
(Si ricorda che n! indica il fattoriale di n, cioè il prodotto n · (n − 1) · . . . · 3 · 2 · 1 dei primi n numeri interi.)
Supponiamo di aver trovato quattro numeri p, q, r, s che verificano l’eguaglianza. Sia, ad
esempio, p ≤ q ≤ r; il numero r sarà certamente strettamente minore di s, e quindi r ≤ s − 1. Avremo
allora
RISOLUZIONE
b = 2 ·OAB
b , P BA
b = 2 ·OBA
b , ma poiché il triangolo
OSSERVAZIONE 1
Dalla proprietà dimostrata segue P AB
b = OBA
b ; pertanto il triangolo P AB di perimetro massimo è isoscele. Se
OAB è isoscele si ricava OAB
anche il punto P giace sulla circonferenza, anche la bisettrice di APbB passerà per il centro di γ e quindi
il triangolo AP B , avendo l’incentro coincidente con il circocentro, risulta equilatero. In questo modo si è
dimostrato che, fra tutti i triangoli inscritti in un cerchio, quello di perimetro massimo è equilatero, risultato
che non è facilmente dimostrabile in modo elementare per altra via.
La proprietà enunciata nel problema si può ottenere anche con il seguente ragionamento
di tipo meccanico. Consideriamo un elastico teso che passa per il punto P (fissato), per i punti A e B (liberi
di muoversi sulla circonferenza γ ) e che si chiude tornando in P . Supponiamo che la tensione dell’elastico,
in ogni configurazione, sia la stessa in tutti i suoi punti. La posizione dei punti A e B per i quali il perimetro
del triangolo P AB è massimo corrisponde al massimo allungamento possibile dell’elastico e rappresenta
quindi una posizione di equilibrio (instabile) dello stesso.
OSSERVAZIONE 2
P
e
s! = p ! + q ! + r ! ≤ 3 · r !
s! = s · (s − 1)! ≥ s · r!
per cui risulterà s ≤ 3.
Ma l’espressione p! + q ! + r! vale almeno 3, e quindi deve essere s = 3.
I numeri p, q, r e di conseguenza i rispettivi fattoriali sono tutti minori o uguali a 2; affinché possa valere
l’eguaglianza p! + q ! + r! = s! = 6 è necessario che sia p = q = r = 2.
1985.2*
Si considerino nel piano due circonferenze γ e γ 0 di eguale raggio. Determinare il luogo dei
punti medi P dei segmenti AA0 con A in γ e A0 in γ 0 .
Indichiamo rispettivamente con O, O0 i centri delle due circonferenze e con M il punto
medio di OO0 . Tracciamo la circonferenza γ0 di centro M e raggio uguale a quello di γ e γ 0 .
RISOLUZIONE
γ
γ
O
O
A
B
B0
γ0
M
A0
P
O0
F2
F
A
F1
B
γ0
Figura 1
Figura 2
−
→
−
→
−
→
È chiaro che in tale configurazione la risultante F delle due forze F1 e F2 che trasmettono al punto A la
tensione nelle direzioni P A e P B (fig. 2) deve essere perpendicolare alla tangente al cerchio nel punto A;
−
→
in caso contrario, infatti, vi sarebbe una componente non nulla di F nella direzione della tangente e dunque
il punto A non sarebbe di equilibrio.
−
→
−
→
−
→
La tesi segue facilmente osservando che |F1 | = |F2 | e di conseguenza il parallelogrammo individuato da F1 ,
−
→ −
→
−
→ −
→ −
→ −
→
F2 , F è un rombo; dunque gli angoli formati dalle forze F1 , F e F2 , F sono uguali.
Prendiamo A su γ e A0 su γ 0 . Costruiamo partendo da A il segmento AB parallelo, congruente e equiverso al
segmento OM : ne segue che ABM O è un parallelogramma, e dunque B è su γ0 . Costruiamo poi partendo
da A0 il segmento A0 B 0 parallelo, congruente e equiverso al segmento O0 M 0 : avremo che A0 B 0 M O0 è un
parallelogramma, e dunque B 0 è su γ0 . Inoltre, i segmenti AB e A0 B 0 sono uguali e paralleli, e dunque
ABA0 B 0 è un parallelogramma. Quindi il punto medio P di AA0 è anche punto medio di BB 0 , che è una
corda della circonferenza γ0 , per cui P è interno a γ0 ; se i punti B e B 0 coincidono (il che accade se il
segmento AA0 è parallelo a OO0 ) il punto medio P giace su γ0 .
D’altra parte se P è un punto interno alla circonferenza γ0 consideriamo la corda BB 0 staccata da γ0 sulla
retta perpendicolare in P alla retta M P . Il punto P è punto medio di tale corda e, con la stessa costruzione
vista sopra, possiamo determinare un punto A su γ e un punto A0 su γ 0 in modo che P sia punto medio
di AA0 ; se P è sulla circonferenza γ0 , possiamo usare la stessa costruzione prendendo B e B 0 coincidenti
con P .
Questo dimostra che il luogo cercato coincide col cerchio racchiuso da γ0 .
56
1985.3
I problemi della Scuola Normale
Fra i triangoli equilateri contenuti in un quadrato assegnato, determinare quelli di area massima.
Osserviamo (fig. 1) che se un triangolo equilatero ABC è contenuto in un quadrato, è
possibile traslarlo in modo che un suo vertice coincida con un vertice del quadrato. Da tale posizione è poi
possibile ruotarlo intorno al vertice in modo che l’altezza relativa giaccia sulla diagonale del quadrato.
RISOLUZIONE
C
Anno 1985
57
1985.4
Fissati due punti P e Q su due lati consecutivi di un dato rettangolo, si determinino sugli altri
due lati due punti R e S tali che il quadrilatero P QRS abbia area massima.
RISOLUZIONE
Siano P , Q, R e S quattro punti rispettivamente sui lati AD, AB , BC e CD del rettangolo. I
punti P e Q sono fissi, mentre R e S vanno scelti in modo da rendere massima l’area del quadrilatero P QRS ,
vale a dire in modo da rendere minima la somma delle aree dei triangoli P DS , SCR e RBQ. Poniamo
AD = a ,
C0
AB = b ,
DS = x ,
PD = c,
CR = y ;
QB = d
la quantità da rendere minima è dunque (a parte un fattore 2)
B
A
S = x c + (b − x) y + (a − y ) d = x c + (b − d) y − x y + a d
B0
A0
S
D
C
P
C 000
R
C 00
A
Q
B
Figura 1
B 000
B 00
A0
A0
Figura 1
0
0 00
0 00
Si vede cosı̀ che l’area massima è realizzata dal triangolo A B C , che si ottiene quando gli altri due
vertici cadono sui lati del quadrato; detta a la lunghezza del lato del quadrato, il lato di tale triangolo
equilatero è dato da
0
AB
00 0
=
a
cos 15◦
a
=
=p
(1 + cos 30◦ )/2
q
a
√
(2 + 3)/4
2
√
e l’area massima ottenibile è a (2 3 − 3).
= 2a
q
√
2− 3
Supponiamo x fissato; essendo i termini x c, a d costanti occorre minimizzare (b − d − x) y , con 0 ≤ y ≤ a.
È ovvio che si hanno tre casi, a seconda del segno di b − d − x:
(1) se x < b − d il minimo si ha per y = 0;
(2) se x > b − d il minimo si ha per y = a;
(3) se x = b − d la grandezza è costantemente nulla.
Facciamo ora variare x, tenendo conto che deve essere 0 ≤ x ≤ b: nel caso (1) si ha S = x c + a d e quindi il
minimo si ha per x = 0, nel qual caso S = a d. Nel caso (2) si ha S = x c + (b − d) a − x a + a d = a b − x (a − c) e
il minimo si avrà in corrispondenza del massimo valore possibile per x, cioè per x = b, nel qual caso S = b c.
Osserviamo infine che il terzo caso si può escludere, in quanto si avrebbe
S = a d + (b − d) c = b c + (a − c) d
che è certamente maggiore tanto di a d quanto di b c.
Concludiamo che se b c > a d l’area massima di P QRS si ha quando S coincide con D e R coincide con C ;
se b c < a d l’area massima si ha quando S coincide con C e R con B .
SECONDA SOLUZIONE Se il quadrilatero P QRS ha area massima, modificando comunque R o S si otterrà un
quadrilatero di area non superiore a quella di partenza. Mostriamo che certe scelte di R e di S non possono
massimizzare l’area. Distinguiamo tre casi, a seconda della lunghezza di DS .
Se DS è più corto di AQ (fig. 2), tenendo fisso S , l’area di P QRS aumenta se aumenta quella del triangolo
SQR e questo ha area massima quando R coincide con C , perché a parità di base l’altezza è maggiore.
Quindi, se DS < AQ, il punto R deve coincidere con C ; il quadrilatero P QCS non ha certamente area
massima se S =
6 D. Dunque, fra i quadrilateri per cui DS < AQ, l’unico possibile candidato è quello con
R = C e S = D.
D
S
C
D
S
C=R
D=S
C=R
R
P
A
Figura 2
P
Q
B
A
P
Q
B
A
Q
B
58
I problemi della Scuola Normale
Anno 1985
Se DS = AQ, qualunque scelta di R porta ad un’area per P QRS pari alla metà di quella di ABCD, ma se
BR 6= AP certamente tale area può essere aumentata.
Se DS > AQ (fig. 3), allora tenendo fisso S conviene prendere R coincidente con B , e per R = B l’area
del quadrilatero non ha un massimo se S 6= C ; l’altro candidato per l’area massima è dunque P QRS con
R = B, S = C .
D
S
C
D
S
C
1
3
(n − 1) + 18 n
2
n
1
(18 − 1) + 18 · 18
2
R
P
A
P
Q
B
A
P
Q
B=R
A
Q
1
182
1
=
n+
−
2
n
2
Tenuto conto che i numeri n e 18/n2 hanno prodotto costante, si avrà che la loro somma è minima quando
sono uguali, cioè quando n = 18.
Il costo minimo dell’opera sarà allora, in miliardi di lire:
C=S
D
59
2
3
18
2
= 8.5 + 9 = 17.5 .
B=R
Figura 3
1985.6*
Fra P QCD e P QBC il primo quadrilatero ha area maggiore se CB · QB < DC · DP , equivalente a
AD · (AB − AQ) < AB · (AD − AP ), cioè AB · AP < AD · AQ, e finalmente AP/AD < AQ/AB .
Con una bilancia a piatti e disponendo di infiniti pesi campione
p, p1 , q1 , p2 , q2 , . . . , pn , qn , . . .
da
1, λ, 1/λ, λ2 , 1/λ2 , . . . , λn , 1/λn , . . .
1985.5
Per la costruzione di un certo ponte si prevede che il costo di ogni arcata sarà di 18 s2 miliardi
di lire, ove s è la distanza in chilometri tra i due piloni di sostegno di quell’arcata, mentre il costo di ogni
pilone sarà di mezzo miliardo.
Se il ponte deve essere lungo 3 chilometri quale sarà il costo minimo dell’opera?
grammi rispettivamente, dove λ è un numero reale maggiore di 1, si vogliono pesare tutti gli oggetti con
una precisione arbitrariamente grande. Per quali valori di λ ciò è possibile?
( N.B. Si dispone di un solo esemplare di ogni peso campione e non si possono mettere pesi campione sul
piatto che contiene l’oggetto da pesare ).
RISOLUZIONE
Ricordiamo anzitutto che la somma dei primi n termini di una progressione geome-
trica a, a2 , a3 , . . . , an è
a (1 − an )/(1 − a) .
3 km
s1
s2
sn
Cosı̀, disponendo di un numero arbitrariamente grande di pesi da 1/λ, 1/λ2 , . . . , 1/λn grammi il peso
massimo che si può bilanciare è
1
1 1 − 1/λn
<
.
λ 1 − 1/λ
λ−1
RISOLUZIONE
Sia n il numero delle arcate, di lunghezza s1 , s2 , . . . , sn ; il ponte avrà quindi n − 1 piloni.
Si richiede di determinare numero e lunghezza delle arcate, in modo da rendere minimo il costo di costruzione
1
(n − 1) + 18 s21 + s22 + · · · + s2n
2
con la condizione
Se λ è maggiore di 2 si ha che 1/(λ − 1) è strettamente minore di 1, e quindi lo scopo non potrà essere
raggiunto. Si consideri infatti un peso p maggiore di 1/(λ − 1) grammi e minore di 1 grammo, ad esempio
1 + 1/(λ − 1)
pari a
; usando il peso da un grammo otteniamo una approssimazione di p per eccesso, mentre
2
usando solo i pesi da 1/λk grammi otteniamo stime per difetto inferiori a 1/(λ − 1). Si vede quindi che tanto
1 − 1/(λ − 1)
la stima per eccesso che quelle per difetto differiscono dal peso vero per almeno
grammi.
2
Mostriamo ora che l’obiettivo può essere conseguito se 1 < λ ≤ 2.
s1 + s2 + · · · + sn = 3 .
Cominciamo col provare che in una configurazione che realizza il costo minimo fra quelle con un numero n
di arcate fissato, queste devono avere uguale lunghezza; infatti, se due arcate consecutive hanno lunghezza
a1 = ā − h e a2 = ā + h, il loro contributo al costo è proporzionale a a21 + a22 = 2 ā2 + 2 h2 e può essere
ridotto prendendo h = 0, ossia sostituendo le due arcate di lunghezza a1 e a2 con due arcate uguali lunghe
ā = (a1 + a2 )/2.
Dunque in una configurazione di costo minimo con n arcate, queste hanno tutte lunghezza 3/n, e il numero
n che minimizza il costo è quello per cui è minima l’espressione
0 λ−11 λ
λ2
λn
w
λ n+1
Figura 1
Fissato un peso w, da λ > 1 segue che vi è un unico intero relativo, che indichiamo con n0 , tale che
λn0 ≤ w < λn0 +1 ;
[1]
60
I problemi della Scuola Normale
Anno 1986
1986.1
poniamo quindi sulla bilancia il peso da λn0 grammi (che sarà il peso p se n0 = 0, il peso pn0 se n0 > 0
oppure il peso qm0 se n0 = −m0 < 0).
Se w = λn0 abbiamo determinato w con precisione infinita; altrimenti troviamo l’intero n1 per cui
n1
λ
n0
≤w−λ
n1 +1
<λ
.
Dalla [1] e dalla diseguaglianza λ ≤ 2 otteniamo
0 ≤ w − λn0 < λn0 +1 − λn0 = λn0 (λ − 1) ≤ λn0
per cui deve essere λn1 +1 ≤ λn0 , e quindi n1 + 1 ≤ n0 . In particolare n1 è distinto da n0 e possiamo dunque
aggiungere sulla bilancia il peso da λn1 grammi.
Proseguendo in questo modo si troverà una sequenza strettamente decrescente di indici, n0 , n1 , n2 , . . ., e sul
piatto della bilancia saranno messi i corrispondenti pesi da λn0 , λn1 , λn2 , . . ..
Questo procedimento può terminare, nel qual caso il peso w è esattamente determinato, o proseguire fintanto
che per il peso residuo d da bilanciare non si ha
nh +1
eλ
λnh ≤ d < λnh +1
61
Si determinino gli interi positivi k tali che il polinomio
x5 + x4 + x3 + k x2 + x + 1
sia prodotto di polinomi a coefficienti interi di grado minore di 5.
RISOLUZIONE
Se il polinomio P è prodotto di due polinomi a coefficienti interi, si devono prendere in
considerazione due possibilità a priori distinte: P (x) = P1 (x) P2 (x) con P1 (x) di grado 1 e P2 (x) di grado
4 oppure P1 (x) di grado 2 e P2 (x) di grado 3. Possiamo sempre supporre che il coefficiente del termine di
grado massimo di P1 e di P2 sia 1.
Nel primo caso si ha che P (x) = (x − a) P2 (x). Questo significa che a è una radice intera di P (x) e, dovendo
essere un divisore del termine noto, può essere solo 1 o −1.
Se a = 1 deve essere P (a) = k + 5 = 0, e quindi k = −5; se a = −1 deve essere P (a) = −1 + 1 − 1 + k − 1 + 1 =
k − 1 = 0, e quindi k = 1.
Per il secondo caso osserviamo che i termini noti di P1 e di P2 hanno come prodotto 1; dunque sono entrambi
1 o entrambi −1.
Se sono entrambi 1 si ha:
P (x) = (x2 + a x + 1) (x3 + b2 x2 + b1 x + 1) =
è inferiore alla precisione richiesta.
x5 + (a + b2 ) x4 + (1 + a b2 + b1 ) x3 +
(b2 + a b1 + 1)x2 + (a + b1 ) x + 1 ;
uguagliando i coefficienti dei termini di grado 4 e di grado 1 si ottiene a + b2 = a + b1 = 1, e quindi
b1 = b2 = 1 − a; uguagliando i coefficienti degli altri due termini, e sostituendo b1 a b2 si ha
1 + a b1 + b1 = 1
b1 + a b1 + 1 = k
e le due relazioni sono compatibili solo se k = 1, nel qual caso si ottiene che (a + 1) b1 = (a + 1) (1 − a) = 0
e finalmente a = 1, b1 = b2 = 0 oppure a = −1, b1 = b2 = 2.
Se invece i termini noti di P1 e di P2 sono entrambi −1 si ha:
P (x) = (x2 + a x − 1) (x3 + b2 x2 + b1 x − 1) =
x5 + (a + b2 ) x4 + (−1 + a b2 + b1 ) x3 +
(−b2 + a b1 − 1) x2 + (−a − b1 ) x + 1 ;
uguagliando i coefficienti dei termini di grado 4 e di grado 1 si ha a + b2 = 1, −a − b1 = 1, da cui b1 = −1 − a,
b2 = 1 −a; uguagliando i coefficienti degli altri termini e sostituendo le relazioni trovate per b1 e b2 otteniamo:
(
da cui
(
− 1 + a (1 − a) − 1 − a = 1
− 1 + a + a ( − 1 − a) − 1 = k
− 2 − a2 = 1
− 2 − a2 = k
ma la prima delle due relazioni non è soddisfatta per nessun valore di a.
Dunque il polinomio si fattorizza in prodotto di polinomi a coefficienti interi solo per k = 1 e per k = −5.
Per k = −5 si ha
P (x) = (x − 1) (x4 + 2 x3 + 3 x2 − 2 x − 1) .
Per k = 1 si ha
P (x) = (x + 1) (x4 + x2 + 1) .
62
1986.2
I problemi della Scuola Normale
Anno 1986
Il piano per P , Q, A contiene ovviamente sia A sia A ; dato che α e β sono minori di 1, il punto A0 è
interno al segmento P A e il punto A00 è interno al triangolo di vertici P , Q, A. Pertanto la retta per A e A00
intersecherà il segmento P Q in un suo punto interno, che indicheremo con R.
Proveremo che il punto R non dipende dalla scelta di A e che il rapporto RA00 : RA è costante. Ne seguirà
che la trasformazione che manda A in A00 è una omotetia dello spazio di centro R.
Sia M l’intersezione della parallela per A0 alla retta P Q con la retta AR; dalla similitudine dei triangoli
AP R e AA0 M e QRA00 e A0 M A00 (fig. 2) si ha:
Si dimostri che il sistema di equazioni
(
y ex − e−y = x y ex
x ey − e−x = ey
ha una sola soluzione.
RISOLUZIONE
(
63
0
Possiamo riscrivere il sistema nella forma
y (1 − x) ex = e−y
00
[1]
(1 − x) ey = −e−x
R
da cui otteniamo che x 6= 1 e anche y 6= 0. Dividendo membro a membro le due equazioni otteniamo
Q
A 00
y ex−y = −ex−y
P
e quindi, dato che ex−y 6= 0,
M
y = −1 .
A0
Sostituendo nella prima equazione di [1] otteniamo
A
(x − 1) ex = e ,
ovvero
Figura 2
(x − 1) ex−1 = 1 .
[2]
Ne segue che x > 1; la funzione (x − 1) ex−1 è, per x ≥ 1, una funzione crescente in quanto prodotto di due
funzioni crescenti non negative; per x = 1 vale 0, e per x = 2 vale e = 2.718... > 1, e quindi l’equazione [2]
ammette una sola soluzione x0 , compresa fra 1 e 2.
L’unica soluzione del sistema proposto è dunque la coppia x = x0 , y = −1.
1986.3
Si dimostri che la composizione di due omotetie nello spazio, con poli P e Q distinti, è ancora
una omotetia di polo R, allineato con P e Q, oppure una traslazione parallela a P Q.
RISOLUZIONE
Si considerino due omotetie di centri rispettivamente P e Q e costanti α e β che, per fissare
le idee, supporremo per ora positive e minori di 1.
Un qualunque punto X dello spazio (diverso da P e Q) viene mandato dalla prima omotetia in un punto X 0
allineato con P e con X in modo che P X 0 : P X = α e dalla seconda in un punto X 00 allineato con Q e con
X in modo che QX 00 : QX = β .
Sia A un qualunque punto dello spazio e applichiamo le due omotetie: la prima manderà A in un punto A0
allineato con P e con A tale che P A0 : P A = α, la seconda manderà A0 in un punto A00 allineato con Q e
con A0 tale che QA00 : QA0 = β (fig. 1).

P A0
M A0 AA0 P A − P A0


=
=
=1−

PR
PA
PA
PA QA00
QA00
QA00
QA00
 RQ


=
=
=
/
1
−
.
M A0 A0 A00 QA0 − QA00
QA0
QA0
[1]
Tali rapporti sono dunque delle costanti (rispettivamente uguali a 1 − α e a β/(1 − β )) indipendentemente
dalla scelta del punto A.
Pertanto il rapporto RQ : P R è indipendente da A e quindi il punto R è univocamente individuato sul
segmento P Q, qualunque sia il punto A iniziale. Inoltre, sempre per similitudine, abbiamo
P A0
RM
=
= α,
RA
PA
RA00 QA00
=
=β,
RM
QA0
e dunque RA00 : RA è una costante (pari a α β ).
Risulta cosı̀ dimostrato, nel caso 0 < α, β < 1, che la composizione di due omotetie è una omotetia.
Il caso in cui sia α sia β sono maggiori di 1 è del tutto simile, con il ruolo dei punti P e Q scambiato (fig. 3).
Q
R
P
R
Q
X 00
P
Figura 1
Q
A
M
A 00
A0
P
X0
X
A 00
A0
A
Figura 3
Nel caso in cui, invece, si ha 0 < α < 1 e β > 1 (o viceversa), occorrerà distinguere, in quanto non è più
garantito che la retta per A e per A00 incontri la retta per P e Q. Se la incontra, certamente il punto di
64
I problemi della Scuola Normale
intersezione è esterno al segmento P Q e l’argomentazione precedente si può ripetere inalterata (anche se
cambierà il segno della costante nella seconda delle [1] in quanto A0 A00 = QA00 − QA0 ).
R
Anno 1986
Per un generico punto A la distanza di t(A) dalla retta l per P e Q è pari alla distanza di A dalla retta l,
moltiplicata per |αβ| =
6 1; dunque se R è un punto unito di t, deve giacere sulla retta l, e quindi ha la forma
R = P + r(Q − P ). Dalla relazione t(R) = R segue allora che
R − P = r(Q − P ) = (Q + β (P − Q) + rαβ (Q − P )) − P
Q
Q
65
e quindi
P
P
A0
M
r(1 − αβ )(Q − P ) = (1 − β )(Q − P ) , r =
A0
A
A
A 00
A 00
Figura 4
Se invece la retta per A, A00 è parallela alla retta per P e Q, i triangoli AA0 A00 e P A0 Q sono simili con
rapporto di similitudine A0 A : P A0 = A0 A00 : QA0 , da cui segue
P A − P A0 QA00 − QA0
=
,
P A0
QA0
cioè
PA
QA00
=
,
P A0
QA0
da cui finalmente α β = 1; in questo caso la composizione delle due omotetie è una traslazione.
I casi in cui uno o entrambi i parametri α e β sono negativi si trattano in maniera analoga.
SECONDA SOLUZIONE
Diamo anche una soluzione basata sulla geometria analitica.
Indichiamo con hP l’omotetia di polo P e con α la sua costante, con hQ l’omotetia di polo Q e con β la
sua costante; se fissiamo un sistema di coordinate e indichiamo con (xP , yP , zP ) le coordinate del punto P
e con (xQ , yQ , zQ ) le coordinate del punto Q, le due omotetie si scrivono come
 0

 x = xP + α (x − xP )
y 0 = yP + α ( y − yP )

 0
z = zP + α (z − zP )
 00

 x = xQ + β (x − xQ )
y 00 = yQ + β (y − yQ )

 00
z = zQ + β (z − zQ )
che possiamo indicare compattamente con la notazione
A0 = P + α (A − P )
A00 = Q + β (A − Q)
È facile vedere che tale notazione è compatibile con le usuali proprietà algebriche (associatività, distributività, . . . ).
Indichiamo con t la trasformazione composta, definita da
t(A) = hQ (hP (A)) = Q + β (P + α(A − P ) − Q) =
Q + β (P − Q) + βα(A − P ) .
Se αβ = 1 si ha
t(A) = Q + β (P − Q) + (A − P ) =
Q + β (P − Q) + (A − Q) + (Q − P ) =
A + (1 − β )(Q − P )
e dunque t è una traslazione con vettore (1 − β )(Q − P ), parallelo alla retta per P e Q.
Dimostriamo ora che se αβ 6= 1 la composizione è una omotetia, mostrando che esiste un solo punto unito
R e che t(A) = R + αβ (A − R).
1−β
1 − αβ
Verifichiamo che la trasformazione t è una omotetia di polo R e di costante αβ : per ogni punto A si ha che
il punto immagine t(A) = Q + β (P − Q) + αβ (A − P ) si può esprimere come R + αβ (A − R). Infatti
Q + β (P − Q) + αβ (A − P ) =
P + (Q − P ) + β (P − Q) + αβ (R − P ) + αβ (A − R) =
αβ (1 − β )
(Q − P ) + αβ (A − R) =
P + 1−β+
1 − αβ
αβ
P + (1 − β ) 1 +
(Q − P ) + αβ (A − R) =
1 − αβ
1−β
P+
(Q − P ) + αβ (A − R) =
1 − αβ
R + αβ (A − R) .
66
I problemi della Scuola Normale
Anno 1986
0
TERZA SOLUZIONE
Diamo una soluzione sintetica più rapida, anche se lievemente più sofisticata.
Il problema è di geometria tridimensionale, ma presenta una simmetria cilindrica intorno alla retta per i poli
delle due omotetie, il che consente di provare la tesi studiandolo su un piano (e dunque su tutti i piani per
P e Q).
Una omotetia manda, per il teorema di Talete, ogni retta del piano in una retta ad essa parallela; ma
allora anche la composizione di due omotetie gode della stessa proprietà. Prendiamo due punti A e B non
giacenti sulla retta P Q e indichiamo con A0 e B 0 le loro immagini attraverso la trasformazione composta.
Per quanto osservato le rette AB e A0 B 0 sono parallele. Vi sono due possibilità:
(a) le rette AA0 e BB 0 sono fra loro parallele; in tal caso AA0 B 0 A0 è un parallelogramma, e in particolare
si ha AA0 = BB 0 e AB = A0 B 0 .
67
0
CR : C R = AR : A R. La trasformazione è quindi una omotetia; che il punto R giaccia sulla retta per
P Q segue dal fatto che esso appartiente alla retta per P e P 0 , e che P 0 giace sulla retta per i poli.
b minore di 60◦ . Si
1986.4
Sia ABC un triangolo isoscele di base BC con l’angolo al vertice B AC
costruisca un altro triangolo P QR, di base QR, circoscritto e simile ad ABC , tale che il punto A appartenga
al segmento QR e si abbia QA = 2 · AR.
Si traccino la circonferenza per A e B , tangente in B al segmento BC , e la circonferenza
per A e C , tangente in C al segmento BC , e siano O e O0 i centri delle due circonferenze.
RISOLUZIONE
B0
A
C0
A0
O0
O
B
T
A
A0
C
B
Figura 4
C
Figura 1
0
0
0
0
Se C è un qualunque altro punto del piano e C è il suo trasformato si ha che i triangoli ABC e A B C
sono simili; essendo AB = A0 B 0 , essi sono uguali e quindi AC = A0 C 0 . Ne segue che ACC 0 A0 è un
parallelogramma e dunque i segmenti CC 0 e AA0 sono paralleli ed hanno ugual lunghezza. La trasformazione
quindi è una traslazione; per provare che il segmento AA0 è parallelo a P Q basta osservare che, per quanto
appena visto, AA0 è parallelo a P P 0 ; d’altra parte il punto P viene lasciato invariato dalla prima omotetia
di polo P e quindi il punto P 0 giace sulla retta per P e Q.
I lati obliqui del triangolo sono visti, dai punti degli archi di tali circonferenze esterni ad ABC , sotto un
angolo uguale a quelli alla base del triangolo stesso. Si individui il punto T del segmento OO0 tale che
b < 60◦ , l’ulteriore punto A0 di contatto delle circonferenze è interno
OT = 2 T O0 . Si noti che, essendo B AC
al triangolo, cosicché il punto T giace sotto la parallela per A alla base BC .
Si mandi da T la congiungente con A e da A la perpendicolare a questa; essa taglierà le circonferenze in
due ulteriori punti Q e R che sono quelli cercati.
(b) le rette AA0 e BB 0 si intersecano in un punto R, che può cadere esternamente o internamente ai
segmenti AA0 e BB 0 .
Q
A
H
B
B
H0
O
C
C
T
R
O0
A
A
B0
R
K
A0
R
B
A0
K
C
B0
Figura 5
Dal fatto che AB e A0 B 0 sono paralleli segue che i triangoli ABR e A0 B 0 R sono simili e quindi che
AB : A0 B 0 = AR : A0 R = BR : B 0 R.
Sia C un qualunque punto non allineato con A e B . Tracciamo la retta per C e R e sia K la sua intersezione
con la parallela condotta per A0 alla retta AC . Per il teorema di Talete si ha CR : KR = AR : A0 R
e quindi, tenendo conto delle proporzionalità precedentemente ricordate, CR : KR = BR : B 0 R;
quindi K è intersezione della parallela ad AC per A0 con la parallela a BC per B 0 , e dunque coincide
con il trasformato di C ; in particolare, per ogni punto C , la retta CC 0 passa per R e vale la proporzione
P
Figura 2
Infatti se H e H 0 sono i piedi delle perpendicolari condotte da O e da O0 a QR, per il teorema di
Talete si avrà HA = 2 AH 0 , ma QH = HA e AH 0 = H 0 R e dunque QA = 2 AR. D’altra parte le rette
68
I problemi della Scuola Normale
Anno 1986
per Q, B e per R, C individuano un triangolo QRP che ha gli angoli alla base QR uguali a quelli di ABC ,
e dunque è isoscele e ad esso simile.
69
1986.6*
Si consideri un biliardo di forma triangolare, come in figura, con α = 30◦ e si supponga di
lanciare una bilia dal punto A.
α
1986.5*
(a) Siano α, β , γ , δ quattro angoli minori di 180◦ . Si dimostri che
sin α + sin β + sin γ + sin δ ≤ 4 sin
α+β+γ+δ
.
4
(b) Utilizzando la relazione precedente,
si dimostri che la somma dei seni degli angoli interni di un triangolo
√
è sempre minore o eguale a 3 3/2.
RISOLUZIONE
Per il punto (a) basta provare che se α e β sono due angoli minori di 180◦ risulta
sin α + sin β ≤ 2 sin
α+β
2
;
A
[1]
la stessa relazione varrà per γ e δ , e anche per gli angoli (α + β )/2 e (γ + δ )/2 che sono anch’essi minori di
180◦ . Avremo quindi
γ+δ
+ 2 sin
≤
2
2
α+β+γ+δ
4 sin
.
4
La diseguaglianza [1] segue immediatamente dalla concavità della funzione seno tra 0 e 180◦ ; ne diamo
comunque una dimostrazione elementare. La [1] vale se e soltanto se
sin α + sin β + sin γ + sin δ ≤ ≤ 2 sin
α+β
B
Si provi che la bilia, qualunque sia la direzione iniziale, effettua solo un numero finito di rimbalzi prima di
battere sulla sponda AB .
Si determini anche il numero massimo di tali rimbalzi.
Si studi infine il caso in cui α è un angolo generico.
(Si supponga che il biliardo sia privo di attrito, che la palla sia puntiforme e che il rimbalzo sulle sponde
obbedisca alla legge di riflessione della luce). 1
RISOLUZIONE
b di 30◦ .
Supponiamo dapprima che il biliardo abbia la forma di un angolo (infinito) con O
sin 2α + sin 2β ≤ 2 sin (α + β )
per α e β fra 0 e 90◦ . L’ultima diseguaglianza equivale a
2 sin α cos α + 2 sin β cos β ≤ 2 sin α cos β + 2 cos α sin β
e ancora a
(sin α − sin β ) (cos α − cos β ) ≤ 0 ;
ma ciò è evidente perché, per angoli α e β fra 0 e 90◦ , sin α ≤ sin β se e solo se cos α ≥ cos β .
Punto (b): siano α, β , γ gli angoli interni di un triangolo. Scriviamo la disuguaglianza del punto (a) per gli
angoli α, β , γ e δ = (α + β + γ )/3 = 60◦ . Si ha
1
sin α + sin β + sin γ + sin δ ≤ 4 sin( · 4 δ )
4
da cui
sin α + sin β + sin γ ≤ 3 sin 60◦ =
√
3 3
.
2
Figura 1
Indichiamo con OS0 la sponda su cui giace il punto di partenza A e con OS1 quella su cui giace il primo
punto di rimbalzo A1 . Siano poi A2 , A3 , . . . , i successivi punti di rimbalzo. La traiettoria della bilia è
costituita dalla spezzata AA1 A2 A3 . . .. Consideriamo un angolo S1 OS2 di 30◦ e sia A02 il punto di OS2 per
cui OA02 = OA2 ; proviamo che i punti A, A1 e A02 sono allineati.
c1 O = A2 A
c1 O; per ipotesi l’angolo di incidenza è
I triangoli OA2 A1 e OA02 A1 sono uguali, pertanto A02 A
c1 O = A2 A
c1 O. Ne segue
uguale a quello di riflessione, quindi A02 A
c1 O + A2 A
c1 O + A2 A
c1 A = AA
c1 S1 + A2 A
c1 O + A2 A
c1 A = 180◦ ,
A02 A
dunque i punti A, A1 e A02 giacciono su una retta.
b 3 , S3 OS
b 4 , . . . , tutti di 30◦ , e sulle semirette OSm si
Allo stesso modo si prendano degli angoli S2 OS
prendano dei punti A0m tali che OA0m = OAm per m = 2, 3, . . . (i punti A01 e A1 coincidono). Con lo stesso
1
Il testo originale del problema è leggermente diverso (N. d. C.).
70
I problemi della Scuola Normale
ragionamento si vede che i punti
A1 , A02
e
A03
sono allineati, e dunque A,
A1 , A02 ,
A03 ,
. . . stanno su una retta
r che è lo sviluppo della traiettoria della bilia.
Anno 1987
71
1987.1
Siano assegnati due numeri reali positivi non nulli r e p, con r < p. Tra tutti i quadrilateri
convessi di perimetro p, aventi la somma delle lunghezze di una coppia di lati consecutivi uguale ad r, si
determini quello di area massima.
Sia ABCD un quadrilatero convesso di area massima fra quelli che verificano le condizioni
assegnate: AB + AD + CB + CD = p e AB + AD = r. In particolare tale quadrilatero ha area maggiore di
tutti quelli che verificano le condizioni precedenti e hanno fissata la diagonale BD.
RISOLUZIONE
S3
S4
S5
S6
S2
A40
A30
A0 A
S1
A20
A
A3 1
O
A2 A4
B
A
D
S0
C0
Figura 2
È chiaro quindi che non vi possono essere più di 5 rimbalzi.
Se ora reintroduciamo la sponda AB (fig. 2) è evidente che la bilia non può toccare più di 5 volte le sponde
laterali prima di battere su AB .
Nel caso di un angolo α generico il numero massimo di rimbalzi sarà dato dall’intero n tale che
Figura 1
Se a e b indicano le lunghezze dei lati AB e AD, l’area del triangolo ABD è data da
S=
π
n < ≤ n + 1.
α
B
C
p
p0 (p0 − a) (p0 − b) (p0 − d) ,
ove p0 è il semiperimetro di tale triangolo. Essendo d e a + b (e quindi anche p0 e p0 − d) quantità fissate, il
massimo di S si avrà quando è massimo (p0 − a) (p0 − b) = p0 2 − p0 (a + b) + ab, ovvero quando è massimo il
prodotto a b, vale a dire quando a = b.
Ne segue che il triangolo ABD, che massimizza l’area, è un triangolo isoscele e quindi AB = AD = r/2.
b
Allo stesso modo si vede che BCD è isoscele e che CB = CD = (p − r)/2; si ha inoltre che gli angoli ABC
b sono uguali perché somme di angoli uguali.
e ADC
Per risolvere il problema basta ora osservare che l’area di ABCD è uguale a quella del parallelogramma di
lati r/2 e (p − r)/2, e tale parallelogramma ha ovviamente area massima quando è un rettangolo.
A0
O
A
B
D
Figura 3
C0
Figura 2
In conclusione il quadrilatero di area massima ha le coppie di lati consecutivi AB , AD e CB , CD di egual
lunghezza e gli angoli in C e D retti.
Figura 4
72
1987.2
I problemi della Scuola Normale
Siano p, q , r tre numeri reali tali che il polinomio
A(x) = x3 + p x2 + q x + r
abbia tre radici reali.
Determinare tre numeri reali a, b, c, espressi in funzione di p, q , r soltanto, in modo che il polinomio
B (x) = x3 + a x2 + b x + c
abbia per radici i quadrati delle radici di A.
RISOLUZIONE
Per il teorema di Ruffini se α, β , γ sono le radici di A(x) si ha
Anno 1987
73
1987.3
Sia dato un segmento AB nel piano. Si consideri il luogo L dei punti del piano che vedono il
segmento AB sotto un angolo di 60◦ . Si scelga P in L e si scelgano due punti C e D rispettivamente interni
ai lati BP e AP del triangolo ABP , in modo che AD = BC .
Si costruisca il triangolo equilatero CDQ di base CD, esterno al quadrilatero ABCD.
Si studi, al variare di P in L e di C , D secondo le condizioni indicate sopra, il luogo dei punti del piano
descritto dal punto Q.
RISOLUZIONE
Per prima cosa osserviamo che il luogo L è costituito dall’unione dei due archi dei cerchi
circoscritti ai triangoli equilateri ABK e ABK 0 costruiti da parti opposte del lato AB , archi contenenti
rispettivamente i punti K e K 0 .
A(x) = (x − α) (x − β ) (x − γ ) =
A
x3 − (α + β + γ ) x2 + (α β + α γ + β γ ) x − α β γ
D
e per il principio di identità dei polinomi si ha
P
p = −(α + β + γ )
q = αβ + αγ +β γ
K
K0
r = −α β γ .
C
L
Analogamente se B (x) ha per radici α2 , β 2 , γ 2 sarà
2
2
2
B (x) = (x − α ) (x − β ) (x − γ ) =
x3 − (α2 + β 2 + γ 2 ) x2 +
(α2 β 2 + α2 γ 2 + β 2 γ 2 ) x−
α2 β 2 γ 2 .
Dobbiamo quindi esprimere α2 + β 2 + γ 2 , α2 β 2 + α2 γ 2 + β 2 γ 2 e α2 β 2 γ 2 in termini di p, q e r.
Avremo
α2 + β 2 + γ 2 =
(α + β + γ )2 − 2 (α β + α γ + β γ ) =
p2 − 2 q ;
α2 β 2 + α2 γ 2 + β 2 γ 2 =
(α β + α γ + β γ )2 − 2α β γ (α + β + γ ) =
2
2
2
2
α β γ =r .
q2 − 2 p r ;
Il polinomio B (x) richiesto sarà quindi dato da
x3 − (p2 − 2 q ) x2 + (q 2 − 2 p r) x − r2 .
B
Figura 1
E’ immediato osservare che se P = K , per qualunque scelta di C e D il vertice Q del triangolo equilatero
costruito coincide con K , e simmetricamente se P = K 0 si ha che Q = K 0 .
Proviamo quindi a dimostrare che, se P appartiene all’arco contenente K , il vertice Q coincide con K ;
otterremo questo mostrando che, per qualunque scelta di P e per qualunque scelta di C e D coerente con le
condizioni, il triangolo CDK risulta equilatero e esterno ad ABCD.
b e K BC
b sono uguali perché angoli alla circonferenza che insistono sullo stesso arco P K ; i
Gli angoli K AD
lati AK e BK sono uguali perché lati di un triangolo equilatero; i lati AD e BC sono uguali per costruzione.
Quindi i triangoli KAD e KBC sono uguali, e in particolare sono uguali i lati KD e KC , e sono uguali
b e B KC
b ; si ha quindi che DKC
b = DKB
b + B KC
b = DKB
b + AKD
b = AKB
b e dunque
gli angoli AKD
b , opposto alla base del triangolo isoscele CKD è di 60◦ . Di conseguenza il triangolo CKD
l’angolo DKC
è equilatero, esterno al quadrilatero ABCD, e quindi Q coincide con K se P sta sull’arco contenente K ,
oppure con K 0 se P sta sull’arco contenente K 0 . Il luogo da determinare consiste dunque nei due punti K
e K 0.
74
I problemi della Scuola Normale
1987.4*
Un punto (x, y ) del piano si dirà razionale se x e y sono numeri razionali. Data una qualunque
circonferenza del piano cartesiano avente centro razionale, si provi che se essa contiene un punto razionale,
allora contiene infiniti punti razionali.
Anno 1987
1987.5*
75
Nella figura è rappresentato lo sviluppo delle facce di un tetraedro regolare nello spazio.
M
A
Mostriamo per prima cosa che la tesi vale per una circonferenza centrata nell’origine.
Supponiamo che una circonferenza C di centro O = (0, 0) e raggio r contenga un punto razionale P = (x1 , y1 ).
Se indichiamo con α l’angolo formato dalla semiretta OP con la semiretta positiva dell’asse x, avremo che
r cos α = x1 e r sin α = y1 sono numeri razionali.
Se β è un angolo per cui cos β e sin β sono entrambi razionali, allora possiamo esibire un altro punto a
coordinate razionali su C , prendendo quello di coordinate
C
RISOLUZIONE
R
P
Q
N
O
x = r cos(α + β ) = (r cos α) cos β − (r sin α) sin β ,
B
y = r sin(α + β ) = (r sin α) cos β + (r cos α) sin β .
I numeri cos β e sin β sono razionali se e solo se esiste una terna di numeri interi a, b, c tali che cos β = a/c
e sin β = b/c; i numeri a, b, c verificano quindi la relazione a2 + b2 = c2 e formano quindi una terna
pitagorica; viceversa, due terne pitagoriche non proporzionali individuano due punti razionali distinti
del cerchio unitario.
Data una successione di terne pitagoriche {(an , bn , cn )} a due a due non proporzionali, gli angoli βn
individuati da cos βn = an /cn e sin βn = bn /cn sono tutti diversi, e dunque i punti
Pn = (r cos(α + βn ), r sin(α + βn ))
al variare di n = 1, 2, . . . sono tutti distinti, e sono infiniti punti razionali che appartengono alla circonferenza
C.
La successione di terne pitagoriche data da
an = n 2 − 1 ,
bn = 2 n ,
Siano P , Q, R tre punti distinti del tetraedro corrispondenti, nello sviluppo, rispettivamente ad un punto
interno al segmento M N , un punto interno al segmento M O, un punto interno al triangolo M OC .
Sia α il piano contenente P , Q, R. Si determini, nello sviluppo piano, l’intersezione fra il piano α e le facce
del tetraedro.
Prolunghiamo il segmento QR fino ad intersecare il perimetro del triangolo M OC . Se
il punto di intersezione S appartiene al segmento M C (e certamente S non coincide con M ), è facile
determinare l’intersezione di α con le facce del tetraedro (fig. 1): dato che il segmento M C coincide, nel
tetraedro, con il segmento M A, il punto di intersezione S sarà rappresentato su M A dal punto S 0 , simmetrico
di S rispetto ad M . Il piano α interseca quindi solo tre facce del tetraedro, lasciando il vertice M da una
parte, e gli altri tre dall’altra, e l’intersezione è data dai segmenti P Q, QS , S 0 P .
RISOLUZIONE
cn = n 2 + 1
A
non contiene terne proporzionali. Da 2 n : 2 m = (n2 +1) : (m2 +1) segue infatti che 2 n (m2 +1) = 2 m (n2 +1)
e infine che 2 (n m − 1) (m − n) = 0, per cui m = n.
S0
M
S
C
R
Nel caso generale, se C è una circonferenza con centro nel punto a coordinate razionali O0 = (x0 , y0 ) e
raggio r, e contiene il punto razionale P = (x1 , y1 ), allora la circonferenza C 0 di centro O e raggio r contiene
il punto razionale (x1 − x0 , y1 − y0 ) = (r cos α, r sin α), e dunque contiene gli infiniti punti razionali di
coordinate (r cos(α + βn ), r sin(α + βn )); ma allora i punti
N
Q
P
O
Pn = (x0 + r cos(α + βn ), y0 + r sin(α + βn ))
al variare di n sono infiniti punti razionali distinti sulla circonferenza C .
B
Figura 1
Se il punto S appartiene invece al segmento OC (fig. 2), marchiamo su OB il punto S 0 con OS 0 = OS e
indichiamo con π1 il piano contenente i vertici M , N e O, e con π2 il piano contenente i vertici B , N e O.
L’intersezione del piano π1 con π2 è data dalla retta s su cui giace lo spigolo N O; l’intersezione del piano
α con π1 è data dalla retta r1 per P e Q.
Distinguiamo due casi, a seconda che le rette s e r1 siano incidenti in un punto Z , oppure siano parallele.
(a) Rette s e r1 incidenti in Z : il punto Z è intersezione dei tre piani α, π1 e π2 ; dunque la retta r2 per Z e il
punto S 0 rappresenta, nello sviluppo piano, l’intersezione di α e π2 .
76
I problemi della Scuola Normale
M
A
T0
P
N
R
S
r1
O
Z
s
r2
S0
T
77
1988.1*
Siano A, B , C , D quattro punti distinti assegnati nello spazio. Determinare una condizione
necessaria e sufficiente affinché ogni superficie sferica che passa per A e B intersechi ogni superficie sferica
che passa per C e D.
C
Q
Anno 1988
B
RISOLUZIONE
Articoliamo la soluzione in vari punti, in modo da mettere prima in evidenza le condizioni
necessarie, e arrivare poi a dimostrarne la sufficienza.
(1) Cominciamo con l’osservare che se esistono due semispazi Π1 e Π2 disgiunti, delimitati dai piani
paralleli σ1 e σ2 , tali che A e B appartengano a Π1 e C e D appartengano a Π2 , allora esiste una sfera S1
passante per A e B e tangente al piano σ1 ed una sfera S2 passante per C e D e tangente al piano σ2 ; S1
è contenuta in Π1 , S2 è contenuta in Π2 e sono quindi disgiunte. Per costruire la sfera S1 , si tracci nel
piano passante per A e B e perpendicolare a σ1 una circonferenza per A e B e tangente a σ1 . Il centro e
il raggio di tale circonferenza sono centro e raggio di S1 ; analoga la costruzione di S2 .
Figura 2
S1
(b) Rette s e r1 parallele: si vede facilmente che la retta lungo cui α taglia il piano π2 non può intersecare
la retta s o, per il ragionamento precedente, anche la retta r1 dovrebbe intersecare s; di conseguenza
l’intersezione di α con la faccia OBN è data dalla retta r2 per S 0 parallela ad s.
Indichiamo con T il punto in cui r2 incontra il segmento BN e con T 0 il suo simmetrico rispetto a N . Il
piano α incontra tutte e quattro le facce del tetraedro, e l’intersezione è data da P Q, QS , S 0 T , T 0 P .
B
A
σ1
σ2
C
1987.6
Tizio si trova nella sua abitazione e deve prendere un treno che parte dalla stazione esattamente
tra mezz’ora. Sotto la sua abitazione c’è la fermata degli autobus della linea A che lo portano alla stazione
in 20 minuti. A 5 minuti di cammino vi è una fermata delle linee B e C che lo possono portare alla stazione
in 18 minuti.
Tizio non conosce l’orario di passaggio degli autobus, ma sa che su ognuna delle linee gli autobus passano
ogni quarto d’ora.
Quale strategia conviene a Tizio per aver maggiore probabilità di prendere il treno?
RISOLUZIONE
Dato che non conosce l’orario di passaggio degli autobus, ma solo che passano ogni 15
minuti, Tizio assume che la probabilità di prendere un autobus aspettando x minuti sia x/15.
L’autobus A passa certamente entro 15 minuti, ma lo porta alla stazione in tempo solo se passa entro
30 − 20 = 10 minuti; scegliendo di aspettare A la probabilità di successo è di 10/15.
Se invece parte subito per la fermata degli autobus B e C , Tizio avrà 25 − 18 = 7 minuti utili per attendere
il passaggio di B o C . La probabilità che B passi in tempo utile è di 7/15, e cosı̀ per C ; Tizio quindi
riuscirà a prendere il treno se non capita che B e C passino entrambi negli 8 minuti successivi, e quindi
la probabilità di successo sarà data da 1 − 8/15 · 8/15 = 161/225, che è maggiore di 10/15 = 150/225,
probabilità di successo che avrebbe aspettando A. Fra le due alternative, la migliore è quella di partire
subito per la fermata degli autobus B e C .
Potrebbe essere più conveniente per Tizio una strategia intermedia, consistente nell’aspettare l’arrivo di A
per x minuti (x ≤ 7) e poi partire per la fermata di B e C ?
La probabilità che A passi nel primi x minuti è x/15; giunto alla fermata di B e C , Tizio ha 7 − x minuti
favorevoli su 15, e quindi la probabilità che non passi nessun autobus in tempo utile è data da ((8 + x)/15)2 .
La probabilità di successo con la strategia intermedia è data da
x
15
"
+ 1−
8+x
15
2 #
=
15x + 225 − 64 − 16x − x2
=
225
161 − x − x2
225
che per x = 0 fornisce la probabilità di successo con la strategia “subito B o C ”, e decresce all’aumentare
di x. Dunque Tizio decide di partire subito per la fermata di B e C .
D
S2
Figura 1
I semispazi Π1 e Π2 esistono certamente nei due seguenti casi.
(a) i segmenti AB e CD appartengono a due rette r1 e r2 sghembe. Siano H su r1 e K su r2 i due punti
tali che HK è il segmento di minima distanza fra r1 e r2 ; allora HK è perpendicolare ad r1 in H e
ad r2 in K . Prendiamo L e M su HK in modo che HL = LM = M K ; il piano perpendicolare ad
HK in L sarà σ1 , frontiera di Π1 ; il piano perpendicolare ad HK in M sarà σ2 , frontiera di Π2 .
(b) A, B , C , D appartengono ad uno stesso piano π e i segmenti AB e CD non si intersecano. In tal
caso esistono due rette parallele r1 e r2 che lasciano da parti opposte le coppie di punti A, B e C ,
D; i piani σ1 e σ2 saranno i piani perpendicolari a π e contenenti rispettivamente le rette r1 e r2 .
H
A
B
r1
A
C
L
M
K
C
D
r2
B
r1
r2
D
Figure 2,3
(2) Quindi, se A, B , C , D sono tali che ogni superficie sferica passante per A e B interseca ogni superficie
sferica passante per C e D, necessariamente i quattro punti sono complanari e i segmenti AB e CD
si intersecano. Consideriamo nel piano π contenente A, B , C , D gli assi dei segmenti AB e CD,
78
I problemi della Scuola Normale
che certamente hanno un punto O in comune dato che i due segmenti non sono paralleli. Le superfici
sferiche di centro O e raggio rispettivamente OA e OC non si intersecano se OA 6= OC e coincidono se
OA = OC ; in tal caso il quadrilatero ACBD è inscrivibile in una circonferenza.
B
Anno 1988
79
n
m
n
m
10 − 1 10 − 1 10 − 10
−
=
=
9
9
9
n−m
10
−
1
10m
= 10m αn−m = r s ;
9
ma r è primo con 10 e quindi deve dividere αn−m .
αn − αm =
(c) Il programma seguente (in qbasic Microsoft) termina, per quanto dimostrato nel punto (b), stampando
il minimo indice k , il corrispondente αk e il quoziente fra questo e r.
C
O
Γ2
Γ1
A
D
Figura 4
fine:
(3) Mostriamo infine che tale condizione è anche sufficiente. Se osserviamo che l’intersezione con π di
ogni superficie sferica S1 passante per A e B è una circonferenza Γ1 per A e B e che, analogamente, S2
ha per intersezione una circonferenza Γ2 per C e D, basta provare che se ACBD è inscrivibile in una
circonferenza, allora ogni altra circonferenza Γ1 per A e B lascia C e D da parti opposte. Infatti, se il
centro O1 di Γ1 non coincide con l’intersezione O degli assi dei segmenti AB e CD, le distanze da O1
dei punti C e D sono una minore e una minore del raggio di Γ1 ; un punto è quindi interno e uno esterno
e qualunque circonferenza Γ2 passante per C e D interseca necessariamente la circonferenza Γ1 .
1988.3
Si considerino i numeri naturali 1, 11, 111, . . . , e in generale si indichi con αn il numero che
si ottiene giustapponendo n cifre uguali a 1.
(a) Si provi che se αn è un numero primo allora n è primo.
(b) Si provi che, assegnato comunque un numero naturale r, non divisibile né per 2 né per 5, si può trovare
un αn multiplo di r.
(c) Si scriva un algoritmo o programma per calcolatore (in un qualunque linguaggio di programmazione)
che, a partire da r, calcoli il minimo n per cui vale la (b).
RISOLUZIONE
Scriviamo gli αn in una forma più maneggevole:
αn = 1 + 10 + 100 + . . . + 10n−1 =
(1 + 10 + 100 + . . . + 10n−1 ) (10 − 1)/9 = (10n − 1)/9 .
(a) Supponiamo per assurdo che sia n = p q , con p, q interi > 1; allora
pq
αn =
ciclo:
10
9
−1
=
10 − 1
10p(q−1) + . . . + 10p + 1
9
è un numero composto.
p
(b) Dobbiamo provare che se r è un intero non divisibile né per 2 né per 5, allora esiste k tale che r divide
αk . Consideriamo, al variare di n, i numeri αn mod r (i resti degli αn nella divisione per r); questi sono
infiniti numeri, tutti compresi fra 0 e r − 1, e quindi dovranno esistere due interi n e m, con n > m, tali
che αn mod r = αm mod r. Si ha quindi che αn − αm = r s con s intero:
DEFINT a-z
’ le variabili sono intere
INPUT "r: ", r
k = 1
’ inizializzazioni
a = 1
’ a(1) = 1
IF r * INT( a / r ) = r THEN GOTO fine
k = k + 1
’ prova l’indice successivo
a = 10 * a + 1
’ calcola il relativo a(k)
GOTO ciclo
PRINT k, a, a / r
END
1988.4
Sia assegnato su un piano un numero n arbitrario di triangoli con la proprietà che tre qualsiasi
di essi abbiano almeno un punto in comune. Si dimostri che tutti i triangoli assegnati contengono uno
stesso punto. Come occorre modificare l’ipotesi perché la stessa conclusione valga per un numero finito di
triangoli nello spazio?
RISOLUZIONE
La tesi è vera per n = 3; supponiamo che sia vera per ogni insieme di n − 1 triangoli e
dimostriamo con un ragionamento per assurdo che deve valere anche per un insieme di n triangoli.
Siano assegnati i triangoli T1 , . . . , Tn−1 , Tn con la proprietà che tre qualsiasi di essi abbiano intersezione
non vuota, e supponiamo per assurdo che C = T1 ∩ · · · ∩ Tn−1 e Tn siano disgiunti.
Dato che C e Tn sono due insiemi convessi disgiunti nel piano, esiste una retta r che li lascia in
semipiani opposti.
Dato però che per ogni scelta di i e j risulta Ti ∩ Tj ∩ Tn 6= ∅, l’insieme Ti ∩ Tj contiene punti che stanno da
parti opposte rispetto a r, e quindi la famiglia di segmenti Si = Ti ∩ r gode della proprietà che Si ∩ Sj 6= ∅
per ogni i e j . Come dimostreremo fra un attimo, gli intervalli S1 , . . . , Sn−1 hanno intersezione non vuota,
e quindi C ∩ r è diverso dal vuoto, contro l’ipotesi.
Il risultato per i segmenti è ovvio nel caso n = 2; per il caso generale, ancora per assurdo, supponiamo che
S1 , . . . , Sn−1 abbiano intersezione non vuota e disgiunta da Sn . Allora è possibile trovare un punto P su r
che lascia da parti opposte il segmento Sn e l’intersezione degli altri. Ma ciò è assurdo, perche Si ∩ Sn è
non vuoto e dunque il punto P appartiene a tutti gli Si .
Una analoga argomentazione si può ripetere nello spazio tridimensionale se si hanno n triangoli tali che
quattro qualunque di essi abbiano un punto in comune, “separando” con un piano l’intersezione dei primi
n − 1 dall’n-mo e utilizzando il risultato precedente nel piano.
Nello spazio se si hanno dei triangoli che hanno a tre a tre intersezione non vuota la proprietà è falsa, come
mostra l’esempio dato dalle quattro facce di un tetraedro, cosı̀ come nel piano non basta che siano non vuote
solo le intersezioni fatte a due a due (fig. 1).
80
I problemi della Scuola Normale
Figure 1, 2
Anno 1988
81
1988.6*
Un laboratorio deve organizzare il trasferimento di 10 m3 di scorie radioattive liquide. Occorre
ordinare un numero n di contenitori, identici, che possano contenere tali scorie e garantire un trasporto
sicuro.
Si stima che il costo di ciascuno di tali contenitori sia 16 V 2 milioni di lire, ove V è il volume (in m3 ) di
scorie che ciascuno di essi può contenere; il costo di riempimento di ciascun contenitore risulta essere di
un milione di lire, indipendentemente dalla sua capienza.
(a) Quanti contenitori e di quale volume dovrà ordinare il laboratorio per spendere il meno possibile?
(b) Al momento di effettuare l’ordine si viene a sapere che la ditta fornitrice pratica sul prezzo dei contenitori
ordinati un piccolo sconto del k per cento, con k intero, se il loro numero uguaglia o supera le 50 unità
(il costo del riempimento rimane inalterato). Qual è il più piccolo k per il quale risulta conveniente
modificare l’ordine, e perché?
RISOLUZIONE
Si noti che la sola proprietà utilizzata dei triangoli è quella di essere una figura convessa. In effetti la tesi
è vera nel piano per figure convesse qualunque con a tre a tre punti in comune e nello spazio per figure
convesse (anche tridimensionali) che hanno a quattro a quattro un punto in comune. Il requisito della
convessità è fondamentale, come mostra la Figura 2.
Per il punto (a), se si indica con n il numero dei contenitori, il costo del trasferimento è, in milioni di lire,
C = 16 V 2 n + n ;
poiché V = 10/n si deve rendere minima la quantità
C = 16 n
1988.5*
Sia p(x) = a x4 + b x3 + c x2 + d x + e un polinomio con coefficienti razionali. Si supponga che,
per ogni intero m maggiore di un certo m0 , il numero p(m) sia intero. Si dimostri che allora 24 a è un
numero intero.
Si generalizzi questo risultato a polinomi p(x) con coefficienti razionali di grado qualsiasi.
Affrontiamo subito il caso generale: se pn (x) è un polinomio a coefficienti razionali che
assume valori interi per ogni intero m ≥ m0 , allora le stesse proprietà valgono anche per il polinomio
pn (x + 1), e anche il polinomio differenza
RISOLUZIONE
pn−1 = pn (x + 1) − pn (x)
ha coefficienti razionali e assume valori interi per ogni intero m ≥ m0 . Se il termine di grado massimo di
pn (x) è a xn , è immediato verificare che il termine di grado massimo di pn−1 sarà n a xn−1 . Analogamente,
se si considera
pn−2 = pn−1 (x + 1) − pn−1 (x) ,
si otterrà un polinomio con le stesse proprietà, di grado n − 2 e coefficiente direttore n (n − 1) a.
Cosı̀ proseguendo, dopo n passi si otterrà un polinomio di grado 0, p0 (x), con “coefficiente direttore”
n (n − 1) · · · 3 · 2 · 1 a = n! a. Dato che p0 (x) è costante, ed è intero per ogni intero m ≥ m0 , otteniamo che
n! a è intero.
Nel caso n = 4 si ha che 24 a = 4! a è intero.
10
n
2
+ n=
1600
n
+ n.
Poiché i due numeri 1600/n e n hanno prodotto costante, il valore minimo della loro somma si ha quando
sono uguali, ovvero per n = 40. Il costo complessivo è quindi C = 80 milioni.
Per il punto (b) notiamo che se k è la percentuale di sconto praticata, il nuovo costo è
16 (100 − k )
k 1
) +n=
+n
100 n
n
e affinché tale costo sia inferiore al precedente occorre che per qualche n ≥ 50 sia
C1 = 1600 (1 −
16 (100 − k )
n
+ n < 80 ,
ovvero
n2 − 80 n + 16 (100 − k ) < 0 .
√
Tale disequazione è soddisfatta nell’intervallo tra le due radici del trinomio, che sono 40 ± 4 k . Affinché
risulti
√ conveniente utilizzare lo sconto, che si applica solo per n ≥ 50, occorrerà dunque che risulti
4 k > 50 − 40, ovvero k > 100/16. Dato che k deve essere intero, lo sconto minimo per cui vale la pena
di modificare l’ordine è k = 7 (anche se il risparmio che si ottiene è solo di 240.000 lire!).
82
I problemi della Scuola Normale
1989.1*
Per 0 ≤ x ≤ π/2 e n = 0, 1, 2, . . . , si ponga:
Fn (x) = 1 − sin x + sin2 x − . . . + (−1)n sinn x.
Provare che:
(a) per 0 ≤ α ≤ 1/2, l’equazione F2k = α non ha soluzioni qualunque sia k naturale;
(b) per 1/2 < α ≤ 1, esiste un numero naturale k ∗ tale che, per ogni k > k ∗ , l’equazione F2k = α ha
almeno due soluzioni.
RISOLUZIONE
Cominciamo con l’osservare che Fn (x) · (1 + sin x) = 1 + (−1)n sinn+1 x e che dunque
1 + sin2k+1 x
.
1 + sin x
Per il punto (a) notiamo che se 0 ≤ x < π/2 risulta
F2k =
F2k = (1 + sin2k+1 x)/(1 + sin x) ≥ 1/(1 + sin x) > 1/2
Anno 1989
83
1989.2
Sia S una superficie sferica di centro O. Per ogni P ∈ S , sia fP : S → S l’applicazione che ad
ogni Q ∈ S associa il punto fP (Q) simmetrico di Q rispetto all’asse OP . Dimostrare che:
(a) per ogni P ∈ S l’applicazione fP è composizione di due simmetrie rispetto a piani;
(b) per ogni P , X , Y ∈ S la distanza tra X e Y è uguale alla distanza tra fP (X ) e fP (Y ).
(c) per ogni P , Q, X ∈ S si ha che fP (fQ (X )) = ffP (Q) (fP (X )).
RISOLUZIONE
L’applicazione fP associa ad ogni Q il punto fP (Q) simmetrico rispetto all’asse OP ; quindi
l’asse OP è perpendicolare al segmento QfP (Q) nel suo punto medio, e di conseguenza il punto fP (Q)
giace sulla circonferenza intersezione di S col piano perpendicolare ad OP passante per Q, e dunque fP (Q)
appartiene ad S .
(a) Si considerino due piani Π0 e Π00 passanti per OP e fra loro perpendicolari. Dato Q su S , sia Q0 il
punto simmetrico di Q rispetto a Π0 e Q00 il simmetrico di Q0 rispetto a Π00 . Se si taglia la figura con un
piano perpendicolare a OP e passante per Q, tale piano contiene sia Q0 sia Q00 e se C è la sua intersezione
con OP si ha
mentre per x = π/2 è
F2k = 1 > 1/2 ;
Q 00
di conseguenza l’equazione F2k = α, con α ≤ 1/2, non ha soluzioni su [0, π/2].
C
Per il punto (b), studiamo il comportamento delle funzioni F2k al crescere di k . Per ogni x con 0 ≤ x < π/2
la successione di numeri F2k (x) tende decrescendo a 1/(1 + sin x). La funzione g (x) = 1/(1 + sin x) è una
funzione continua, decrescente con minimo 1/2 assunto nel punto x = π/2.
Q
Q0
1
Figura 1
1/2
π/2
x
Figura 1
Per ogni α con 1/2 < α ≤ 1, esiste un punto x0 con x0 < π/2 per cui si ha g (x0 ) = α.
Se si considera il punto
b 0 + Q0 CQ
b 00 = (180◦ − 2 C Q
c0 Q) + (180◦ − 2 C Q
c0 Q00 ) =
QCQ
c0 Q) + C Q
c0 Q00 ) =
360◦ − 2 (C Q
360◦ − 2 · 90◦ = 180◦ .
Pertanto i punti Q, C , Q00 sono allineati e si ha ovviamente CQ = CQ0 = CQ00 ; dunque Q00 coincide con il
simmetrico di Q rispetto alla retta OP , cioè Q00 = fP (Q).
(b) È una conseguenza del punto precedente: in una simmetria rispetto a un piano α le distanze si
conservano perché il quadrilatero di vertici X , Y , Y 0 , X 0 (fig. 2), essendo simmetrico rispetto a α, è un
trapezio isoscele.
1 π
x0 +
2
2
si avrà che g (x̄) < g (x0 ) = α e siccome F2k (x̄) tende a g (x̄), da un certo k ∗ in poi si avrà F2k (x̄) − g (x̄) <
α − g (x̄), ovvero F2k (x̄) < α.
Siccome F2k (0) = F2k (π/2) = 1, per il teorema di esistenza dei valori intermedi vi saranno
almeno due punti, x1 e x2 , il primo nell’intervallo (0, x̄), il secondo nell’intervallo (x̄, π/2), per i quali
F2k (x1 ) = F2k (x2 ) = α.
x̄ =
X0
Y0
X
Y
α
Figura 2
84
I problemi della Scuola Normale
(c) Iniziamo osservando che se P e Q sono allineati con O (Q coincide con P o col suo simmetrico
rispetto ad O) la tesi è ovvia, perché in tal caso fP (Q) = Q, fP = fQ e fP (fQ (X )) = fP 2 (X ) = X =
fQ (fP (X )) = ffP (Q) (fP (X )) per ogni X . Per il caso in cui P e Q non sono allineati, ricordiamo che per
ogni K ∈ S l’applicazione fK è una isometria; inoltre, in quanto composizione di simmetrie, pensata come
trasformazione di tutto lo spazio in sé, è lineare: fK (x + y ) = fK (x) + fK (y ) e fK (αx) = αfK (x), per ogni
coppia di vettori dello spazio x, y e per ogni numero reale α.
E’ quindi sufficiente provare che la tesi vale per tutti i punti del piano σ passante per O, P e Q, e per un
altro punto T , non in σ, che possiamo prendere su S , e tale che OT sia ortogonale a σ.
Proviamo che fP (fQ (X )) = ffP (Q) (fP (X )) vale per ogni X di σ. Indichiamo per semplicità di notazione con
R il punto fP (Q). Sul piano σ l’azione di fP , fQ , fR è quella di una simmetria rispetto alle rette per OP ,
OQ, OR; conseguentemente la composizione fP ◦ fQ agisce come una rotazione intorno ad O di un angolo
pari al doppio di quello formato dalle semirette OQ e OP ; il punto R è il simmetrico di Q rispetto alla retta
OP , dunque l’angolo fra OP e OR è in valore assoluto e segno uguale a quello formato dalle semirette OQ
e OP ; dunque le due trasformazioni fP ◦ fQ e fR ◦ fP coincidono, e la tesi vale certamente per ogni punto
X appartenente a σ .
Indichiamo con T 0 il simmetrico di T rispetto ad O. L’applicazione fQ manda T nel simmetrico rispetto ad
OQ, e quindi in T 0 , dato che OT e OQ sono ortogonali, e lo stesso fa fP con T 0 ; a primo membro abbiamo
dunque fP (fQ (T )) = T ; per il secondo membro, essendo P , Q e fP (Q) complanari, OT è ortogonale anche
ad OfP (Q) e quindi ffP (Q) (fP (T )) = ffP (Q) (T 0 ) = T , e ciò conclude la dimostrazione.
Anno 1989
1989.3
85
Trovare le soluzioni reali del sistema:

2
2

2y + x − x − y = 0
z − x + y − y (x + z ) = 0


− 2 y + z − y2 − z 2 = 0
RISOLUZIONE
Sottraendo termine a termine la prima e la terza equazione otteniamo
1
(x − z ) (x + z − 1).
4
Sostituiamo l’espressione per y nella seconda equazione, ottenendo che
4 y + x − z − x2 + z 2 = 0, da cui y =
(z − x) +
(x − z ) (x + z − 1)
(1 − x − z ) = 0 ,
4
e quindi
(z − x) 1 +
(x + z − 1)2
= 0,
4
che equivale a
z = x.
Di conseguenza, dall’espressione per y , sappiamo che y = 0; ricaviamo finalmente x sostituendo nella prima
equazione, che diventa x − x2 = 0; le soluzioni reali del sistema sono date da:
x = 0, y = 0, z = 0 ;
x = 1, y = 0, z = 1 .
1989.4*
Consideriamo la legge che ad ogni punto P = (x, y ) del piano cartesiano fa corrispondere il
punto f (P ) dello stesso piano, definito da:
P
se OP ≤ 1
x
y ,
se OP ≥ 1
OP OP
dove O = (0, 0) e OP indica la distanza da O a P .
Provare che per ogni coppia di punti P e Q la distanza fra f (P ) e f (Q) non supera la distanza fra P e Q.
f (P ) =
RISOLUZIONE
Notiamo anzitutto che l’applicazione f associa ad ogni punto P del cerchio C di centro
l’origine e raggio unitario il punto stesso, mentre ad ogni punto P esterno associa il punto intersezione del
bordo di detto cerchio con la semiretta OP .
Se P e Q appartengono al cerchio C si ha f (P ) = P , f (Q) = Q e quindi, ovviamente dist(f (P ), f (Q)) =
dist(P, Q).
Se uno dei due punti, ad esempio P , appartiene a C , e l’altro è esterno (fig. 1) l’angolo P fd
(Q)Q è ottuso e
quindi
dist(f (P ), f (Q)) = dist(P, f (Q)) < dist(P, Q)
in quanto in un triangolo ottusangolo il lato opposto all’angolo ottuso è maggiore degli altri lati.
86
I problemi della Scuola Normale
y
87
P
y
Q
f(P )
( )
P = fP
f(Q)
O
O
x
f(Q)
Anno 1989
Q0
H
O
Q
A
P
x
N
Q
M
K
Figure 1, 2
B
Figura 1
Se poi sia P che Q sono esterni al cerchio C , si consideri la circonferenza che passa per il punto più vicino
all’origine, diciamo P , e sia Q0 la sua intersezione con la semiretta OQ. I triangoli di vertici O, f (P ), f (Q)
e O, P , Q0 sono simili e isosceli; essendo il segmento di estremi O, f (P ) minore del segmento di estremi
O, P si avrà
e allo sviluppo che ha la faccia superiore connessa al lato P N (e allora il percorso taglia sia lo spigolo P N
sia lo spigolo laterale uscente da N ).
dist(f (P ), f (Q)) < dist(P, Q0 ) ,
O
c0 Q è ottuso (in quanto OP Q0 è isoscele) e quindi per la stessa considerazione precedente
l’angolo P Q
Q
dist(P, Q0 ) < dist(P, Q) .
In conclusione, si è provato che in ogni caso
A
P LH
N
M
dist(f (P ), f (Q)) ≤ dist(P, Q) .
B
Figura 2
1989.5
Sia S la superficie di un prisma a base ottagonale regolare inscritto in un cilindro√circolare
retto di raggio R e altezza h = 3R sin(π/8). Siano A e B due punti come in figura, con OA = R/ 2.
P
O
A
N
M
Confrontiamo la lunghezza di tali percorsi. Posto α = π/8, si ha che la lunghezza del lato dell’ottagono è
2R sin α e la lunghezza dell’apotema è R cos
√α.
b = π/4 il triangolo ON A (fig. 2) è rettangolo in A e
Osserviamo che essendo ON = R, OA = R/ 2 e A√ON
b L = α e AL = R sin α/ 2 = LH . Pertanto la retta che passa per i punti A e H
isoscele. Ne segue che AN
forma un angolo di π/4 col lato orizzontale P N e quindi incontrerà la vertical uscente da M in un punto X
tale che XM = M H = 3R sin α, ovvero nel punto B . Pertanto il segmento AB è parallelo al lato QP della
base (fig. 2) e dunque il suo prolungamento incontra QO in un punto A0 che dista da O una quantità pari ad
OA (fig. 3). Il segmento A0 B 0 ha quindi la stessa lunghezza del segmento AB della Figura 1.
O
B
A0
Q
A
Determinare la lunghezza del minimo percorso su S tra A e B .
Un percorso minimo che congiunge A con B corrisponde ad un cammino rettilineo che
connette A e B nello sviluppo piano del prisma, cammino interamente contenuto nelle facce dello sviluppo.
È chiaro che vi sono solo due possibilità da prendere in considerazione, corrispondenti allo sviluppo che
vede la faccia superiore del prisma connessa al lato N M (nel qual caso il percorso taglia solo lo spigolo
NM)
P
RISOLUZIONE
K
B0
B
Figura 3
In conclusione ci siamo ridotti a confrontare le lunghezze dei segmenti AB e A0 B 0 della Figura 3.
88
I problemi della Scuola Normale
0
Si mandi la perpendicolare A K ad AB ; si ha
B0B √
KB = √ = 2 R sin α = AA0 ,
2
ma l’ipotenusa A0 B 0 del triangolo rettangolo A0 KB 0 è maggiore del cateto A0 K e pertanto
A0 B 0 > A0 K = AK + AA0 = AK + KB = AB .
Possiamo cosı̀ concludere che il percorso AB della Figura 2 è minore del percorso AB della Figura 1.
Anno 1990
89
1990.2
Sia P un poligono semplice (cioè tale che da ogni vertice escono esattamente due lati) non
necessariamente convesso, con almeno 4 lati. Supponiamo che P abbia al più un vertice concavo.
E’ vero che esistono due vertici non consecutivi con la proprietà che il segmento che li congiunge è contenuto
in P ? In caso affermativo dimostrarlo, altrimenti trovare un controesempio.
RISOLUZIONE
Se il poligono è convesso ogni diagonale è interamente contenuta in esso; se non è convesso
sia P il vertice del suo solo angolo concavo e siano A e B i vertici adiacenti ad esso; due dei semipiani
individuati dalle rette per A, P e per B , P tagliano il poligono in due figure convesse e una almeno di
esse contiene un quarto vertice C ; dunque il segmento P C congiunge due vertici non consecutivi ed è
interamente contenuto nel poligono.
C
1989.6
Sia f (x) una funzione a valori reali definita sulla semiretta reale {x ≥ 0}. Supponiamo che:
(a) f (x) sia derivabile con derivata f 0 continua;
(b) f (0) = 0;
(c) per ogni x ≥ 1 risulti
0 < f (x) ≤ xf 0 (x).
P
Provare che l’equazione f (x) = k ha almeno una soluzione x ≥ 0, per ogni k ≥ 0 .
A
Proveremo che la funzione f (x) tende a +∞ al tendere di x a +∞. Essendo f (0) = 0, dal
teorema dei valori intermedi seguirà che essa assume almeno una volta ogni valore k ≥ 0.
La funzione g (x) = f (x)/x ha derivata
RISOLUZIONE
f 0 (x) x − f (x)
x2
non negativa su [1, +∞); ne segue che g (x) è non decrescente su [1, +∞) e dunque si avrà
g 0 (x) =
g (x) ≥ g (1) = f (1) > 0 .
Pertanto per ogni x > 1 si ha
f (x) ≥ f (1) x
e quindi
lim f (x) = +∞ .
x→+∞
1990.1*
Considerare nello spazio euclideo nove punti distinti a coordinate intere. Dimostrare che ne
esistono due tali che il segmento che li congiunge contiene almeno un punto interno (cioè distinto dagli
estremi) a coordinate intere.
RISOLUZIONE
Come prima osservazione, notiamo che con meno di nove punti esistono controesempi;
basta prendere gli otto punti distinti dello spazio che hanno come possibili coordinate 0 oppure 1; nessun
segmento che abbia come estremi due di questi punti ha punti interni a coordinate intere.
Se P1 = (x1 , y1 , z1 ) e P2 = (x2 , y2 , z2 ) hanno coordinate intere, il segmento P1 P2 contiene certamente un
punto interno a coordinate intere, il suo punto medio, se x1 + x2 , y1 + y2 e z1 + z2 sono tutti e tre pari, e questo
accade se x1 e x2 sono entrambi pari o entrambi dispari, e analogamente per le altre coordinate. Le possibili
combinazioni di parità o disparità per le tre componenti sono solo 23 = 8 e dunque, per una qualsiasi scelta
di 9 punti a coordinate intere, ne esisteranno almeno due, Q e R tali che le relative componenti x siano
nella stessa classe di parità e cosı̀ per le componenti y e z . Quindi M , punto medio di Q e R, sarà interno
al segmento QR, e avrà coordinate intere.
B
Figure 1, 2
Si noti che se il poligono è intrecciato la proprietà non è più vera, come mostra la Figura 2.
90
1990.3
I problemi della Scuola Normale
Anno 1990
91
n
Dato il sistema

x + x2 + . . . + x100 = 5050

 1



 x22 − x21 = 3



 · · · ·

x2k − x2k−1 = 2k − 1




 · · · · · ·



 2
x100 − x299 = 199
trovare tutte le soluzioni x1 , x2 , . . . , x100 , con xk ≥ 0, k = 1, 2, . . . , 100.
RISOLUZIONE
Sommando le relazioni x22 − x21 = 3 , x23 − x22 = 5, . . . , x2k − x2k−1 = 2k − 1 possiamo ricavare
xk in funzione di k e della sola x1 , infatti
n−1
1990.4
Sia dato il polinomio F (x) = x + an−1 x
+ . . . + a0 con coefficienti ai interi. Supponiamo
che esistano quattro interi distinti a, b, c, d tali che F (a) = F (b) = F (c) = F (d) = 7.
Dimostrare che non esiste nessun numero intero k tale che F (k ) = 12.
Dato che il polinomio F (x) assume il valore 7 in a, b, c, d il polinomio G(x) = F (x) − 7 si
annulla in tali punti e dunque per il teorema di Ruffini si ha
RISOLUZIONE
G(x) = (x − a) (x − b) (x − c) (x − d) G1 (x)
con G1 (x) polinomio a coefficienti interi. Provare che per k intero F (k ) non può valere 12 equivale a provare
che G(k ) non assume il valore 5. Ma G(k ) = (k − a) (k − b) (k − c) (k − d) G1 (k ) e, dato che a, b, c, d sono
tutti distinti, almeno due fra i numeri k − a, k − b, k − c, k − d sono diversi da ±1 e quindi G(k ) non può
mai valere 5, che è un numero primo.
x2k − x21 = (x2k − x2k−1 ) + (x2k−1 − x2k−2 ) + . . . + x22 − x21 =
(2k − 1) + (2k − 3) + . . . + 3
e quindi
x2k − x21 = (k − 1)
p
(2 k − 1) + 3
= k2 − 1
2
da cui xk = k 2 − 1 + x21 ; si vede facilmente che xk è una funzione strettamente crescente di x1 . Inoltre,
se x1 = 1 si avrà xk = k e quindi x1 + x2 + . . . + x100 = 100 · 101/2 = 5050, per cui è verificata anche la prima
equazione.
Non vi sono altre soluzioni perché se x1 < 1 la somma delle cento incognite risulta strettamente minore di
5050, e se x1 > 1 risulta strettamente maggiore.
Trovare il più piccolo numero α > 1 tale che risulti:
1990.5
α + sin x
≤ ey−x
α + sin y
RISOLUZIONE
per ogni x, y con x ≤ y
Da α > 1 segue che α + sin y è sempre positivo; separando x e y otteniamo la diseguaglianza
equivalente
(α + sin x) ex ≤ (α + sin y ) ey
per ogni x, y con x ≤ y
che a sua volta equivale al fatto che la funzione fα (t) = (α + sin t) et è crescente.
La funzione fα (t) è derivabile; risulterà crescente se e solo se
fα0 (t) = (cos t + α + sin t) et ≥ 0 .
Deve dunque essere α ≥ −(cos t + sin t) per ogni t, e quindi il più piccolo α per cui vale la diseguaglianza
sarà dato dal massimo dei valori assunti da
√
−(cos t + sin t) = − 2 sin(t + π/4) ,
√
che si ha per x = −(3/4) π + 2 k π e vale 2.
92
I problemi della Scuola Normale
Anno 1990
1990.6*
Si consideri un rettangolo R di misure 6 × 4 metri. Nel punto di mezzo O di un lato è
incernierato un braccio articolato, della lunghezza totale di due metri, formato da due segmenti OA e AB
(vedi figura). Il braccio può muoversi solo all’interno di R. Più precisamente il segmento OA può ruotare
intorno al punto fisso O e, per ogni posizione assunta, il segmento AB può ruotare intorno al punto A;
naturalmente, durante il movimento, il braccio deve restare in R.
93
R
B
R
A0
B
Figura 3
A
O
E’ possibile scegliere le lunghezze dei segmenti in modo che ogni punto di R a distanza minore o uguale a
due metri da O sia raggiunto da B ?
Indichiamo con r la lunghezza del braccio OA e con r0 quella del braccio AB . Si ha
OA + AB = 2 m.
Per la diseguaglianza triangolare si ha
RISOLUZIONE
|OA − AB| ≤ OB ≤ OA + AB
cioè
|r − r0 | ≤ OB ≤ r + r0 .
È quindi chiaro che se r 6= r0 i punti interni del semicerchio di centro O e raggio |r − r0 | interno al rettangolo
R non sono raggiungibili dal braccio (fig. 1 e 2).
R
R
B
A
B
A
O
O
Figure 1, 2
Quindi la sola possibilità è che sia r = r0 = 1 m. Proviamo che in questo caso il problema si può
effettivamente risolvere.
Preso un qualunque punto B del semicerchio C di raggio 2 m e centro in O, si costruiscano i due triangoli
isosceli OAB , OA0 B di base AB e aventi lati OA = AB = OA0 = A0 B = 1 m. I punti A e A0 sono simmetrici
rispetto alla retta OB , cioè ciascuno dei due appartiene ad una delle due semicirconferenze di centro in O e
raggio 1 m suddivise dalla retta per O e B , il che implica che uno dei punti A, A0 è interno al semicerchio
C e quindi l’intero triangolo formato da tale punto e dai punti O e B è interno a C, pertanto il problema è
risolto.
A
O
94
I problemi della Scuola Normale
Provare che per ogni numero intero n ≥ 2 si ha
1991.1
2
√
n
95
1991.2
Fra tutti i quadrilateri convessi inscritti in un quadrato, in modo che ogni lato del quadrato
contenga almeno un vertice del quadrilatero, si determinino quelli aventi minimo e massimo perimetro.
√
n+1
n
n! <
e che (n + 1)/2 non è mai un multiplo intero di
Anno 1991
RISOLUZIONE
Determiniamo per prima cosa i quadrilateri M N OP inscritti di perimetro massimo.
n!.
√
D
Applichiamo la diseguaglianza fra le medie geometrica e aritmetica, ab <
(a + b)/2 , valida per ogni coppia a, b di numeri positivi diversi fra loro, ai prodotti 1 · n, 2 · (n − 1), . . . ,
(n − 1) · 2, n · 1 ; moltiplicando le n diseguaglianze otteniamo
RISOLUZIONE
n! =
√
p
p
Per il secondo punto dobbiamo provare che per nessun k intero vale l’uguaglianza
k
n! =
C
P
N
√
1 · n 2 · (n − 1) . . . (n − 1) · 2 n · 1 <
(n − 1) + 2 n + 1
1 + n 2 + (n − 1)
···
=
2
2
2
n 2
n+1
2
da cui, estrendo la radice n-esima, otteniamo la diseguaglianza richiesta.
√
n
O
n+1
2
o, equivalentemente, che non è mai verificata
(2k )n · n! = (n + 1)n .
Ma l’ultima uguaglianza non può essere soddisfatta per nessun k perché n e n + 1 sono sempre primi tra loro,
e quindi ogni numero primo che divide n divide il primo membro dell’uguaglianza ma non può dividere il
secondo membro.
A
M
B
Figura 1
Per la diseguaglianza triangolare abbiamo che
M N ≤ M B + BN ,
OP ≤ OD + DP ,
N O ≤ N C + CO ,
P M ≤ P A + AM ,
da cui, sommando, otteniamo
M N + N O + OP + P M ≤ AB + BC + CD + DA .
Dunque, ogni quadrilatero inscritto in ABCD ha perimetro non superiore a quello di ABCD, e l’uguaglianza
si ottiene scegliendo i vertici di M N OP coincidenti con quelli del quadrato.
D
O
C
O0
D0
3
4
N0
P
N
2
A
M
1
B
A0
Figura 2
Cerchiamo ora di determinare quali siano i quadrilateri che hanno il perimetro minimo fra quelli inscritti.
Denotiamo con 1, 2, 3, 4 i lati AB , BC , CD, DA del quadrato e costruiamo l’immagine speculare di ABCD
rispetto al segmento 2; se N 0 è l’intersezione col lato 2 della retta per M e per O0 , immagine di O, dalla
diseguaglianza triangolare si ha M N 0 + N 0 O0 = M O0 ≤ M N + N O0 = M N + N O.
Pertanto il percorso di lunghezza minima da M a O si avrà quando N coincide con N 0 .
Alla stessa stregua riflettiamo il quadrato attorno al lato 3 e poi al lato 4.
96
I problemi della Scuola Normale
A 00 M 000
Anno 1991
97
Analogamente, supponiamo per assurdo che p sia multiplo di 3, p = 3 k .
sostituendo otteniamo 2 p2 − 1 = 18 k 2 − 1 = 9 (2 k 2 − 1) + 8 e quindi dividendo 2 p2 − 1 per 9 si ha resto 8,
mentre i resti ottenibili da un quadrato perfetto, per l’identità (9 r + s)2 = 9 (9 r2 + 18 r s) + s2 , sono tutti e
soli quelli che si hanno dai quadrati dei numeri da 0 a 8 e cioè, nell’ordine:
0,
1,
4,
0,
7,
7,
0,
4,
1,
2
A M
A M
A0
e dunque se 2 p − 1 è un quadrato, allora p non è multiplo di 3.
In conclusione se 2 p2 − 1 è un quadrato perfetto il numero N = p2 − 1 è divisibile per N0 = 24.
Figure 3, 4
Il percorso di lunghezza minima che connette il punto M con la sua immagine M 000 dopo le tre riflessioni
è quello rettilineo. Siccome ovviamente AM = A00 M√000 la retta per M e M 000 forma angoli di 45◦ con i lati
del quadrato e quindi tale percorso ha lunghezza 2 2 AB . Sul quadrato originario ciò corrisponde a un
percorso che da M torna a M dopo aver toccato i lati 2, 3, 4 lungo i lati di un rettangolo con lati paralleli
alle diagonali del quadrato. La lunghezza di tale percorso non dipende dalla posizione del punto M su AB
e quindi il problema ammette infinite soluzioni (fig. 6).
M
Figure 5, 6
1991.4
Su un treno, inizialmente senza passeggeri e formato da n carrozze, salgono k viaggiatori
disponendosi in modo casuale e indipendente l’uno dall’altro. Qual è la probabilità che solo tre carrozze
siano occupate da almeno un viaggiatore?
RISOLUZIONE
I k passeggeri si distribuiscono sulle n carrozze, che possiamo supporre contraddistinte dalle
etichette c1 , c2 , . . . , cn , in un numero di modi distinti dato da nk . Vogliamo calcolare il numero dei casi
in cui solo le carrozze ci1 , ci2 e ci3 risultano occupate da almeno un passeggero. Qualunque scelta delle
carrozze i1 , i2 e i3 con 1 ≤ i1 < i2 < i3 ≤ n porta allo stesso numero di disposizioni accettabili che si
ottiene per i1 = 1, i2 = 2 e i3 = 3 e quindi, indicando tale numero con S (k, 3), i casi favorevoli saranno
dati
da S (k, 3) moltiplicato per il numero di modi distinti di scegliere tre delle n carrozze, cioè Cn,3 = n3 .
Calcoliamo quindi il numero S (k, 3) di modi distinti in cui k passeggeri occupano 3 carrozze, senza lasciarne
nessuna vuota. Ogni viaggiatore ha 3 scelte, il che porta ad un numero totale di 3k disposizioni; fra queste
però vi sono anche quelle in cui nessun passeggero occupa la vettura c1 (pari a 2k possibilità), oppure la
vettura c2 oppure ancora la vettura c3 . Dobbiamo dunque escludere questi casi, che darebbero un totale di
3 · 2k ; tali casi però non sono disgiunti: i tre casi in cui tutti i passeggeri salgono sulla stessa carrozza sono
stati conteggiati due volte. Il numero di modi distinti in cui i k viaggiatori possono occupare le prime tre
carrozze è quindi dato da S (k, 3) = 3k − 3 · 2k + 3.
L’evento “solo 3 delle n carrozze hanno almeno un passeggero” ha quindi probabilità
P =
1991.3*
Trovare il più piccolo numero intero N0 ≥ 1 con la proprietà che N0 + 1 e 2N0 + 1 siano
entrambi quadrati perfetti.
Mostrare poi che ogni intero N con questa proprietà è multiplo di N0 .
RISOLUZIONE
E’ più comodo descrivere l’insieme degli interi N per cui N + 1 e 2 N + 1 sono quadrati
perfetti come l’insieme degli interi p2 − 1 tali che 2 p2 − 1 è un quadrato perfetto. Per verifica diretta, il più
piccolo intero p per cui 2 p2 − 1 è un quadrato perfetto è 5, e il numero N0 richiesto è 52 − 1 = 24.
Per il secondo punto, dobbiamo provare che se 2 p2 − 1 è un quadrato perfetto allora p2 − 1 è divisibile per
24 = 23 · 3.
Ma il numero p2 − 1 = (p + 1) (p − 1) è pari se e solo se p è dispari; in tal caso esso è prodotto di due pari
consecutivi e quindi è divisibile per 8. Inoltre, uno e uno solo dei numeri p − 1, p e p + 1 è divisibile per 3,
e quindi p2 − 1 è divisibile per 3 se e solo se p non è divisibile per 3.
Dobbiamo quindi far vedere che se 2 p2 − 1 è un quadrato perfetto, allora p è dispari, e non è divisibile per
3.
Supponiamo che p sia pari, p = 2 k . Dividendo per 8 il numero 2 p2 − 1 = 8 k 2 − 1 = 8 (k 2 − 1) + 7 il resto è
7, ma dall’identità
(4 r + s)2 = (16 r2 + 8 r s + s2 ) = 8 (2 r2 + r) + s2
vediamo che dividendo per 8 un quadrato perfetto i resti possibili sono solamente 0, 1 o 4. Ne deduciamo
che, se 2 p2 − 1 è un quadrato perfetto, allora p è dispari.
n
3
S (k, 3) n (n − 1) (n − 2) (3k − 3 · 2k + 3)
=
.
nk
6 nk
98
1991.5
I problemi della Scuola Normale
Anno 1991
99
Costruire un polinomio (a coefficienti reali)
r
P (x, y ) = a x2 + b x y + c y 2
verificante le proprietà:
(i) P (x, y ) = 0 soltanto per x = y = 0 ;
(ii) se x e y sono numeri interi allora anche P (x, y ) è intero.
Determinare poi il massimo della quantità
Q
M
P
P0
A
O
B
Z
∆ = b2 − 4 a c
al variare di P nell’insieme dei polinomi soddisfacenti le proprietà precedenti.
Se P (x, y ) = 0 e y 6= 0, allora x/y è una soluzione reale di a t2 + b t + c = 0, e dunque il
discriminante di tale equazione b2 − 4 a c è non negativo; analogamente, se P (x, y ) = 0 e x 6= 0, allora y/x è
una soluzione reale di c t2 + b t + a = 0, e dunque ancora b2 − 4 a c ≥ 0.
Un polinomio P verifica quindi l’ipotesi (i) se e solo se b2 − 4ac < 0.
Per la seconda condizione, se P (x, y ) è intero per x, y interi, avremo che P (1, 0) = a, P (0, 1) = c e
P (1, 1) = a + b + c sono tutti interi, e quindi a, b, c debbono essere interi.
La funzione ∆ = b2 − 4 a c assume solo valori interi negativi; il polinomio P (x, y ) = x2 + x y + y 2 verifica
le ipotesi (i) e (ii) e fornisce −3 come valore di ∆. Vediamo se le equazioni ∆ = −1 o ∆ = −2 ammettono
soluzione.
∆ = b2 − 4 a c = −1 equivale a b2 + 1 = 4 a c, che non è mai verificata perché b2 + 1 non è mai divisibile per 4.
∆ = b2 − 4 a c = −2 equivale a b2 + 2 = 4 a c, che non è mai verificata perché b2 + 2 non è mai divisibile per 4.
Quindi il massimo valore della funzione ∆ è −3, e si ottiene, ad esempio, per P (x, y ) = x2 + x y + y 2 .
RISOLUZIONE
Figura 1
In entrambi i casi si riesce quindi ad ottenere un costo di costruzione inferiore prendendo un percorso
simmetrico rispetto a r.
Supponiamo dunque P Q parallelo ad AB , indichiamo con H e H 0 i piedi delle perpendicolari da P e Q a
AB e con x la lunghezza del segmento OH . Le tangenti da A e da B al cerchio formano un angolo 30◦ con
AB (AO ha lunghezza doppia del raggio).
P
Q
x
A
1991.6*
Ci si propone di congiungere con una strada due località A e B che distano 4 km, fra le quali
si trova una zona Z costituita da terreno pietroso e avente la forma di un cerchio con centro nel punto medio
di AB e raggio di 1 km.
(a) Sapendo che, a parità di lunghezza, il costo di costruzione della strada nella zona pietrosa è λ volte
(λ numero reale maggiore di 1) quello relativo alla zona circostante, determinare due punti P , Q sul
bordo di Z in modo che il percorso AP QB (vedi figura) sia il più economico possibile.
H0
B
Figura 2
1
≤ x ≤ 1.
2
Z
O
O
La geometria del problema impone dunque la condizione
Q
P
A
H
B
Si ha AP =
p
(2 − x)2 + 1 − x2 =
F (x) = λ x +
(b) Discutere poi il caso più generale in cui si considerano percorsi formati, oltre che da tratti rettilinei,
anche da eventuali tratti curvilinei contenuti nel bordo di Z (dove il costo unitario di costruzione si può
considerare lo stesso che nella zona esterna a Z).
RISOLUZIONE
(a) Ci proponiamo anzitutto di dimostrare che il percorso cercato è simmetrico rispetto alla perpendicolare
r al segmento AB passante per il centro O.
Supponiamo che il percorso ottimale sia AP QB e sia M l’intersezione del segmento P Q con la retta r. Sia
C1 il costo di costruzione del tratto AP M e C2 quello tratto M QB . Si dovrà avere C1 ≤ C2 oppure C1 > C2 .
Nel primo caso, sia P 0 il simmetrico di P rispetto a r; il percorso M P 0 B avrà costo C1 e, essendo
P P 0 < P M + M P 0 , il percorso AP P 0 B avrà costo inferiore a quello di AP M P 0 B che è 2 C1 ≤ C1 + C2 .
Allo stesso modo si ragiona se C2 < C1 , considerando il percorso AQ0 QB , ove Q0 è il simmetrico di Q
rispetto a r.
p
√
5 − 4 x e dunque il costo di costruzione sarà proporzionale a
5− 4x .
Cerchiamo i punti di minimo di tale funzione su [1/2, 1]. Si ha
2
5 − 4x
F 0 (x) = λ − √
√
e dunque F 0 (x0 ) = 0 per x0 = 5/4 − 1/λ2 . Il punto x0 appartiene a (1/2, 1) solo se 2/ 3 < λ < 2, e in tal
caso F (x) ha in x0 un massimo, dato che a sinistra di x0 si ha F 0 (x) > 0 mentre a destra di x0 è F 0 (x) < 0.
Pertanto il minimo di √
F (x) è da ricercarsi agli estremi dell’intervallo
√ [1/2, 1].
√
Si ha F (1/2) = λ/2 + 3, F (1) = λ + 1 e quindi√se λ + 1 ≤ λ/2 + 3, ovvero se λ ≤ 2 ( 3 − 1), il percorso
ottimale sarà quello rettilineo AB , se λ > 2 ( 3 − 1) il percorso più economico sarà costituito dai due
segmenti AT , BT 0 tangenti al cerchio e dal tratto T T 0 .
100
I problemi della Scuola Normale
P
Anno 1992
1992.1*
Q
101
E’ assegnata una legge che a ogni coppia di interi x, y associa un intero x y in modo che
x (y + z ) = y x + z x
per tutti gli interi x, y , z . Si dimostri che
A
x y = x y (1 1).
B
RISOLUZIONE
Osserviamo intanto che
x x = x (x + 0) = x x + 0 x ,
e quindi
0x = 0 .
Figure 4, 5
Di conseguenza,
(b) Volendo considerare anche tratti curvilinei sul bordo di Z la sola possibilità da prendere in esame è
_
quella del percorso costituito dai segmenti AT , BT 0 e dall’arco T T 0 . Si vede facilmente infatti che altri
percorsi avrebbero lunghezza, e quindi costo, maggiore.
x y = x (y + 0) = y x + 0 x = y x ;
dunque l’operazione è commutativa, e in particolare si ha che
x (y + z ) = y x + z x = x y + x z
(vale la proprietà distributiva rispetto alla somma).
Dimostriamo usando il principio di induzione che se y ≥ 1 si ha x y = y (x 1). Per y = 1 la tesi
è immediata; dalla proprietà distributiva segue che se
x y = y (x 1)
A
B
vale per un certo y , allora
x (y + 1) = x y + x 1 = y (x 1) + x 1 = (y + 1) (x 1) .
Se y = 0 si ha che x y = 0 = 0 · (1 1).
Se y < 0, osserviamo che
x y + x (−y ) = x (y − y ) = x 0 = 0 ,
Figura 6
_
Il costo di costruzione del percorso AT , T T 0 e T 0 B è proporzionale a
√
√
π
3 π
+ =2 3+
2
3
3
e dunque detto percorso avrà costo inferiore a quello AT T 0 B se
2·2
√
2
λ>2 3+ π .
3
e dunque
x y = −(x (−y )) = −((−y ) (x 1)) = y (x 1) .
Possiamo quindi concludere:
x y = y (x 1) = y (1 x) = y (x (1 1)) = x y (1 1) .
102
I problemi della Scuola Normale
0
0
0
0
Nel piano due quadrati ABCD e A B C D sono disposti come in figura.
1992.2
C
B
A
103
1992.3
Verificare che la somma delle quarte potenze di due numeri reali di assegnato prodotto p > 0
a) decresce se decresce il valore assoluto della differenza dei due numeri;
b) raggiunge il valore minimo quando i due numeri sono uguali.
RISOLUZIONE
B0
D
Anno 1992
Supponiamo di aver due coppie, x1 e y1 , x2 e y2 , di numeri tali che x1 y1 = x2 y2 = p, e tali
che sia
x41 + y14 ≥ x42 + y24 .
A0
C0
Sommando a primo e secondo membro
2 p2 = 2 x21 y12 = 2 x22 y22 ,
D0
otteniamo una diseguaglianza equivalente che possiamo scrivere come
Si dimostri che la retta passante per A e perpendicolare a DD0 incontra il segmento BB 0 nel punto medio.
RISOLUZIONE
0
Si mandi per B la parallela ad AB e sia Q il punto in cui essa interseca la perpendicolare
AP a DD0 .
Sottraendo ora da primo e secondo membro il termine
B
2 p = 2 x1 y1 = 2 x2 y2 ,
M
si ottiene la diseguaglianza
B0
A
D
(x1 − y1 )2 ≥ (x2 − y2 )2 ,
A0
C0
P
D0
Figura 1
b 0 = DD
c0 A perché entrambi gli angoli sono complemenSi considerino i triangoli ADD e B AQ. Si ha QAB
b 0 è retto e AP D0 è un triangolo rettangolo). Dato che per costruzione B 0 Q è parallelo
b 0 (D0 AB
tari a P AD
b perché sono entrambi angoli
b 0 è uguale a B AQ
b , e questo a sua volta è uguale a D0 DA
ad AB , l’angolo AQB
b .
complementari a P AD
I triangoli ADD0 e B 0 AQ hanno quindi tutti gli angoli uguali e, avendo uguale anche un lato (AD0 = AB 0 ),
sono tra loro uguali. Pertanto B 0 Q = AD = AB ; ne segue che il quadrilatero ABQB 0 è un parallelogramma
e dunque la diagonale AQ biseca l’altra diagonale BB 0 .
0
0
che, ancora, equivale a
x21 + y12 ≥ x22 + y22 .
Q
C
(x21 + y12 )2 ≥ (x22 + y22 )2
che a sua volta è equivalente a
|x1 − y1 | ≥ |x2 − y2 | .
Questo prova il punto (a).
Il punto (b) segue immediatamente dal fatto che x4 + y 4 è minima quando lo è |x−y|, cioè quando x = y =
il minimo di x4 + y 4 sarà quindi dato da 2 p2 .
√
p;
104
I problemi della Scuola Normale
Anno 1992
Mostrare che, per ogni intero positivo fissato k , esiste almeno un intero n tale che
1992.4
100 ≤ nk + n ≤ 101 + k nk−1 .
RISOLUZIONE
La funzione f (n) = nk + n è una funzione crescente di n, qualunque sia l’intero positivo k ;
inoltre essa tende all’infinito al crescere di n. Siccome f (1) = 2, vi sarà un minimo numero intero n0 > 1
per il quale 100 ≤ f (n0 ). Dato che n0 è il più piccolo intero con tale proprietà, si avrà f (n0 − 1) < 100, cioè
(n0 − 1)k + (n0 − 1) < 100 .
[1]
1992.5*
Sia F (x) = a3 x3 + a2 x2 + a1 x + a0 un polinomio di terzo grado con coefficienti interi. Si
dimostri che
i) se p/q , con p, q interi primi tra loro e q 6= 0, è una radice del polinomio, per ogni intero m il numero
F (m) è divisibile per p − m q ;
ii) se esistono due interi x1 e x2 tali che F (x1 ) = 1, F (x2 ) = −1, e che |x1 − x2 | > 2, allora F (x) non ha
radici razionali.
RISOLUZIONE
ak − k ak−1 ≤ (a − 1)k .
[2]
Usufruendo di [1] e di [2] si ricava
1
nk0 − k nk−
+ n0 − 1 ≤ (n0 − 1)k + n0 − 1 < 100 ,
0
da cui segue che q è un divisore di a3 , cioè a3 = A q con A intero, e che p è un divisore di a0 , cioè a0 = C p
con C intero.
Siccome p/q è radice del polinomio, per il teorema di Ruffini si avrà F (x) = (x − p/q ) P (x) con
P (x) polinomio di secondo grado a coefficienti razionali, e quindi
F (x) = (q x − p) (a x2 + b x + c)
vale a dire
1
nk0 + n0 < 101 + k nk−
0
e quindi n0 è l’intero cercato.
Per dimostrare la [2] si può procedere per induzione su k . Ovviamente la proprietà è vera per k = 1; se si
suppone che la disuguaglianza sia soddisfatta per k − 1, cioè che valga
a2 = q b − p a ,
a1 = q c − p b
moltiplicando entrambi i membri per a − 1 si ottiene
a −ka
+ (k − 1) a
k−2
[1]
con a, b, c razionali.
Ma ovviamente a q = a3 = A q , e quindi a è intero; allo stesso modo −c p = a0 = C p e dunque c è intero.
Siccome
ak−1 − (k − 1) ak−2 ≤ (a − 1)k−1 ,
k−1
Supponiamo che sia F (p/q ) = 0; si avrà che
a3 p 3 + a2 p 2 q + a1 p q 2 + a0 q 3 = 0
Per ogni numero a > 1 si ha che
k
105
k
≤ (a − 1) ,
da cui
ak − k ak−1 ≤ (a − 1)k ;
sono numeri interi, ne segue che sia q b, sia p b sono interi.
Dato che p e q sono primi fra loro se ne deduce che anche b è un numero intero.
La [1] consente dunque di concludere che per ogni m intero F (m) è divisibile per q m − p.
Per il punto (ii), ragioniamo per assurdo, supponendo che p/q sia una radice razionale di F (x); per il punto
(i), F (m) è divisibile per p − m q . In particolare, per gli interi x1 e x2 risulta:
1 = F (x1 ) è divisibile per p − x1 q
dunque, in base al principio di induzione, la diseguaglianza è verificata per ogni intero positivo k e per ogni
numero a > 1.
−1 = F (x2 ) è divisibile per p − x2 q
e quindi p − x1 q = ±1, p − x2 q = ±1, da cui, tenendo conto del fatto che x1 6= x2 e q 6= 0, seguono due
possibilità
(
p − x1 q = −1
(
p − x1 q = 1
oppure
p − x2 q = 1
p − x2 q = −1
e sottraendo membro a membro otteniamo (x2 − x1 ) q = −2 oppure (x2 − x1 ) q = 2 e comunque |x2 − x1 | =
(2/q ) ≤ 2, contro l’ipotesi. Dunque, nelle ipotesi del punto (ii), F (x) non ha radici razionali.
106
I problemi della Scuola Normale
Anno 1992
107
1992.6*
1) Dimostrare che, presi comunque tre vertici di un cubo, il triangolo da essi individuato è rettangolo
oppure equilatero.
2) Calcolare la probabilità che tre distinti vertici del cubo, scelti a caso, individuino un triangolo rettangolo
(la probabilità è il rapporto tra il numero di casi favorevoli e il numero di casi possibili).
3) Si escludano tre vertici del cubo, e si considerino “ammissibili” i restanti cinque. Si indichi con P
la probabilità che tre vertici ammissibili del cubo, scelti a caso, individuino un triangolo rettangolo.
Stabilire quanti valori può assumere P al variare dei tre vertici esclusi all’inizio.
RISOLUZIONE
Per il primo punto osserviamo che, presi tre vertici del cubo, si hanno due casi:
a) due vertici appartengono ad uno stesso spigolo; in tal caso il terzo vertice o appartiene a una delle facce
che confluiscono in tale spigolo, e allora il triangolo è la “metà” di tale faccia, o appartiene allo spigolo
opposto, e in tal caso il triangolo è “metà” del rettangolo formato dai due spigoli e da due diagonali
delle facce.
Figura 1
In entrambi i casi il triangolo è rettangolo.
b) Se nessuna coppia di vertici appartiene ad uno stesso spigolo la sola possibilità è che il triangolo sia
formato da tre diagonali delle facce, e dunque il triangolo è equilatero.
Figura 3
Per il terzo punto osserviamo che, esclusi i vertici non ammissibili, vi sono 5 scelte possibili per il primo
vertice, 4 per il secondo e 3 per il terzo, ma ovviamente una permutazione dei vertici dà luogo allo stesso
triangolo.
Pertanto in ogni caso vi sono solo 5 · 4 · 3/3! = 60/6 = 10 triangoli possibili.
Se i tre vertici “proibiti” stanno su una stessa faccia è possibile formare con i 5 vertici rimasti un solo
triangolo con i lati che siano diagonali di facce. Pertanto la probabilità che un triangolo con vertici
ammissibili sia rettangolo è
10 − 1
9
=
.
10
10
Figura 2
Fissato un vertice del triangolo, diciamo A, vi sono 7 possibilità per il vertice B e, fissato anche B , altre 6
possibilità per il terzo vertice C . Dunque vi sono 7 · 6 = 42 scelte possibili; i triangoli distinti con un vertice
in A sono di fatto la metà, in quanto per ogni scelta di B e di C vi è la possibilità di scambiare tali vertici.
Dei 21 triangoli possibili, quelli formati da diagonali di facce sono solo 3. Infatti da A devono partire due
lati, e possiamo sceglierli in 3 modi distinti decidendo quale delle 3 diagonali di facce escludere.
Pertanto la probabilità che presi 3 vertici di un cubo essi formino un triangolo rettangolo è data da
Figura 4
La stessa situazione si ha se i tre vertici non ammissibili, pur non giacendo su una stessa faccia, sono tali
che due di essi sono vertici di uno stesso spigolo.
21 − 3 6
= .
21
7
La Figura 3 mostra i casi possibili.
Figura 5
108
I problemi della Scuola Normale
Se invece i tre vertici non ammissibili sono i secondi estremi degli spigoli che escono da un vertice, è
possibile formare quattro triangoli, con i vertici ammissibili, che hanno i lati che sono diagonali di facce, e
pertando la probabilità cercata è
10 − 4 3
= .
10
5
Anno 1993
109
Detto t l’angolo formato dal lato AD con l’asse x e H il piede della perpendicolare calata da P su AD si
avrà che l’angolo formato da P H con la verticale da P è uguale a t (fig. 2) e quindi l’ascissa di P è data
da a cos t + b sin t; analogamente si vede che l’ordinata di P è data da (1 − a) sin t + b cos t. Abbiamo cosı̀
ottenuto le equazioni parametriche del luogo descritto dal punto P :
(
x = a cos t + b sin t
[1]
y = (1 − a) sin t + b cos t .
Consideriamo la [1] come un sistema lineare in cos t, sin t. Se risulta a (1 − a) − b2 6= 0, tale sistema può
essere risolto in cos t, sin t ottenendo
cos t =
x (1 − a) − y b
,
a (1 − a) − b2
sin t =
ya − xb
.
a (1 − a) − b2
Quadrando e sommando, la relazione cos2 t + sin2 t = 1 diviene
(x (1 − a) − y b)2 + (y a − x b)2 = a (1 − a) − b2
ovvero
Figura 6
x2 (1 − a)2 + b2 − 2 x y b+y 2 a2 + b2 =
a(1 − a) − b2
1993.1*
Nel piano cartesiano riferito a coordinate ortogonali, un quadrato striscia in guisa che due
suoi vertici consecutivi appartengano rispettivamente all’asse delle ascisse non negative e a quello delle
ordinate non negative.
Descrivere analiticamente la traiettoria di un punto P interno al quadrato e rigidamente ancorato ad esso.
Caratterizzare le posizioni di P nel quadrato per le quali la traiettoria appartiene ad una circonferenza,
oppure è un segmento di retta.
D
P
,
[2]
2
Questa è l’equazione cartesiana soddisfatta dai punti del luogo descritto da P (per valori positivi di x, y );
essa rappresenta una circonferenza se e solo se il coefficiente del termine x y è nullo e i coefficienti di x2 e
y 2 sono uguali. Pertanto deve essere
b=0,
(1 − a)2 = a2 ,
ovvero a = 1/2, b = 0. Dunque l’unico punto del quadrato che si muove lungo un arco di circonferenza è
il punto medio del lato AD, e l’arco da esso descritto appartiene alla circonferenza di centro (0, 0) e raggio
1/2.
C
y
2
y
D
B
P
O
A
x
O
RISOLUZIONE
Supporremo che il quadrato abbia lato unitario. Siano A e D i vertici del quadrato che sono
a contatto rispettivamente con gli assi x e y e sia P un punto del quadrato che nella posizione iniziale (fig. 1)
ha coordinate (a, b).
y
y
B
D
P
b
a
D
Figure 1, 2
x
H
A
B
x
t
A
Figura 3
Se invece a (1 − a) − b2 = 0, allora le relazioni [1] comportano che x e y sono proporzionali e precisamente
e pertanto il punto P si muove lungo un segmento di retta che passa per l’origine. Pensando il quadrato
nella posizione iniziale la condizione a (1 − a) − b2 = 0 equivale a imporre che il punto P appartenga alla
semicirconferenza di equazione a2 + b2 = a, ossia (a − 1/2)2 + b2 = 1/4.
P
a b t
x
x a
=
y
b
C
C
A
110
I problemi della Scuola Normale
D
P
O
x
Figura 4
D’altra parte in ogni posizione la circonferenza che ha centro nel punto medio di AD e raggio 1/2 passa
per O e dunque l’angolo che il segmento P O forma con l’asse delle x è l’angolo che sottende l’arco P A e
dunque è costante, il che significa che, viceversa, ogni punto P della semicirconferenza sopra descritta si
muove lungo una retta. Si noti che in generale il luogo del punto P descritto dalle [2] è un arco di ellisse di
centro nell’origine (fig. 5).
y
RISOLUZIONE
Vi sono 22n percorsi possibili; occorre calcolare il numero dei percorsi che terminano nel
punto di ascissa 2 k con k = −n, . . . , −1, 0, 1, . . . , n (i punti di ascissa dispari non sono raggiungibili perché
il numero dei passi è pari, e si parte dall’ascissa 0). Si terminain (2 k,
0) se e solo se si sono fatti n + k passi
2n
modi. Dunque
verso destra e n − k passi verso sinistra, il che è possibile in
n+k
pn (2 k ) =
P
k = −n, . . . , −1, 0, 1, . . . , n .
1
22 n−2
2n − 2
n
≥
1
1
2 2n − 1
proviamola per n:
pn (2) =
A
2n
1
,
22 n n + k
La diseguaglianza cercata si può dimostrare per induzione su n. Essa è certamente soddisfatta per n = 1,
perché p1 (2) = 1/4 ≥ 1/(2 · 3). Supponendola vera per n − 1, cioè
pn−1 (2) =
D
O
111
1993.2
In un piano cartesiano un oggetto puntiforme parte dal punto (0, 2 n) (con n intero positivo)
e scende fino all’asse delle ascisse compiendo 2 n passi, con la seguente regola: se prima di compiere un
passo si trova nel punto di coordinate intere (k, l), può recarsi o in (k − 1, l − 1) o in (k + 1, l − 1) con uguale
probabilità.
Le mosse eseguite nei diversi passi sono indipendenti.
Si indichi con pn (k ) la probabilità che dopo 2 n passi l’oggetto si trovi nel punto (k, 0).
i) Calcolare pn (k ).
ii) Mostrare che 2 pn (2) ≥ 1/(2 n + 1).
y
A
Anno 1993
x
2n
1
=
22 n n + 1
1 (2 n)(2 n − 1)
pn−1 (2) ≥
4 (n + 1)(n − 1)
1 (2 n)(2 n − 1) 1
1
=
4 (n + 1)(n − 1) 2 2 n − 1
1 (2 n)(2 n + 1) 1 1
.
4 (n + 1)(n − 1) 2 2 n + 1
Siccome si vede facilmente che per ogni n
Figura 5
2 n (2 n + 1) ≥ 4 (n + 1) (n − 1)
la diseguaglianza cercata è dimostrata.
112
I problemi della Scuola Normale
Anno 1993
1993.3*
Dati tre numeri interi p > 2, q > 2, r > 2 si consideri un parallelepipedo di legno tale
che i tre spigoli uscenti da un vertice abbiano lunghezza p, q , r. Dopo aver dipinto la superficie esterna
del parallelepipedo, questo viene tagliato, mediante sezioni parallele alle facce, in cubetti aventi spigoli
di lunghezza 1. Ovviamente alcuni dei cubetti sono parzialmente colorati, mentre altri non sono colorati
affatto.
Si dimostri che esiste solo un numero finito di terne (p, q, r) per ciascuna delle quali il numero dei cubetti
parzialmente colorati è uguale al numero di quelli che non sono colorati affatto.
113
1993.4
Sia data una circonferenza Γ. Un arco circolare γ congiunge due punti distinti di Γ ed è interno
al cerchio C racchiuso da Γ. Dimostrare che, se le due regioni in cui γ divide C hanno aree uguali, la
lunghezza di γ supera il diametro di Γ.
RISOLUZIONE
Indichiamo con A e B gli estremi dell’arco γ e con M il suo punto medio. Il triangolo AM B
deve contenere al suo interno il centro O di Γ; se cosı̀ non fosse il diametro parallelo ad AB dividerebbe
il cerchio in due parti di egual area, una delle quali è interamente contenuta in una delle regioni in cui γ
divide il cerchio; tali regioni non potrebbero perciò avere area uguale.
RISOLUZIONE
I cubetti non colorati formano un parallelepipedo con spigoli di misura p − 2, q − 2 e r − 2.
Pertanto se il numero dei cubetti colorati è uguale a quello dei cubetti non colorati si avrà
p q r = 2 (p − 2) (q − 2) (r − 2) ,
[1]
O
che possiamo riscrivere come
1 p−2 q−2 r−2
=
.
2
p
q
r
Supponiamo che sia p ≤ q ≤ r; si avrà allora
p−2
p
3
≤
da cui
1 p−2
<
≤
2
p
1 p−2
<
,
2
p
r
3
A
[2]
M
O
A
M
B
B
Figura 1
Ciò osservato basterà notare che un arco di circonferenza ha lunghezza maggiore della corda che esso
sottende e dunque
_
_
_
AB = AM + M B > AM + M B .
1
2
e pertanto 4 < p < 10; dunque vi è solo un numero finito di possibilità per il più piccolo intero p. Con p
fissato l’equazione [2] diventa
p
2 (p − 2)
=
q−2 r−2
,
q
r
A
e con la stessa argomentazione si trova che
q−2
q
2
≤
p
2 (p − 2)
da cui
p
2 (p − 2)
<
q−2
≤
q
<
r
M
O
H
B
q−2
,
q
Figura 2
p
2 (p − 2)
< 1.
Per ogni p ammissibile vi è quindi un numero finito di q possibili e, fissati p e q , vi è (al più) un valore
possibile per r, il che conclude la dimostrazione.
D’altra parte AM > AO perché i triangoli rettangoli AM H e AOH hanno uno stesso cateto e si ha
OH < M H . Analogamente BM > BO e dunque
_
AB > AO + BO = diametro .
114
I problemi della Scuola Normale
1993.5
Sia n un intero positivo pari. Mostrare che si possono trovare (in modo non necessariamente
unico) n numeri reali a1 > a2 > . . . > an > 0 tali che
(
Anno 1993
Per il punto (b) mostriamo che la condizione a1 − an ≤ n è incompatibile con le altre due. Infatti per la
decrescenza delle ah si ha:
(a1 − a2 ) + (a3 − a4 ) + . . . + (an−1 − an ) =
a1 + a2 + . . . + an = 2 n
(a1 − an ) + (a3 − a2 ) + (a5 − a4 ) + . . . + (an−1 − an−2 ) <
(a1 − a2 ) + (a3 − a4 ) + . . . + (an−1 − an ) = n .
a1 − an ≤ n
Mostrare poi che questo non è possibile (per n > 2) se si impone la condizione ulteriore
contro l’ipotesi
a1 − an ≤ n .
Cominciamo osservando che, se n = 2, vi è una sola soluzione data da a1 = 3, a2 = 1.
Nel caso che sia n > 2 mostriamo che se esiste una soluzione allora possiamo costruirne infinite altre.
Consideriamo ad esempio i primi quattro numeri a1 > a2 > a3 > a4 e notiamo che è possibile trovare un
numero h abbastanza piccolo in modo che, posto a01 = a1 + h e a03 = a3 − h, si abbia ancora a01 > a2 > a03 > a4 ;
i numeri a01 , a2 , a03 , a4 . . . , an verificano per costruzione anche le due uguaglianze.
RISOLUZIONE
Per dimostrare l’esistenza cerchiamo di costruire gli n numeri della sequenza, cercando nel repertorio delle
progressioni più familiari.
Proviamo a vedere per prima cosa se le due condizioni possono essere verificate prendendo a1 , a2 , . . . , an
in progressione aritmetica, ak = a1 − (k − 1) q , con q > 0. Sostituendo nella seconda relazione si ha:
n = (a1 − a2 ) + (a3 − a4 ) + . . . + (an−1 − an ) = q · n/2 ,
e quindi deve essere q = 2; sostituendo nella prima relazione otteniamo
2n =
n
X
1
115
[a1 − 2 (k − 1)] = n a1 − 2
n
X
1
(k − 1) = n a1 − n (n − 1) ;
deve essere a1 = n + 1, e i numeri ak = (n + 1) − 2 (k − 1), con 1 ≤ k ≤ n verificano le condizioni richieste,
tranne quella di essere tutti positivi. Tentativo fallito.
Proviamo con ak in progressione geometrica, ak = a1 · q k−1 , con q compreso fra 0 e 1; se a1 > 0, le ak sono
positive e decrescenti.
Notiamo poi che anche i numeri a1 − a2 , a3 − a4 , a5 − a6 , . . . , an−1 − an sono i primi n/2 termini di
una progressione geometrica di ragione q 2 e primo termine (1 − q ) a1 . Imponiamo le due condizioni per
determinare a1 e q .
a1 + a2 + . . . + an = = a1 (1 + q + . . . + q n−1 ) =
1 − qn
= 2n
1−q
(a1 − a2 ) + (a3 − a4 ) + . . . + (an−1 − an ) =
a1
a1 (1 − q ) (1 + q 2 + . . . + (q 2 )(n/2)−1 ) =
a1 (1 − q )
a1
1 − qn
=
1 − q2
1 − qn
= n,
1+q
da cui
n
1 1 − qn
1 − qn
=
.
=
1+q
a1 2 1 − q
Deve essere quindi (1 + q )/2 = 1 − q , da cui q = 1/3, e sostituendo, ad esempio nella prima equazione,
otteniamo
1−q
4
n
a1 = 2 n
=
.
1 − q n 3 1 − (1/3)n
(a1 − a2 ) + (a3 − a4 ) + . . . + (an−1 − an ) = n .
116
I problemi della Scuola Normale
Anno 1994
Per ogni intero positivo n si indichi con θ(n) il numero reale, compreso fra 0 e 2 π , tale che
1993.6
1994.1
Sia X un insieme di n elementi, dove n è un numero pari e sia k un intero positivo. Diciamo che
una funzione f da X in X ha molteplicità k se per ogni a ∈ X l’insieme {x | f (x) = f (a)} ha k elementi.
Sono di più le funzioni di molteplicità 1 o quelle di molteplicità 2?
θ(n) = 3 n (mod 2 π )
cioè tale che θ(n) − 3 n sia un multiplo intero di 2 π .
i) Mostrare che 0 ≤ θ(n) ≤ π/2 per infiniti valori di n (si può usare il fatto che 3.14 < π < 3.15).
ii) Mostrare che θ(n) 6= θ(m) se n 6= m.
RISOLUZIONE
dunque che
π
2
per definizione di θ si ha quindi che
0 < θ(105) <
π
;
.
2
Mostriamo ora che, dato un intero n0 per il quale 0 < θ(n0 ) < π/2, se ne può trovare uno più grande che
gode della stessa proprietà.
Infatti, si scelga un intero k grande a sufficienza in modo che sia
2π
k
< θ ( n0 ) <
π
2
−
2π
k
C (2 m, m) ·
θ(n0 ), θ(n0 + 1), θ(n0 + 2), . . . , θ(n0 + k )
appartengono tutti a [0, 2 π ] e per il principio dei cassetti due di essi devono appartenere a uno
stesso intervallo. Ciò significa che esistono m, n distinti tali che
2π
k
[2]
e per ragioni di simmetria si può supporre m > n.
Se p, q , r sono gli interi per cui
θ ( n0 ) + 2 π p = 3 n0
θ ( m) + 2 π q = 3 m
θ ( n) + 2 π r = 3 n
se ne deduce che
θ(n0 ) + θ(m) − θ(n) = 3 (n0 + m − n) − 2 π (p + q − r) .
A causa delle diseguaglianze [1] e [2] il primo membro di questa uguaglianza è compreso fra 0 e π/2, e
dunque esso è uguale a θ(n0 + m − n). Si è cosı̀ trovato un intero n1 = n0 + m − n strettamente maggiore di
n0 tale che 0 < θ(n1 ) < π/2; proseguendo in questo modo se ne trovano infiniti.
La seconda parte segue dalla irrazionalità di π : se fosse θ(n) = θ(m) con n 6= m si avrebbe
3n − 2πj = 3m − 2πk
con j , k interi distinti, da cui
π=
3 m−n
2 k−j
il che è assurdo.
(2 · 1) (2m)!
2 m (2 m − 1) (2 m − 2)(2 m − 3)
...
= m .
2
2
2
2
Vi sono dunque
[1]
e si suddivida l’intervallo [0, 2 π ] in k intervalli di ampiezza 2 π/k (si noti che k è necessariamente maggiore
di 8). I k + 1 numeri
|θ(m) − θ(n)| <
Sia X un insieme di n = 2 m elementi. Le funzioni f da X in X di molteplicità 1 sono le
funzioni biunivoche di X in sé, e sono quindi tante quante le permutazioni di n oggetti, cioè n!.
Per contare le funzioni f da X in X di molteplicità 2, possiamo pensare di raggrupparle in base all’insieme
immagine: f1 e f2 stanno nello stesso sottoinsieme se f1 (X ) e f2 (X ) sono lo stesso sottoinsieme con m
elementi di X ; quindi il numero delle funzioni di molteplicità 2 è il prodotto del numero dei sottoinsiemi
con m elementi di un insieme con 2 m elementi (che è C (2 m, m), numero delle combinazioni di 2 m oggetti
ad m ad m), per il numero di funzioni di molteplicità 2 da un insieme X di 2 m oggetti in un insieme di m
oggetti, Y = {y1 , . . . , ym } .
Contiamo queste funzioni. Per ogni i = 0, 1, . . . , m − 1 dobbiamo scegliere i due punti di X che hanno come
immagine il punto yi ; questo è possibile farlo in (2 m − 2 i) (2 m − 2 i − 1)/2 modi distinti, e di conseguenza
il numero delle funzioni di molteplicità 2 da X in Y è
RISOLUZIONE
Da 3.14 < π < 3.15 otteniamo che 50 · 2 π < 315 e che 315 < 50 · 2 π + 1 < 50 · 2 π + π/2 e
0 < 3 · 105 − 50 · 2 π <
117
(2 m)! ((2 m)!)2
= m
2m
2 (m!)2
funzioni di molteplicità 2 da X in X ; il rapporto fra tale numero e il numero di quelle di molteplicità 1 è
((2 m)!)2 1
1 2 m(2 m − 1) . . . (m + 1)
=
=
2m (m!)2 (2 m)! 2m
m (m − 1) . . . 1
2 m(2 m − 1) . . . (m + 1)
> 1;
2 m (2 m − 2) . . . 2
Vi sono quindi (molte) più funzioni di molteplicità 2 che funzioni di molteplicità 1.
118
I problemi della Scuola Normale
Anno 1994
119
1994.2*
Vi sono 4 città collegate a due a due da 6 strade che non si intersecano (cioè ogni coppia
di città è collegata da una sola strada). Tutte le strade sono aperte al traffico con la stessa probabilità
p = 1/2.
Determinare la probabilità che in un determinato istante partendo da una qualsiasi città si possa arrivare
ad ogni altra città.
Per fissare le idee, tracciamo il grafo corrispondente ai collegamenti fra le quattro città,
numerando le strade.
RISOLUZIONE
4
1
5
3
Figura 3
6
2
Figura 1
Indichiamo con una sestupla di numeri lo stato delle strade: uno 0 in posizione i-esima indicherà che la
i-esima strada è chiusa al traffico, un 1 indicherà invece che essa è percorribile. Cosı̀ (110011) indicherà
che sono aperte la prima, la seconda, la quinta e la sesta strada, chiuse la terza e la quarta.
Vi sono 26 = 64 possibilità per lo stato delle strade, il numero delle sestuple distinte di 0 o 1, e, supponendo
che lo stato di una strada sia indipendente da quello delle altre, ciascuna di tali possibilità è equiprobabile.
Occorre dunque calcolare quante sono le sestuple che consentono da ogni città di raggiungere tutte le altre.
Ovviamente, con la sestupla di tutti 0 il traffico risulterà bloccato; neppure si riuscirà a raggiungere tutte le
città con una sola strada aperta (6 possibilità), né con due strade aperte (se esse sono consecutive risulteranno
collegate tre città, ma sarà isolata la quarta, e in caso contrario sarà possibile viaggiare tra due coppie di
città, ma non da una città di una coppia ad una dell‘altra). Le sestuple con due soli 1 sono 62 = 15.
Se vi sono tre strade aperte, occorre distinguere due casi: se tali strade formano un circuito chiuso, vi sarà
una città che rimane non collegata, e questo può avvenire in 4 modi diversi (le possibili scelte di 3 città su
4).
Con 4 “1” (quattro strade aperte, e 64 = 15 possibilità) si può sempre andare da una qualsiasi città a
qualunque altra, perché l’aggiunta di una strada aperta ai “casi peggiori” con tre strade (fig. 2) connette
necessariamente anche la quarta città. A maggior ragione ciò si potrà fare con cinque strade aperte (6
possibilità) o con tutte le strade aperte (1 possibilità).
In conclusione i casi favorevoli sono 16 + 15 + 6 + 1 e quindi la probabilità cercata è 38/64 = 19/32.
1994.3
Mostrare che 41 non può essere espresso come differenza di una potenza di 2 e di una potenza
di 3, cioè che non può sussistere nessuna delle due uguaglianze seguenti:
41 = 2n − 3m ,
41 = 3n − 2m
con n, m interi positivi.
RISOLUZIONE
Iniziamo provando che non è mai verificata 41 = 3n − 2m ; se per assurdo vi fossero due interi
n, m per cui l’uguaglianza vale, prendendo i resti modulo 3 del primo e del secondo membro dovremmo
avere che
41 mod 3 = 2 = −2m mod 3
m
e −2 mod 3 vale 2 per m pari e 1 per m dispari; quindi m deve essere pari, m = 2 k , e dunque esistono n,
k tali che 41 = 3n − 22k . Passando ai resti modulo 4 abbiamo
41 mod 4 = 1 = 3n mod 4
Figura 2
In tutti gli altri casi, che sono 63 − 4 = 16, sarà sempre possibile raggiungere da ogni città le altre tre;
infatti, o le tre strade sono consecutive e quindi, essendo il percorso aperto, connettono tutte e quattro le
città, o formano un percorso ramificato, nel qual caso escono tutte da una stessa città e dunque connettono,
tramite questa, le altre tre. In figura sono rappresentati i 16 casi possibili:
e 3n mod 4 vale 1 per n pari e 3 per n dispari; quindi n deve essere pari, n = 2 h, ed esistono h, k tali
che 41 = 32h − 22k = (3h + 2k ) (3h − 2k ) ; dato che 41 è primo, dovrebbe aversi che 3h − 2k = 1, e quindi
41 = (2k + 1) + 2k = 2k+1 + 1 e questo è impossibile perché 40 non è una potenza di 2.
Proviamo ora che non è mai verificata 41 = 2n − 3m ; se per assurdo vi fossero due interi n, m per cui
l’uguaglianza vale, prendendo i resti modulo 3 a primo e a secondo membro dovremmo avere che
41 mod 3 = 2 = 2n mod 3 ;
l’espressione 2n mod 3 vale 1 per n pari e 2 per n dispari e quindi n deve essere dispari, n = 2 h + 1; esistono
pertanto h, m tali che 41 = 22h+1 − 3m . Passando ai resti modulo 4 abbiamo
m
41 mod 4 = 1 = −3m mod 4 ,
ma −3 mod 4 vale 3 per m pari e 1 per m dispari, e quindi m deve essere dispari, m = 2 k + 1, ed esistono
h, k tali che 41 = 22h+1 − 32k+1 = 2 · 4h − 3 · 9k ; questo però è impossibile, perché passando ai resti modulo
8 avremmo che 41 mod 8 = 1, mentre
120
I problemi della Scuola Normale
h
k
Anno 1994
121
k
(2 · 4 − 3 · 9 ) mod 8 = −3 (8 + 1) mod 8 = 5 .
B
B
XA
KC
KC
P
KA
P
1994.4
Sia P un punto interno ad un triangolo equilatero. Per ogni retta passante per P siano X e Y i
due punti di intersezione tra la retta e i lati del triangolo. Determinare, per ogni punto P , la retta o le rette
che rendono minimo il prodotto
PX · PY
b con un’ampiezza di 60◦
RISOLUZIONE
Cominciamo col risolvere un sottoproblema: dato un angolo M ON
e un punto P in tale angolo, determinare le rette per P che rendono minimo il prodotto [1].
M
X
H ϑ
P
N
τ
O
K
Y
Figura 1
Indichiamo con H e K i piedi delle altezze condotte da P sui lati dell’angolo. Se indichiamo con ϑ e con τ
b e P Yb K abbiamo
gli angoli P XH
PX · PY =
PH
·
KB
A
[1]
YA
KB
A
C
C
Figure 2, 3
Per il risultato precedente applicato all’angolo Ab si ha che il valore minimo è vA = (4/3) P KC · P KB e
b si ha che il valore minimo è
si ottiene con la retta XA YA per P parallela a BC ; lavorando sull’angolo B
vB = (4/3) P KC · P KA , ottenuto con la retta XB YB parallela ad AC ; relativamente alle intersezioni con i
lati dell’angolo Cb si ha che il valore minimo è vC = (4/3) P KA · P KB , ottenuto con la retta XC YC parallela
ad AB e il valore minimo ottenibile è dunque il più piccolo fra vA , vB e vC .
Ricordando che la bisettrice AHA è il luogo dei punti P per cui si ha P KB = P KC , e analogamente per le
altre due bisettrici, possiamo cosı̀ concludere:
• se P è interno al quadrilatero AHC OHB , intersezione dei triangoli ABHB e ACHC , la soluzione è data
da vA , ed è realizzata da una unica retta, XA YA ;
• se P è interno al quadrilatero BHA OHC il minimo è vB , realizzato da XB YB ;
• se P è interno al quadrilatero CHB OHA il minimo è vC , realizzato da XC YC .
È chiaro che se P = O i valori vA , vB e vC coincidono, e sono ottenuti dalle rette XA YA , XB YB e XC YC ;
se invece P è interno al segmento OHA , i valori vB e vC coincidono, e sono ottenuti dalle due rette XB YB
e XC YC , e analogamente se P è interno a OHB oppure a OHC .
PK
sin ϑ sin τ
e quindi l’espressione [1] sarà minima quando ϑ e τ rendono massimo il prodotto
1
(cos(ϑ − τ ) − cos(ϑ + τ )) =
2
1
(cos(ϑ − τ ) − cos(120◦ )) ,
2
cioè quando ϑ = τ = 60◦ ; il triangolo OXY deve quindi essere equilatero e il valore minimo dell’espressione
[1] è dato da P H · P K / sin2 60◦ = (4/3) P H · P K .
sin ϑ · sin τ =
Torniamo ora al problema di partenza, consistente nel determinare, fissato P nel triangolo equilatero ABC ,
quali rette per P forniscano il valore minimo del prodotto [1].
1994.5*
Consideriamo un triangolo e dividiamo i suoi lati in n parti uguali mediante n − 1 punti
su ciascun lato. Congiungiamo ogni vertice con i punti cosı̀ ottenuti sul lato opposto. Si dimostri che se
n è primo maggiore di 2 allora non esistono punti appartenenti simultaneamente a tre dei segmenti cosı̀
costruiti.
RISOLUZIONE
Supponiamo che il punto P appartenga a tre delle rette considerate, AR, BS , CT e che sia
SA =
h
AC ,
n
TB =
k
AB ,
n
RC =
j
BC .
n
Tracciamo, come in Figura 1, la parallela a AC passante per B , e siano M e N le intersezioni con tale retta
delle rette per A e R e per C e T rispettivamente.
N
B
M
R
T
A
Figura 1
S
C
122
I problemi della Scuola Normale
Per similitudine si avrà
SA BM
=
,
SC
BN
123
SECONDA SOLUZIONE
RC
CA
=
,
RB BM
T B BN
=
.
TA
CA
Moltiplicando fra loro tali relazioni si ha
SA RC T B
=1
SC RB T A
e quindi
ovvero
Anno 1994
Ad ogni punto P di un triangolo P1 P2 P3 corrisponde in modo univoco la terna di numeri non negativi
(a1 , a2 , a3 ) con a1 + a2 + a3 = 1, tali che
P = a1 P1 + a2 P2 + a3 P3
dove, se α è un numero reale e A, B sono i due punti A = (x1 , y1 ) e B = (x2 , y2 ), la scrittura α A indica
il punto di coordinate cartesiane (α x1 , α y1 ) e con A + B si intende il punto (x1 + x2 , y1 + y2 ). La terna
(a1 , a2 , a3 ) fornisce le coordinate baricentriche del punto P .
h
k
j
= 1,
n−h n−k n−j
P1
n3 − ( h + k + j ) n2 + ( h k + h j + k j ) n = 2 h k j .
Ne segue che 2 h k j , con 0 < h, k, j < n, è divisibile per n, ma questo non è possibile se n è primo diverso
da 2.
P
P1
P3
Figura 2
Determiniamo le coordinate baricentriche del punto ottenuto proiettando P da P1 sul lato opposto P2 P3 : la
semiretta che esce da P1 e passa per P ha equazione parametrica
[(1 − s) + sa1 ]P1 + sa2 P2 + sa3 P3
ed interseca P2 P3 quando (1 − s) + sa1 = 0, e dunque per s = 1/(1 − a1 ); la proiezione ha quindi coordinate
baricentriche (0, a2 /(1 − a1 ), a3 /(1 − a1 )).
Analogamente, la proiezione da P2 sul lato P1 P3 ha coordinate baricentriche (a1 /(1 − a2 ), 0, a3 /(1 − a2 )) e
la proiezione da P3 sul lato P1 P2 ha coordinate (a1 /(1 − a3 ), a2 /(1 − a3 ), 0).
Dobbiamo dimostrare che se n è un numero primo maggiore di due, per nessun punto (a1 , a2 , a3 ) le proiezioni
sui lati hanno coordinate baricentriche con componenti della forma i/n, con i intero, 0 < i < n.
Supponiamo che n, h1 , h2 , h3 siano numeri interi tali che
h3
h1
h2
(1 − a3 ) , a2 =
(1 − a1 ) , a3 =
(1 − a2 ) .
n
n
n
Siccome a1 + a2 + a3 = 1 possiamo riscrivere le uguaglianze precedenti come
a1 =
h3
h
h
( a1 + a2 ) , a2 = 1 ( a2 + a3 ) , a3 = 2 ( a1 + a3 ) .
n
n
n
Quindi a1 , a2 , a3 verificano la condizione a1 + a2 + a3 = 1 e risolvono il sistema


 ( n − h 3 ) a1 − h 3 a2 = 0
( n − h 1 ) a2 − h 1 a3 = 0


−h2 a1 + (n − h2 ) a3 = 0 .
a1 =
Perchè tale sistema ammetta soluzioni diverse da a1 = a2 = a3 = 0, il determinante della matrice associata
deve annullarsi, pertanto
det
"
n − h3
0
−h2
−h3
n − h1
0
0
−h1
n − h2
#
=
( n − h1 ) ( n − h2 ) ( n − h3 ) − h1 h2 h3 = 0
124
I problemi della Scuola Normale
Deve quindi essere
n3 − ( h1 + h2 + h3 ) n2 + ( h1 h2 + h1 h3 + h2 h3 ) n = 2 h1 h2 h3 .
Se n è primo e diverso da 2, per qualunque scelta di h1 , h2 , h3 fra 1 e n − 1, il secondo membro non è
divisibile per n, mentre il primo lo è, e dunque la tesi è dimostrata.
Anno 1995
125
1995.1
Siano C1 , C2 due circonferenze di centri e raggi rispettivi O1 , O2 e r1 , r2 . Dato un punto P
esterno alle due circonferenze si considerino le tangenti per P alle due circonferenze e siano M1 , N1 e M2 ,
N2 i rispettivi punti di contatto.
Si determini il luogo dei punti tali che P M1 2 + P M2 2 = 1.
Si determinino i punti P per cui P M1 2 + P M2 2 è minima.
RISOLUZIONE
1994.6
Siano a1 , a2 , . . . , an numeri reali e siano b1 , b2 , . . . , bn definiti da
bi = max (i · j − aj )
1≤j≤n
per ogni i = 1, 2, . . . , n.
O1
Allo stesso modo si costruiscono c1 , c2 , . . . , cn a partire da b1 , b2 , . . . , bn e poi d1 , d2 , . . . , dn a partire da
c1 , c2 , . . . , cn ; si dimostri che
ci ≤ a i
di = bi
RISOLUZIONE
N1
M1
r2
P
Scriviamo esplicitamente la dipendenza dei c1 , c2 , . . . , cn dai numeri a1 , a2 , . . . , an .
max (h · i − max (i · j − aj )) =
max
1≤j≤n
min (i · (h − j ) + aj ) .
1≤i≤n 1≤j≤n
Si vede allora che, qualunque sia i,
min (i · (h − j ) + aj )
1≤j≤n
è il minimo di un insieme di numeri fra i quali compare anche ah e dunque
ch = max
min (i · (h − j ) + aj ) ≤ max ah = ah ,
1≤i≤n 1≤j≤n
1≤i≤n
il che prova la prima parte della tesi.
Per la seconda parte, notiamo che da ai ≥ ci segue che, per ogni i e j risulta i · j − aj ≤ i · j − cj ; ma allora
il massimo dei numeri a primo membro al variare di j è minore o uguale del massimo al variare di j dei
numeri a secondo membro, e quindi:
bi = max (i · j − aj ) ≤ max (i · j − cj ) = di
1≤j≤n
M2
Figura 1
1≤i≤n
1≤i≤n
O2
Q
r1
per ogni i = 1, 2, . . . , n,
per ogni i = 1, 2, . . . , n.
ch = max (h · i − bi ) =
C2
N2
C1
1≤j≤n
Osserviamo che 1 = P M1 2 + P M2 2 = P O1 2 − r12 + P O2 2 − r22 ; il luogo è quindi descritto anche dalla
condizione P O1 2 + P O2 2 = 1 + r12 + r22 .
Sia Q il punto medio del segmento O1 O2 ; dal teorema della mediana applicato al triangolo O1 P O2
si ha
1
2 P Q2 = P O1 2 + P O2 2 − O1 O2 2
[1]
2
e quindi dovrà essere
P Q2 =
Se O1 O2 ≤
p
r
1 + r12 + r22 1
− O1 O2 2 .
2
4
2 (1 + r12 + r22 ) il luogo cercato giace sulla circonferenza di centro Q e raggio r dato da
1 + r12 + r22 1
− O1 O2 2 ,
2
4
e sarà rappresentato dai punti di tale circonferenza che sono esterni sia a C1 sia a C2 (eventualmente,
dall’intera circonferenza).
e dunque, per ogni i, risulta
P
bi ≤ di ;
P
ma i di sono costruiti a partire dai ci come i bi sono costruiti partendo dagli ai e dunque per quanto dimostrato
nella prima parte si ha anche che
r1
O1
di ≤ bi ,
e quindi deve essere
di = bi
per ogni i = 1, 2, . . . , n.
Figure 2, 3
Q
r2
O2
O1
Q
O2
126
I problemi della Scuola Normale
2
2
2
2
La quantità P M1 + P M2 è minima se e solo se è minima P O1 + P O2 e dalla [1] segue subito che ciò si
ha per P ≡ Q.
Anno 1995
127
1995.4
Sia f : R → R una funzione non negativa, con concavità rivolta verso il basso, derivabile e tale
che f 0 (0) > 0 e f (x) = f (2 − x) per ogni x ∈ R.
Dimostrare che
Z
2
f (x) dx ≤ 2f (1) −
0
1995.2*
Dati quattro punti distinti nel piano dimostrare che è sempre possibile sceglierne tre che
determinino un angolo inferiore o uguale a 45◦ . In generale dati n punti dimostrare che se ne possono
scegliere 3 che determinino un angolo inferiore o uguale a 180◦/n.
RISOLUZIONE
Formuleremo l’ipotesi che nessuna terna di punti sia allineata; in tal caso infatti si ha una
soluzione banale.
È sempre possibile, dati n punti nel piano (n ≥ 3), trovare un angolo convesso che ha vertice in uno di essi,
lati che passano per altri due punti, e che contiene i restanti n − 3 punti: basta infatti prendere l’involucro
convesso degli n punti, scegliere un vertice P del poligono che delimita l’involucro e le due semirette
che escono da P e passano per i due vertici adiacenti A e B .
A
A
P
P
[f (1) − f (0)]2
f 0 (0)
Provare che se la condizione f (x) = f (2 − x) non è verificata, tale diseguaglianza può non valere.
RISOLUZIONE
L’ipotesi f (x) = f (2 − x) implica che la funzione abbia derivata nulla nel punto x = 1: infatti
se h è un incremento positivo si ha f (1 + h) = f (2 − (1 − h)) = f (1 − h) e dunque i rapporti incrementali
destro e sinistro nel punto 1
f (1 + h) − f (1)
,
h
f (1 − h) − f (1)
−h
hanno segno opposto.
Poiché la funzione è derivabile per x = 1, e dunque derivata destra e sinistra coincidono, tale derivata deve
necessariamente essere 0.
Il grafico di ogni funzione concava e derivabile giace al di sotto di ogni sua retta tangente. Quindi il grafico
di f (x) nell’intervallo [0, 1] è contenuto nel trapezio delimitato dalla retta tangente in (0, f (0)), dalla retta
y = f (1), tangente in (1, f (1)), e dalle rette y = f (0) e x = 1.
C
B
y
B
f(1)
f(0)
Figure 1, 2
Consideriamo dapprima il caso n = 4 (fig. 2). Se l’angolo in P è ottuso allora nel triangolo AP B il più
piccolo fra gli angoli in A o in B deve essere minore di 45◦ , perché la somma degli angoli in A, in B e in
P vale 180◦ . Se viceversa tale angolo è retto o acuto, allora la retta congiungente P con il quarto vertice C
divide l’angolo in due parti, una almeno delle quali deve essere minore o uguale a 45◦ .
Nel caso generale si procede in modo simile. Scelto un angolo AP B che contiene i restanti punti all’interno
si hanno due casi: o tale angolo supera (1 − 2/n)180◦ , e allora nel triangolo AP B uno degli angoli in A o in
B deve essere minore di 180◦/n, oppure, in caso contrario, le rette che congiungono P con i restanti n − 3
punti dividono l’angolo in P in n − 2 angoli che, se fossero tutti maggiori di 180◦/n darebbero per l’angolo
in P una ampiezza più grande di (1 − 2/n)180◦ , contro l’ipotesi.
Si noti che il risultato non è migliorabile: nel caso del poligono regolare di n lati gli angoli formati da due
diagonali consecutive uscenti dallo stesso vertice valgono esattamente 180◦/n.
1995.3
3
3
3
Dimostrare che se a, b, c sono tre interi consecutivi allora a + b + c è multiplo di 9.
Supponiamo che sia a < b < c; uno dei tre numeri è multiplo di 3, e gli altri due sono
rispettivamente della forma 3 h + 1 e 3 k + 2. Il cubo del multiplo di 3 è certamente divisibile per 9; basta
provare che (3 h + 1)3 + (3 k + 2)3 è anch’esso multiplo di 9. Infatti
RISOLUZIONE
(3 h + 1)3 + (3 k + 2)3 =
3
2
3
2
27 h + 27 h + 9 h + 1 + 27 k + 54 k + 36 k + 8 =
9 (3 h3 + 3 h2 + h + 3 k 3 + 6 k 2 + 4 k + 1) .
1
2
x
Figura 1
La retta tangente in x = 0 ha equazione y = f (0) + f 0 (0) x e incontra la retta y = f (1) nel punto di ascissa
(f (1) − f (0))/f 0(0). L’area di detto trapezio è quindi
1
f (1) − f (0)
1+ 1−
2
f 0 (0)
(f (1) − f (0)) .
L’integrale di f (x) su [0, 1] sarà dunque minore o uguale a tale area sommata all’area del rettangolo di base
1 e altezza f (0), cioè
Z
1
f (x) dx ≤ f (1) −
0
1 [f (1) − f (0)]2
.
2
f 0 (0)
La tesi segue poi osservando che
Z
2
f (x) dx =
1
Z
2
1
f (2 − x) dx =
Z
1
f (x) dx .
0
Se la condizione f (x) = f (2 − x) non è soddisfatta la diseguaglianza è in generale falsa, come prova
l’esempio f (x) = x (3 − x) per cui si ha:
Z
0
2
f (x) dx =
4 8
10
[f (1) − f (0)]2
> 2 f (1) −
=2·2− = .
3
f 0 (0)
3 3
128
I problemi della Scuola Normale
2
2a
2a
1995.5
Dato A = (x, y ) ∈ R : |x| + |y|
variare di k ≥ 0 la classe C degli insiemi
Bk = (x, y ) ∈ R2 : |x| + |y| ≤ k
≤ 1 ove a è un numero reale positivo, si consideri al
contenuti in A. Determinare k in funzione di a in modo che Bk abbia area massima tra gli insiemi di C .
Gli insiemi Bk sono una famiglia di quadrati di centro (0, 0) e vertici sugli assi, e se k1 > k2
l’insieme corrispondente Bk1 contiene Bk2 ; quindi, per ogni fissato a, dobbiamo determinare il più grande
k per cui Bk sia contenuto in A.
RISOLUZIONE
a=2
a=1
a=1/4
Anno 1996
129
2
6
2
6
1995.6
Siano a, b numeri reali non negativi tali che b + b ≤ a − a . Dimostrare che allora risulta:
1) a ≤ 1;
2) b < 2/3.
RISOLUZIONE
1) Se fosse a > 1 si avrebbe a2 − a6 = a2 (1 − a4 ) < 0 e quindi nessun numero b può verificare la
diseguaglianza proposta.
2) Si ha a2 − a6 = a2 (1 − a2 ) (1 + a2 ) ≤ (1/4) · 2. Infatti il prodotto dei due numeri a2 e 1 − a2 , che hanno
somma indipendente da a, è massimo quando i due numeri sono uguali, cioè per a2 = 1/2, mentre il
fattore 1 + a2 non supera 2, per la prima parte.
Se fosse b ≥ 2/3 si dovrebbe avere
2
2
3
+
6
2
3
≤ b 2 + b 6 ≤ a2 − a6 ≤
1
,
2
il che è assurdo, come si verifica facilmente.
Figura 1
Per a = 1/2 gli insiemi A e B1 coincidono e quindi il massimo valore possibile per k è 1.
In ogni caso, se A contiene Bk deve contenere anche i suoi vertici (±k, 0), (0, ±k ), da cui segue che k 2a ≤ 1.
Ciò significa che k non può superare 1, qualunque sia il valore di a.
Se a > 1/2 l’insieme A contiene B1 : infatti se (x, y ) appartiene a B1 si ha |x| + |y| ≤ 1, e dalla disuguaglianza
|x|2a + |y|2a ≤ |x| + |y| ≤ 1 segue che (x, y ) appartiene anche ad A; quindi k = 1 è un valore ammissibile ed
è anche il massimo.
Se a ≤ 1/2 le diseguaglianze si invertono e per trovare kmax dobbiamo determinare il più piccolo valore
assunto da x + y quando x2a + y 2a = 1 e x, y > 0. Parametrizziamo questo insieme ponendo xa = cos θ e
y a = sin θ con 0 < θ < π/2, e cerchiamo il minimo di
1996.1*
Dato un quadrato ABCD di lato unitario, determinare la massima costante α e la minima
costante β per cui si ha
α ≤ PA + PB + PC + PD ≤ β
per ogni punto P contenuto nel quadrato.
Per il minimo osserviamo che α ≥ min(P A + P C ) + min(P B + P D) = AC + BD per la
diseguaglianza triangolare; d’altra parte AC + BD è un valore assunto da P A + P B + P C + P D,
se P è l’intersezione delle diagonali del quadrato.
RISOLUZIONE
x + y = (cos θ)1/a + (sin θ)1/a = f (θ)
D
con 0 < θ < π/2. Determiniamo i valori per cui f 0 (θ) si annulla
C
P
f 0 (θ) = (cos θ)1/a−1 · (− sin θ) + (sin θ)1/a−1 · cos θ = 0
da cui
sin θ
=
cos θ
sin θ
cos θ
1/a−1
;
per 0 < θ < π/2 l’unica soluzione è θ = π/4, che è un punto di minimo perché f (0) = f (π/2) = 1 e f 0 (θ) < 0
per θ vicino a 0. Quindi
√
kmax = min{f (0), f (π/4)} = min{1, 2 (1/ 2)1/a } = 21−1/(2a) .
A
B
Figura 1
√
Quindi α = AC + BD = 2 2.
Per studiare il valore massimo, consideriamo un sistema di assi cartesiani di origine in A, con l’asse x
lungo il lato AB e l’asse y lungo il lato AD; i punti B e D avranno coordinate rispettivamente (1, 0) e
(0, 1). Cerchiamo il massimo dell’espressione P A + P B per un punto P a distanza h dal lato AB . Dato che
punti simmetrici rispetto alla retta x = 1/2 forniscono lo stesso valore per l’espressione P A + P B , poniamo
P = (1/2 + x, h), con −1/2 ≤ x ≤ 1/2.
130
I problemi della Scuola Normale
D
Anno 1996
131
C
D
P
R
S
1 /2
A
C
P
R
B
A
S
B E
Figura 2
Studiamo quindi
f (x) = P A + P B =
s
2
1
+x
2
+ h2 +
s
Figura 3
2
1
−x
2
+ h2
La disuguaglianza
f 0 (x) = q
1
2
1
2
equivale a
h2
"
1
2
+x
−q
2
+ x + h2
1
2
−x
≥0
2
− x + h2
I punti R e S sono esterni all’ellisse di fuochi C e D passante per P ed è quindi chiaro che fra tutti i punti
di E il massimo di P C + P D si ha quando P coincide con R o con S .
Prendiamo in considerazione, per fissare le idee, il punto S interno a BC e vediamo come rendere minima
la quantità SA + SD (SB + SC è sempre 1 per ogni posizione di S ).
D
C
2 2 #
1
1
= 2 h2 x ≥ 0 ;
+x −
−x
2
β =2+
S
2
quindi il punto medio di RS (fig. 2) è punto di minimo.
Il massimo di f (x) = P A + P B ad h fissato è raggiunto in S e in R; qualunque delle due scelte rende
massima anche P C + P D e dunque tutta la somma. Se P appartiene, ad esempio, al lato BC , si ha
P A + P B + P C + P D = P A + P D + BC e l’espressione P A + P D√è minima√nel punto medio di BC ed
è massima negli estremi, ad esempio in C , dove vale BC + DC + 2 = 2 + 2, per quanto visto prima.
Dunque
√
2.
A
Figura 4
Allo scopo consideriamo l’ellisse di fuochi A e D passante per S ; i punti B e C sono esterni a tale ellisse
qualunque sia S sul segmento BC , pertanto tali punti realizzano il massimo della somma
SA + SB + SC + SD
SECONDA SOLUZIONE
Sia P un punto del quadrato ABCD; mantenendo costante la somma P A + P B troviamo quali posizioni di
P rendono massima P C + P D.
Il luogo dei punti P per i quali P A + P B è una assegnata costante (maggiore di AB ) è una ellisse E di fuochi
A e B ; siano R e S le intersezioni di tale ellisse con i lati AD e BC .
B
√
che vale (ad esempio per S ≡ C ) BC + DC + AC = 2 + 2. Dunque
β =2+
√
2.
132
I problemi della Scuola Normale
1996.2
Il prezzo di mercato P di una certa merce dipende dalla quantità totale Q venduta secondo la
legge P = a − b Q, dove a e b sono due assegnati valori positivi.
Sul mercato operano solo due produttori, in concorrenza fra loro.
A regime, cioè quando nessuno dei due ha interesse a cambiare la quantità di merce da lui venduta, i due
produttori vendono rispettivamente le quantità X e Y di merce. Supponendo che la produzione avvenga a
costo zero, determinare X e Y .
RISOLUZIONE
Si intende che in questo “modello” il mercato è in grado di assorbire qualunque quantità di
prodotto.
Osserviamo intanto che nessuno dei due produttori ha interesse ad immettere sul mercato una quantità di
merce maggiore o uguale a a/b, perché ciò azzera il prezzo; quindi X < a/b, Y < a/b.
Il ricavo del primo produttore è dato da
(a − b (X + Y )) X = (a − b Y ) X − b X 2 .
Se Y è fissato, con 0 < Y < a/b, tale ricavo ha per grafico una parabola che passa per 0 quando X = 0 e
quando X = (a − b Y )/b, e ha massimo per X = (a − b Y )/(2 b); per valori di X inferiori il primo produttore
ha interesse ad aumentare la produzione, per valori superiori a diminuirla.
La condizione simmetrica, Y = (a − bX )/(2a) è la condizione di equilibrio per il secondo produttore, e
devono valere entrambe:
2bX + bY = a
bX + 2bY = a.
Sottraendo si ha b (X − Y ) = 0, e quindi X = Y , da cui 3 b X = a; i valori di X e Y all’equilibrio sono
dunque
X=Y =
a
3b
a−b·
a−b·2
a 3b
RISOLUZIONE
Sapendo che in un poliedro ogni spigolo appartiene esattamente a 2 facce, se supponiamo
che ogni faccia abbia almeno 6 lati, arriviamo alla stima
·2
a
3b
=
2 a2
.
9 b
F ·6
1
= 3 F , da cui F ≤ S .
2
3
Stimiamo ora il numero di vertici in base a quello degli spigoli: da ogni vertice escono almeno 3 spigoli,
ma ogni vertice è contato due volte, una volta come primo estremo, l’altra come secondo estremo di uno
stesso spigolo e di conseguenza
S≥
3
2
V , da cui V ≤ S .
2
3
Sostituendo nella uguaglianza di Eulero F − S + V = 2 si ottiene
S≥
1
2
S − S + S = 0,
3
3
il che è chiaramente un assurdo; con ciò è dimostrato il punto (a).
Supponiamo ora che k facce abbiano meno di 6 lati; dobbiamo determinare il valore minimo di k . La prima
stima di F si modifica nel modo seguente; per F − k facce sappiamo che i lati sono almeno 6, mentre le
rimanenti k facce hanno ognuna almeno 3 lati. Si ha quindi
2=F −S+V ≤
S≥
a a
1 a2
=
·
2b 2b
4 b
ed è maggiore della somma ricavata dai due produttori in concorrenza, data da
133
1996.3*
Sia P un poliedro e siano F il numero delle facce, S il numero degli spigoli e V il numero
dei vertici di P. Sapendo che per il poliedro P, per la nota formula di Eulero, vale la relazione
F − S + V = 2,
(a) provare che P ha qualche faccia con meno di 6 lati;
(b) detto k il numero delle facce con meno di 6 lati, determinare il minimo valore possibile per k .
.
Può essere interessante confrontare i risultati dello stesso modello in condizione di monopolio.
Il ricavo dell’unico produttore, fornito dall’espressione (a − b X ) X , è massimo per X = a/(2 b); tale volume
di produzione è inferiore a quello di equilibrio per la situazione con due produttori, dato da 2 · a/(3 b).
Il massimo ricavo possibile per il monopolista è
Anno 1996
3k
(F − k ) · 6 + k · 3
= 3F −
,
2
2
da cui
1
1
S + k,
3
2
e, sostituendo come prima,
F ≤
1
1
2
1
S + k −S + S = k,
3
2
3
2
il che porta a k ≥ 4. Che k possa assumere il valore 4 si vede considerando il tetraedro (che contiene 0
facce con almeno 6 lati); d’altra parte è evidente che se F − k fosse 1 o 2 o più grande, il numero di facce
con meno di 6 lati sarebbe certamente maggiore di 4, e dunque 4 è la soluzione del problema di minimo.
2=F −S+V ≤
134
I problemi della Scuola Normale
1996.4
Sia f (t) una funzione iniettiva definita sui numeri reali positivi. Dati x > 0 e y > 0, chiamiamo
f -Media di x e y l’unico numero z tale che
f (z ) =
f (x) + f (y )
.
2
√
Mostrare che la media geometrica x y e quella armonica 2 x y/(x + y ) sono delle f -Medie.
Fra le funzioni convesse f , individuare quelle per le quali la f -Media risulta minore o uguale della media
aritmetica.
RISOLUZIONE
M (x, y ) =
Anno 1996
135
1996.5
Dato un triangolo nel piano euclideo si indichi con I il centro della circonferenza in esso
inscritta e con Γ la circonferenza passante per I e per due qualunque dei vertici del triangolo.
Provare che il centro di Γ si trova sulla circonferenza circoscritta al triangolo.
Sia, ad esempio, Γ la circonferenza passante per I , B e C . Allora necessariamente il centro
di Γ deve giacere sull’asse di BC ; si dovrà provare che il centro di Γ è il punto H , intersezione dell’asse del
_
_
segmento BC con la circonferenza circoscritta. Si noti che, essendo BH = HC , la bisettrice dell’angolo in
A passa per H (oltre che per I ).
RISOLUZIONE
Per il primo punto, notiamo che:
A
√
x y = Mf (x, y ) prendendo come f (t) la funzione log t: infatti
√
z = xy
se e solo se
√
I
log z = log x y = (log x + log y )/2 ;
O
N (x, y ) = 2 x y/(x + y ) = Mf (x, y ) per f (t) = 1/t; infatti
B
z = 2 x y/(x + y ) = 2/(1/x + 1/y ) ,
C
che equivale a 1/z = (1/x + 1/y )/2 .
Se f è convessa ed è iniettiva, f è o strettamente crescente o strettamente decrescente.
Se f è strettamente crescente si ha: Mf (x, y ) ≤ (x + y )/2 se e solo se f (Mf (x, y )) ≤ f (x + y )/2, equivalente
a 1/2 f (x) + 1/2 f (y ) ≤ f (x + y )/2; d’altra parte la f è convessa, e quindi f ((1 − t)x + ty ) ≤ (1 − t)f (x) + tf (y )
per ogni t ∈ [0, 1] e in particolare f (1/2 x + 1/2 y ) ≤ 1/2 f (x) + 1/2 f (y ); quindi il grafico di f sta al di sopra
di quello della retta secante per x e y , e coincide con questa nel punto medio e quindi è una funzione
affine.
Se invece f è strettamente decrescente si ha: Mf (x, y ) ≤ (x + y )/2 se e solo se f (Mf (x, y )) ≥ f (x + y )/2,
equivalente a 1/2 f (x) + 1/2 f (y ) ≥ f (x + y )/2 che segue direttamente dalla convessità di f .
Concludendo, le funzioni convesse per cui risulta Mf (x, y ) ≤ (x + y )/2 per ogni x, y > 0 sono o convesse
strettamente decrescenti, oppure sono funzioni affini.
H
Figura 1
Dimostriamo che HB = HI .
Il centro I della circonferenza inscritta è punto di intersezione delle bisettrici; se indichiamo con α, β , γ
rispettivamente gli angoli del triangolo in A, B , C si avrà che per il teorema dell’angolo esterno
b =
B IH
α
2
+
β
2
.
b = H AC
b perché insistono sullo stesso arco e pertanto
D’altra parte H BC
b .
b = H AC
b + C BI
b = α + β = B IH
H BI
2 2
Risulta cosı̀ provato che il triangolo IBH è isoscele e quindi HB = HI . Allo stesso modo si prova che
HC = HI e ciò conclude la dimostrazione.
136
I problemi della Scuola Normale
1996.6
A partire da un cerchio C1 tracciare successivamente: un triangolo equilatero P1 inscritto in
C1 , il cerchio C2 inscritto in P1 , un quadrato P2 inscritto in C2 , il cerchio C3 inscritto in P2 , un pentagono
regolare P3 inscritto in C3 , e cosı̀ via, ottenendo in tal modo una successione
C1 ⊃ P1 ⊃ C2 ⊃ P2 ⊃ . . . Cn ⊃ Pn ⊃ Cn+1 ⊃ Pn+1 . . .
137
Si ha quindi
log
Rm
≥ −2 π 2
R1
1
1
1
1
+
+
+ ...+
.
32 42 52
(m + 1)2
Usando il suggerimento con k = 2 otteniamo
di cerchi e poligoni regolari concentrici, dove Pn ha n + 2 lati.
Mostrare che l’intersezione di tutti i cerchi Cn è un cerchio di raggio positivo. Il candidato può ricorrere
alla diseguaglianza, valida per ogni intero k ≥ 1:
1
1
1
1
+
+
+... < .
(k + 1)2 (k + 2)2 (k + 3)2
k
log
Rm
1
≥ −2 π 2 · = −π 2 .
R1
2
Possiamo concludere osservando che, qualunque sia m, si ha
2
Rm ≥ R1 e−π ;
RISOLUZIONE
Indichiamo con Rn il raggio del cerchio Cn , e calcoliamo il rapporto Rn+1 /Rn ; Rn+1 è
b = 2 π/(n + 2).
l’altezza di un triangolo isoscele AOB di lato Rn e angolo AOB
O
Rn
P1
C2
Rn+1
C1
A
B
Figura 1
Si ha quindi che
Rn+1
π
= cos
Rn
n+2
e moltiplicando queste diseguaglianze per n da 1 a m − 1 otteniamo:
π
π
π
π
Rm
= cos · cos · cos · . . . · cos
R1
3
4
5
m+1
Per provare la tesi è necessario e sufficiente dimostrare che i rapporti Rm /R1 non possono avvicinarsi
indefinitamente a 0 al crescere di m, o, il che è lo stesso, che i logaritmi di tali rapporti debbono essere
limitati dal basso. Passando ai logaritmi a primo e secondo membro
log
Anno 1996
π
π
π
π
Rm
= log cos + log cos + log cos + . . . + log cos
R1
3
4
5
m+1
D’altra parte, per ogni n ≥ 1, posto per comodità di notazione αn = π/(n + 2), si ha che:
1
1
1
log cos αn = − log
= − log(1 + tg2 αn ) .
2
cos2 αn
2
Ricordando che se t è maggiore di −1 vale la diseguaglianza log(1+ t) ≤ t, che per ogni x si ha | sin(x)| ≤ |x|
e che per ogni n > 1 il valore di cos αn è maggiore di cos α1 = cos(π/3) = 1/2, abbiamo:
π 2
1
1 sin2 αn
.
log cos αn ≥ − tg2 αn = −
r ≥ −2
2
2 cos2 αn
n+2
quindi tutti i Cn contengono uno stesso cerchio e dunque la loro intersezione è un cerchio centrato nell’origine
2
di raggio maggiore o uguale a R1 e−π .
138
I problemi della Scuola Normale
1997.1
Un cubo è appoggiato su un piano. Un bambino lo muove n volte, facendolo rotolare (senza
strisciare) ogni volta su uno dei lati della faccia su cui è appoggiato. Si suppone che la prima mossa sia
casuale e che, ad ogni mossa successiva, il bambino scelga casualmente di far rotolare il cubo su uno dei
due lati contigui al lato scelto in precedenza (vedi Figura: dopo aver rotolato sul lato a, il cubo rotolerà
sul lato b oppure sul lato c).
Anno 1997
1
3k
139
1−
1
4k
a
b
a) Dimostrare che se il cubo è tornato nella posizione iniziale (non necessariamente appoggiato sulla
stessa faccia) allora n è divisibile per 4.
b) Calcolare la probabilità p(n) che il cubo sia tornato nella posizione iniziale.
c) Dimostrare che
5
1
1
1
−
≤ p(4 k ) ≤
−
,
16 k 16 k 2
3 k 12 k 2
k ≥ 1.
RISOLUZIONE
a) Ad ogni mossa il cubo si sposta di un segmento pari al suo lato in direzione Nord, Sud, Ovest o Est.
Affinché dopo n mosse il cubo sia di nuovo nella posizione iniziale, il numero h degli spostamenti verso
Nord deve uguagliare quello degli spostamenti verso Sud; analogamente, se vi sono k spostamenti verso
Est, altrettanti devono esservi verso Ovest.
D’altra parte, per ipotesi, il bambino alterna una mossa Est–Ovest con una mossa Nord–Sud e dunque si ha
che k = h per cui, in definitiva, n = 4 h.
b) Per il punto precedente, se n non è multiplo di 4, nessuna sequenza di n mosse può lasciare il cubo
nella posizione iniziale, e dunque in tal caso p(n) = 0.
Se n = 4 h, le sequenze di mosse possibili si ottengono scegliendo in ogni modo possibile le 2 h mosse
Nord–Sud e le 2 h mosse Est–Ovest; tenendo conto del fatto che la prima mossa può essere arbitrariamente
in direzione Nord–Sud o Est–Ovest, il che porta ad un ulteriore fattore 2, il numero totale di percorsi
possibili è dato da 2 · 22k · 22k .
Le sequenze di mosse favorevoli sono quelle per cui su 2 h mosse Nord–Sud esattamente h sono mosse
verso Nord, e su 2 h mosse Est–Ovest ve ne sono h verso Est. Ognuna delle due scelte, fra loro indipendenti,
2
può essere effettuata in 2hh modo distinti; vi sono quindi 2 2hh casi favorevoli, e si ha quindi
p(4 k ) =
1
22k
2k
k
2
2k + 1
2 (k + 1)
2
.
Per la disuguaglianza a destra si ha per induzione:
p(4 (k + 1)) ≤
1
3k
Si verifica facilmente che
1
3 (k + 1)
1−
1
.
4 (k + 1)
5
16 k
1−
1
5k
2k + 1
2 (k + 1)
2
.
Ma per k ≥ 1 è facile verificare che vale la disuguaglianza
5
16 k
1−
1
5k
2k + 1
2 (k + 1)
2
≥
5
16 (k + 1)
1−
1
5 (k + 1)
il che conclude la dimostrazione.
1997.2
Si dice che un punto P esterno ad una circonferenza C “vede” la circonferenza sotto un angolo
α se l’angolo (contenente C ) compreso fra le tangenti a C condotte da P è uguale a α.
a) Data una circonferenza C e un angolo A di ampiezza α > 0, costruire il luogo dei punti del piano che
vedono C sotto l’angolo α.
b) Date due circonferenze C e C 0 esterne l’una all’altra, di centri O, O0 e raggi r, r0 rispettivamente,
costruire il luogo L dei punti del piano che vedono le due circonferenze sotto lo stesso angolo.
c) Dire (in termini dei dati) in che intervallo varia l’angolo di visuale al variare di P in L e quali sono i
punti in L dove tale angolo è minimo e massimo.
RISOLUZIONE
b 0 di ampiezza pari a quella
a) Preso un diametro M N della circonferenza C , costruiamo l’angolo M OM
di A e tracciamo le tangenti a C per M 0 e per N . Detta P l’intersezione di dette tangenti, è evidente che,
per differenza di angoli uguali, l’angolo in P ha ampiezza α; il luogo richiesto è la circonferenza di centro
O passante per P .
P
M0
M
α
N
O
C
Figura 1
.
c) Per k = 1 le diseguaglianze sono verificate poiché tutti i membri valgono 1/4. Useremo un ragionamento per induzione su k :
p(4 (k + 1)) = p(4 k )
≤
Per la disuguaglianza a sinistra si ha per induzione:
p(4 (k + 1)) ≥
c
2k + 1
2 (k + 1)
2
1−
1
4k
2k + 1
2 (k + 1)
2
.
b) Sia P un punto esterno alle circonferenze C e C 0 . Tracciamo, come in Figura 2, le tangenti da P alle
due circonferenze e i segmenti P O e P O0 . Il punto P appartiene a L se e solo se OPb T = O0 Pb T 0 , e in tal
caso i triangoli OP T e O0 P T 0 sono simili, e quindi OP : O0 P = OT : O0 T 0 . Dunque il luogo L è dato dai
punti P per cui OP : O0 P = r/r0 .
Se r = r0 il luogo L è l’asse del segmento OO0 ; se r e r0 sono diversi, il luogo è dato dalla circonferenza
di Apollonio relativa al triangolo OP O0 .
140
I problemi della Scuola Normale
T0
P
T
Anno 1997
141
1997.3
Si determini, al variare dei parametri α e β interi pari e positivi, il numero di soluzioni reali
dell’equazione
r0
(α + β ) x sin(π x) = x2 + α β .
r
O
O0
RISOLUZIONE
Poiché α + β è maggiore di 0 possiamo dividere per questo numero e scrivere l’equazione
nella forma
C
C0
x sin(π x) =
Figura 2
x2 + α β
α+β
Il numero delle soluzioni è uguale al numero dei punti comuni ai grafici delle funzioni
Tale luogo contiene chiaramente i punti P1 e P2 da cui si possono mandare tangenti comuni alle due
circonferenze, e questo consente di costruire L tracciando la circonferenza avente centro in M , punto medio
di P1 P2 e raggio M P1 .
y = x sin(π x) ,
y=
x2 + α β
.
α+β
Poiché le due funzioni sono pari basta studiarle per x ≥ 0. Si noti che la prima non dipende dai parametri
α e β e il suo grafico è il seguente:
P
y
P1
C
O
P2
O0
C0
L
1
2
3
4
x
Figura 3
c) La distanza P O0 è massima quando P coincide con P1 , e decresce fino ad assumere il suo valore minimo
per P = P2 ; per quanto osservato nel primo punto, quindi, l’angolo di visuale assume il suo massimo per
P = P2 e il minimo per P = P1 . Nel caso che le due circonferenze abbiano uguale raggio, il luogo è dato
dall’asse del segmento OO0 . Il massimo dell’angolo visuale si ha quando P coincide con il punto medio di
OO0 , ma nessun punto dell’asse è punto di minimo per l’angolo di visuale.
Figura 1
La seconda invece rappresenta una famiglia di parabole con l’asse coincidente con l’asse y e con la concavità
rivolta verso l’alto; l’ordinata del vertice è data da y0 = α β/(α + β ).
La generica parabola di tale famiglia incontra la bisettrice del primo quadrante nei punti la cui ascissa
verifica l’equazione
x=
x2 + α β
α+β
che si può scrivere nella forma x2 − (α + β ) x + α β = 0; le soluzioni sono quindi date da
x1 = α,
x2 = β .
Si noti che tutte le parabole incontrano la bisettrice del primo quadrante in punti con ascissa intera positiva e
che in tali punti la prima funzione vale sempre 0. Sugli intervalli (2 n − 1, 2 n) la prima funzione è negativa e
quindi sicuramente il suo grafico non incontra la parabola; gli incontri possono avvenire solo sugli intervalli
del tipo (2 n, 2 n + 1) contenuti tra α e β , e su ciascuno di questi si sono esattamente due punti di incontro.
Chiaramente non si hanno intersezioni se α = β . Supponiamo per fissare le idee che sia α < β ; l’intervallo
(α , β ) conterrà (β − α)/2 intervalli del tipo (2 n, 2 n + 1) e dunque l’equazione avrà β − α soluzioni con
ascissa positiva e 2 (β − α) soluzioni su tutta la retta.
142
I problemi della Scuola Normale
Anno 1997
143
Necessariamente α e β hanno segno discorde. Supponiamo che sia α ≥ 0 e β ≤ 0 (nel caso contrario si
ragiona in modo del tutto analogo). Scriviamo
y
mα = 1 − 2β .
2
4
6
x
Vi è dunque un cammino di lunghezza s = −2 β ed un altro di lunghezza s + 1 = 1 − 2 β = m α che partono
da X e tornano in X . Fissiamo N0 = s2 e dimostriamo che per ogni n ≥ N0 esiste un cammino di lunghezza
n che inizia e termina in X ; in effetti, dividendo n per s si ottiene n = a s + b, dove a ≥ s e 0 ≤ b < s, e
quindi
n = a s + b = (a − b) s + b (s + 1) .
α=2
β=6
Questo dimostra l’esistenza del cammino di lunghezza n. Sappiamo per ipotesi che preso un qualunque
altro punto Y ∈ Ω, esiste un cammino cY da Y ad X di lunghezza lY . Siano ora dati due punti A, B ∈ Ω.
Percorrendo il cammino cA , poi un cammino di lunghezza n che parte e termina in X e poi il cammino cB
in senso inverso si ottiene un cammino che parte in A e termina in B di lunghezza lA + lB + n.
Figura 2
cA
1997.4*
Sia dato un insieme finito Ω di punti distinti del piano tra loro collegati da un certo numero di
percorsi elementari congiungenti coppie di vertici distinti come esemplificato nel disegno seguente:
A
cB
X
B
Figura 1
Se indichiamo con L la massima lunghezza dei cammini lA al variare del punto A in Ω e definiamo
N = 2 L + N0
abbiamo chiaramente che una qualunque coppia di punti è collegabile con un cammino di quella lunghezza.
Dati due punti A e B di Ω un cammino che parte da A e termina in B è una successione di vertici
v0 , v1 , v2 , . . . , vn di punti di Ω tali che v0 = A, vn = B, e tale che vi e vi+1 sono congiunti da un percorso
elementare; in questo caso si dice che n è la lunghezza del cammino.
I punti, i percorsi elementari e i cammini soddisfano le seguenti proprietà:
i) i percorsi elementari non si incontrano fuori dai punti di Ω;
ii) dati due qualsiasi punti A, B ∈ Ω, esiste almeno un cammino che parte da A e termina in B ;
iii) c’è un particolare punto X ∈ Ω per il quale esiste un cammino che parte e termina in X avente
lunghezza dispari.
Si dimostri allora che esiste N intero positivo tale che, scelti due qualsiasi punti A e B di Ω, esiste un
cammino di lunghezza N che parte da A e termina in B .
Osserviamo per prima cosa che se Ω è ridotto al solo punto X la tesi è vera, con N = 1.
Notiamo che se vi sono due cammini di lunghezza l1 e l2 che iniziano e terminano in X , ce n’è anche uno di
lunghezza l1 + l2 che si ottiene percorrendo il primo cammino e di seguito il secondo. In particolare esiste
un cammino di lunghezza 2 che inizia in X , passa per un altro punto e torna in X . Dunque per X passa
almeno un cammino chiuso di lunghezza 2 e, per ipotesi, un cammino di lunghezza dispari m; per un noto
risultato sulle equazioni diofantee,dal fatto che m e 2 sono primi fra loro, segue che esistono due
interi α e β tali che
RISOLUZIONE
mα + 2β = 1.
144
I problemi della Scuola Normale
1997.5
In ognuna delle tre figure seguenti è disegnato un poligono P . Si immagini che, in ognuno dei
tre casi, il poligono P sia costituito di materiale elastico e flessibile. Si chiede di disegnare o descrivere
sinteticamente la figura S che, in ognuno dei tre casi, si ottiene facendo combaciare le frecce dei lati che
hanno la stessa lettera, e di illustrare come questi lati appaiono in S .
c
a
a
d
b
P
c
a
145
Il primo di tali poligoni si può ottenere da quello di partenza contraendolo o dilatandolo (in maniera non
uniforme) nella direzione orizzontale; il secondo si ottiene dal primo trascinando verso l’alto il secondo
lato verticale.
Si vede facilmente che l’ultima figura può essere arrotolata attorno ad un cilindro in modo che i segmenti a
si sovrappongano, rispettando la direzione delle frecce. Il cilindro cosı̀ ottenuto (fig. 4)
b
b
P
b
Anno 1997
d
c
P
a
c
b
a
b
c
b
c
b
a
a
Figura 4
Iniziamo con il primo poligono. Possiamo arrotolarlo a forma di cilindro, facendo coincidere
i segmenti marcati con a, e curvarlo poi orizzontalmente portando a combaciare le basi del cilindro, che
sono le trasformate dei segmenti b. È facile vedere che la direzione delle frecce viene rispettata.
RISOLUZIONE
può essere ora curvato in modo da portare a coincidere gli archi b e c, ottenendo ancora un toro (fig. 5).
a
b
b
a
c
b
Figura 1
La figura che si ottiene è quella di una ciambella, un toro in linguaggio matematico.
Figura 5
Per l’ottagono centrale iniziamo osservando che il problema presenta una simmetria. Come si vede in
Figura 6, introducendo un nuovo segmento z possiamo dividere l’ottagono in due poligoni tali che su
ognuno gli incollamenti avvengono indipendentemente dall’altro. La figura ottenuta può essere deformata
in maniera equivalente come mostrato in Figura 7.
a
b
c
d
d
b
Figura 2
z
c
Procedendo in ordine di numero di lati, passiamo al terzo poligono. Dato che il materiale di cui è costituito
è elastico, il risultato a cui si arriva effettuando gli incollamenti partendo da questo poligono è lo stesso di
quello che si ottiene partendo dai poligoni in Figura 3.
z
d
a
b
a
a
c
b
z
z
c
b
d
a
Figure 6, 7
a
a
b
c
c
b
b
b
a
Figura 3
c
c
a
Facendo ora combaciare i lati d e i lati a otteniamo due cilindri, che deformiamo ulteriormente ritraendo gli
archi z fino a far coincidere gli estremi degli archi c e b(fig. 8).
146
I problemi della Scuola Normale
d z
z a
c
c
b
b
Anno 1997
147
1997.6* √ Si determinino tutti gli interi positivi n che sono divisibili per tutti gli interi positivi minori o
uguali a n.
√
(Suggerimento: considerare il minimo comune multiplo dei numeri minori o uguali a n).
RISOLUZIONE
Sia m il minimo comune multiplo di tutti gli interi minori o uguali a
√
n e sia
m = pν1 1 pν2 2 . . . pνss
d
c
z
c
z
b
a
b
la sua scomposizione in fattori primi.
√
Se n è divisibile per tutti gli interi non superiori a n si ha necessariamente che
√
n < pi νi +1
Figura 8
per i = 1, 2, . . . , s
e quindi
Pieghiamo ora ad anello i due cilindri in modo da incollare gli archi c e b; si ottengono due ciambelle a cui
sono state tolte due calottine che hanno per bordo gli archi z . Incollando i due tori lungo tali archi si ottiene
una ciambella con due buchi (fig. 9).
√ s
n < p1 ν1 +1 · · · ps νs +1 = m · (p1 · · · ps ) ≤ m2 ;
d’altra parte m è minimo comune multiplo di divisori di n e dunque è ancora un divisore di n; in particolare
si ha m ≤ n.
Si ha quindi
√
( n ) s < m2 ≤ n 2
b
a
Figura 9
√
e dunque s < 4; ne segue che p4 = 7 > n e quindi n < 49.
A questo punto una semplice analisi dei vari casi mostra che gli unici valori possibili per n sono:
1, 2, 3, 4, 6, 8, 12, 24.
c
d
SECONDA SOLUZIONE
Da una analisi immediata vediamo che 1, 2, 3 godono della proprietà richiesta; possiamo quindi studiare il
caso n ≥ 4.
Sia k il più grande intero per cui k 2 ≤ n; avremo quindi
k 2 ≤ n < (k + 1)2 .
Tanto k quanto k − 1 sono, per ipotesi, divisori di n e sono primi fra loro, per cui il loro prodotto è un
divisore di n, è strettamente minore di n perché k (k − 1) < k 2 ≤ n e dunque k (k − 1) ≤ n/2.
Possiamo dedurne che
(k + 1)2
n
k (k − 1) ≤ ≤
2
2
2
da cui 2 k (k − 1) ≤ (k + 1)2 , o ancora
√ k − 4 k − 1 ≤ 0.
Ne segue che deve essere k ≤ 2 + 5 < 5, e quindi n < 36.
Come nella soluzione precedente, l’ultimo passo consiste nel verificare che i valori ammissibili sono dati
da 1, 2, 3, 4, 6, 8, 12, 24.
148
I problemi della Scuola Normale
Glossario
In questa appendice si richiamano succintamente alcune nozioni e proprietà utili nella soluzione dei
problemi riportati in questo volume o di altri problemi simili.
Come ricordato nella introduzione, i richiami sono schematici ed essenziali; si è preferito rinunciare
ad una maggiore generalità e ad un maggior rigore a favore di una presentazione il più possibile agile e
semplice.
BARICENTRO,
di un triangolo. È il punto in cui concorrono le mediane del triangolo. Il baricentro
divide ogni mediana in due parti, di cui quella che ha un estremo nel vertice è doppia dell’altra. Si dice
baricentro di un sistema di punti P1 , P2 , . . . , Pn il punto le cui coordinate sono la media aritmetica delle
coordinate dei punti Pk .
BINOMIO DI NEWTON
Dati due numeri a, b e un intero positivo n si ha
n n−2 2
a
b + ···+
an−1 b +
2
n
a bn−1 + bn ,
n−1
n
( a + b ) n = an +
ove i coefficienti
1
n
sono i coefficienti binomiali (vedi Combinazioni).
k
CIRCONFERENZA DI APOLLONIO
Dati due punti A, B e un numero positivo k , è il luogo dei punti P per
i quali
AP = k BP ;
tale luogo è una circonferenza che ha come diametro il segmento M N , ove M e N sono i due punti della
retta per A e B che hanno distanze da A e da B nel rapporto assegnato k (uno di tali punti è interno ad AB ,
l’altro esterno).
COMBINAZIONI,
di n elementi a k a k (con k ≤ n). Sono i sottoinsiemi (non ordinati) di k elementi
scelti in un insieme di n elementi. Il loro numero è dato dal coefficiente binomiale
n!
n
.
=
k ! (n − k )!
k
150
I problemi della Scuola Normale
CONGRUENZE ARITMETICHE
Si dice che l’intero a è congruo all’intero b modulo m, e si scrive a ≡ b
(mod m), se a e b hanno ugual resto nella divisione per m (cioè a − b è multiplo di m).
Valgono le seguenti proprietà:
se a ≡ b (mod m) e c ≡ d (mod m), allora a + c ≡ b + d (mod m) e a c ≡ b d (mod m).
COORDINATE BARICENTRICHE,
di un punto in un triangolo. Dato un triangolo non degenere di vertici
A ≡ (x1 , y1 ), B ≡ (x2 , y2 ), C ≡ (x3 , y3 ), le coordinate baricentriche di un punto P ≡ (x, y3 ) appartenente al
triangolo sono la terna di numeri, c1 , c2 , c3 , con ci ≥ 0, c1 + c2 + c3 = 1 per cui si ha
x = c1 x1 + c2 x2 + c3 x3
y = c1 y 1 + c2 y 2 + c3 y 3 .
A
Glossario
151
a1 + a2 + · · · + an
,
A=
n
mentre la loro media geometrica è
G=
r
n
a1 a2 · · · an
.
n
Risulta A ≤ G e l’uguaglianza vale solamente quando a1 = a2 = · · · = an .
DISUGUAGLIANZA TRIANGOLARE
1. Ogni lato di un triangolo è minore della somma e maggiore della
differenza degli altri due; tale condizione è necessaria e sufficiente affinché tre segmenti possano essere lati
di un triangolo.
2. Dati due numeri a, b si ha
||a| − |b|| ≤ |a + b| ≤ |a| + |b| .
N
P
M
C
B
L
EQUAZIONE DIOFANTEA
È una equazione polinomiale, a coefficienti interi, di cui si cercano soluzioni
intere. Ad esempio l’equazione diofantea a2 + b2 = c2 ha per soluzioni le terne pitagoriche.
È un importante risultato sulle equazioni diofantee del primo ordine che l’equazione in x, y
mx +ny = p
Condotte le rette per P dai vertici del triangolo fino ad incontrare i lati opposti, è facile vedere che le
coordinate baricentriche di P sono date da
AM
AN
=
AC
AB
BL BN
=
c2 =
BC
BA
CM
CL
c3 =
=
.
CA
CB
c1 =
In maniera del tutto analoga si definiscono le coordinate baricentriche per un punto in un tetraedro.
ammette soluzioni se e solo se il massimo comun divisore dei coefficienti m e n divide il termine noto p e
in particolare ha soluzione per ogni p se m e n sono primi fra loro.
Vari problemi riguardanti le equazioni diofantee hanno trovato soluzione solo in tempi recenti. Nel 1970
Matyasevich ha dimostrato che non esiste un algoritmo in grado di decidere se una data equazione diofantea
ammetta soluzioni, rispondendo cosı̀ al decimo problema di Hilbert.
Nel 1995 Wiles ha dimostrato che se n è un intero maggiore di 2 l’equazione diofantea an + bn = cn non ha
soluzioni (ultimo teorema di Fermat).
FATTORIALE,
di un numero naturale n. È il prodotto dei numeri naturali minori o uguali a n, e si
indica con n!:
La derivata di una funzione f (x) in un punto x0 è il limite (se esiste) per h → 0 del
rapporto incrementale
DERIVATA
f (x0 + h) − f (x0 )
.
h
Tale numero è anche la tangente trigonometrica dell’angolo formato con l’asse x dalla retta tangente al
grafico di f nel punto (x0 , f (x0 )).
n! = n · (n − 1) · (n − 2) · · · 3 · 2 · 1 .
Si pone per convenzione 0! = 1. Il numero di permutazioni di n oggetti è dato da n!, come si verifica
facilmente usando il principio di induzione.
FORMULA DI EULERO
Se un poliedro convesso ha V vertici, S spigoli e F facce, allora
V − S + F = 2.
tra due rette sghembe. Date due rette sghembe, esiste uno e un solo segmento che ha un
estremo su ciascuna di esse ed è perpendicolare ad entrambe; esso è il segmento più breve fra quelli che
congiungono un punto dell’una con un punto dell’altra; la sua lunghezza è la distanza fra le due rette.
DISTANZA,
DISUGUAGLIANZA FRA LE MEDIE
Dati n numeri a1 , a2 , . . . , an , la loro media aritmetica è
Ad esempio per il cubo si ha V = 8, S = 12, F = 6, per il tetraedro V = 4, S = 6, F = 4.
Una versione della formula di Eulero riguarda i grafi planari con archi che non si intersecano: in un tale
grafo, se n è il numero dei nodi, a quello degli archi, allora il grafo divide il piano in r regioni e si ha
r = a − n + 2.
152
I problemi della Scuola Normale
2t
sin α =
,
1 + t2
α
ove t = tg .
2
FUNZIONE CONTINUA
Informalmente si dice continua una funzione f (x) tale che il valore da essa
assunto cambia di poco per piccoli cambiamenti della variabile x. Più precisamente, una funzione f (x) si
dice continua in un punto a se per ogni numero ε > 0 si può trovare un numero δ tale che
se
|x − a| < δ
allora
153
INVOLUCRO CONVESSO
o inviluppo convesso di un insieme A è il più piccolo insieme convesso che
contiene A; esso è dato dall’intersezione di tutti gli insiemi convessi contenenti A.
FORMULE PARAMETRICHE
1 − t2
,
cos α =
1 + t2
Glossario
OMOTETIA
Dato un punto O e un numero reale k , l’omotetia di polo (o centro) O e rapporto k è una
trasformazione in cui ad ogni punto A corrisponde un punto A0 sulla semiretta per A uscente da O se k è
positivo, della semiretta opposta se k è negativo, in modo che
OA0 = k OA .
Ogni omotetia manda rette in rette, conserva gli angoli e i rapporti fra i segmenti ed è dunque una similitudine.
|f (x) − f (a)| < ε .
Una funzione è continua in un intervallo se è continua in ogni punto di tale intervallo.
FUNZIONE CONVESSA
Una funzione f (x) definita su un intervallo I della retta si dice convessa se su
ogni intervallo [x, x0 ] contenuto in I il grafico della funzione giace al di sotto della retta secante nei punti
0
0
(x, f (x)) e (x , f (x )) o, in simboli,
f ((1 − t) x + t x0 ) ≤ (1 − t) f (x) + t f (x0 )
per ogni t ∈ [0, 1].
Una funzione convessa su (a, b) è continua in ogni punto dell’intervallo; una funzione f (x) derivabile due
volte in (a, b) è convessa se e soltanto se f 00 (x) ≥ 0 in ogni punto x di (a, b).
FUNZIONE AFFINE
È una funzione espressa da un polinomio di primo grado. Una funzione di una
variabile è affine se e solo se ha per grafico una retta; in due variabili il grafico è un piano. Una trasformazione
in cui le coordinate del punto di arrivo sono funzioni affini delle coordinate del punto di partenza è detta
affine.
GRAFO
Un grafo è un insieme finito di oggetti, detti nodi del grafo, e di archi che connettono coppie
di nodi. Un grafo G si dice completo se ogni coppia di nodi è collegata da un arco, si dice connesso se presi
comunque due nodi n, n0 è possibile passare da n a n0 seguendo un cammino formato da archi di G. Un
cammino chiuso, che parte ed arriva in uno stesso nodo, visitando gli altri nodi una sola volta, è detto ciclo.
Un grafo connesso privo di cicli è detto albero.
Se G è un grafo, il suo complementare G0 è definito come il grafo che ha gli stessi nodi di G ed ha l’insieme
degli archi complementare a quello degli archi di G: due nodi sono connessi in G0 se e solo se non lo
sono nel grafo G. Un primo risultato sui grafi è il seguente: se un grafo non è connesso allora il suo
complementare lo è (ma il viceversa è chiaramente falso in generale).
PRINCIPIO DEI CASSETTI,
(o dei nidi di piccione): se n oggetti vengono posti in (n − 1) cassetti almeno
un cassetto contiene più di un oggetto. Se infatti in ogni cassetto vi fosse al più un oggetto, il numero degli
oggetti non potrebbe superare quello dei cassetti.
PRINCIPIO DI IDENTITÀ DEI POLINOMI
Se due polinomi P (x) e Q(x) di grado minore o uguale a n
assumono valori uguali in almeno n + 1 valori distinti di x, essi hanno uguali i coefficienti dei termini simili
e quindi coincidono; in particolare si ha che P (x) = Q(x) per ogni x.
PRINCIPIO DI INDUZIONE
Ad ogni intero positivo n sia associata una proprietà P (n), che può essere
vera o falsa. Se
(a) P (1) è vera;
(b) per ogni k , P (k ) implica P (k + 1),
allora P (n) è vera per ogni intero positivo n.
Il principio di induzione può essere assunto come assioma nella presentazione dei numeri naturali, o, a
volte, è derivato assumendo come assioma altre proprietà equivalenti, come ad esempio che ogni insieme
di numeri naturali ammette un primo elemento.
Ecco alcuni esempi tipici di risultati che si possono dimostrare facilmente con il principio di induzione:
- per ogni intero positivo n
n
X
k2 =
k=1
1
n (n + 1) (2 n + 1) ;
6
- per ogni intero positivo n, (cos ϑ + i sin ϑ)n = cos n ϑ + i sin n ϑ;
- per ogni intero positivo n, n3 − n è divisibile per 6.
Ovviamente il principio di induzione può essere usato per dimostrare che la proprietà P (n) è verificata da
ogni numero naturale maggiore o uguale a un fissato intero n0 se la (a) è sostituita da
(a’) la proprietà P (n0 ) è vera.
INSIEME CONVESSO
Un insieme di punti della retta, del piano o dello spazio si dice convesso se, dati
due qualunque suoi punti, il segmento che li unisce è interamente contenuto nell’insieme.
PROBABILITÀ
Se un esperimento ha n possibili risultati e un certo evento E si realizza in k di essi, si
dice che la probabilità dell’evento E è
INSIEME NUMERABILE
Un insieme A è numerabile se si può mettere in corrispondenza biunivoca
con l’insieme dei numeri naturali. L’insieme dei numeri razionali è numerabile, l’insieme dei polinomi a
coefficienti interi è numerabile mentre non è numerabile l’insieme dei numeri reali.
P (E ) =
k
.
n
In questa definizione si presuppone di sapere che, degli n risultati dell’esperimento, nessuno sia privilegiato
rispetto agli altri, cioè che essi siano equiprobabili; la definizione è quindi tautologica. In molti casi
154
I problemi della Scuola Normale
concreti, tuttavia, basandosi sul comportamento statistico dei risultati è ragionevole assumere l’ipotesi che
i singoli risultati siano equiprobabili. Cosı̀ ad esempio lanciando un dado non truccato possiamo dire che
l’evento “esce 1” ha probabilità 1/6, l’evento “esce un numero pari” ha probabilità 3/6, ecc.
Glossario
RAPPRESENTAZIONE NUMERICA IN BASE b,
dove b è un intero positivo maggiore di 1. Ogni intero
positivo a si può scrivere (dividendolo ripetutamente per la base e prendendo i resti di tali divisioni) nella
forma
a = a0 + a1 b + a2 b 2 + · · · + an b n ,
di due vettori nel piano o nello spazio. Il prodotto scalare dei vettori u e v,
indicato u · v, è definito come prodotto delle lunghezze dei vettori per il coseno dell’angolo compreso; il
suo valore assoluto coincide con l’area del parallelogramma che ha u e v per lati, e il segno è positivo se
la proiezione di u su v ha lo stesso verso di v, è negativo altrimenti. Se u e v hanno componenti (u1 , u2 ) e
(v1 , v2 ) rispetto ad un sistema di riferimento ortogonale, il loro prodotto scalare è dato da
PRODOTTO SCALARE,
u · v = u1 v1 + u2 + v2 ,
ed una analoga espressione ha il prodotto scalare di due vettori dello spazio.
PROGRESSIONE ARITMETICA
Una successione di numeri a1 , a2 , . . . , an , . . . è detta progressione
aritmetica se la differenza tra due termini consecutivi è una costante r, detta ragione della progressione.
Si può dunque definire per ricorrenza una progressione aritmetica tramite le formule
a1 = a ,
an = an−1 + r
per n > 1 .
L’espressione generale del termine n-esimo è data da
an = a + (n − 1) r .
La somma sN dei primi N termini di una progressione aritmetica può essere calcolata con la formula
sN =
a1 + aN
2
N=
2 a + (N − 1) r
N.
2
PROGRESSIONE GEOMETRICA
È detta progressione geometrica una successione di numeri a1 , a2 , . . . ,
an , . . . per la quale il rapporto tra due termini consecutivi è una costante r; tale costante è detta ragione
della progressione.
Una progressione geometrica si può definire per ricorrenza tramite le formule
a1 = a ,
an = an−1 r
per n > 1 .
L’espressione generale del termine n-esimo è data da
an = a rn−1 .
La somma sN dei primi N termini di una progressione geometrica con ragione r 6= 1 è fornita da
sN = a
rN − 1
.
r−1
Se |r| < 1 si possono anche “sommare” gli infiniti termini della progressione ottenendo
s = a (1 + r + r2 + . . . + rn + . . .) = a
Ad esempio
1+
1
.
1−r
1 1
1
1
+ +... + n + ... =
= 2.
2 4
2
1 − 1/2
155
ove 0 ≤ ai < b per ogni i. Possiamo scrivere più compattamente tale rappresentazione come
a = (an an−1 . . . a1 a0 )b
Se b è 10, tale scrittura coincide con la usuale rappresentazione decimale.
156
I problemi della Scuola Normale
REGOLA DEI SEGNI DI CARTESIO
Glossario
157
Sia
P (x) = a0 xn + a1 xn−1 + · · · + an−1 x + an
b
un polinomio a coefficienti reali. Si scrivano in ordine i segni dei coefficienti non nulli (+ o −). Il numero
delle radici positive del polinomio non supera il numero dei cambiamenti di segno. Ad esempio, il numero
di radici positive del polinomio 8 x9 − 7 x6 − 3 x5 + 2 x3 + 4 non supera 2.
c
a
SIMILITUDINE
Trasformazione che conserva i rapporti tra i segmenti. Similitudini particolari sono le
omotetie, le traslazioni, le rotazioni, le simmetrie e le loro composizioni.
TEOREMA DEL PARALLELOGRAMMA
La somma dei quadrati costruiti sui lati di un parallelogramma è
equivalente alla somma dei quadrati costruiti sulle diagonali. È una facile conseguenza del teorema della
mediana.
1. La bisettrice interna di un triangolo divide il lato opposto in parti
proporzionali ai due lati che la comprendono.
2. Il quadrato di una bisettrice di un triangolo è equivalente al rettangolo dei lati che la comprendono meno
il rettangolo dei segmenti che essa determina sul terzo lato.
TEOREMA DELLA BISETTRICE
a
c
b
a
b
TEOREMA DI RUFFINI
Un polinomio P (x) = a0 xn + a1 xn−1 + · · · + an−1 x + an è divisibile per il
binomio (x − α) se e solo se α è radice di P (x).
Se P (x) ha radici α1 , α2 , . . . , αk rispettivamente di molteplicità h1 ,h2 , . . . , hk , (h1 + h2 + · · · + hk = n) allora
P (x) si può scomporre in fattori lineari
P (x) = a0 (x − α1 )h1 (x − α2 )h2 · · · (x − αk )hk .
TEOREMA DI TALETE
Tre rette parallele staccano su due trasversali coppie di segmenti in proporzione.
Vale anche il viceversa: se i segmenti staccati sono fra loro in proporzione le tre rette sono parallele. In
particolare una omotetia del piano o dello spazio trasforma una retta in una retta ad essa parallela.
TEOREMA DELLA MEDIANA
Il doppio del quadrato di una mediana di un triangolo è equivalente alla
somma dei quadrati dei due lati che la comprendono meno la metà del quadrato del terzo lato.
TEOREMA DELLA SECANTE E DELLA TANGENTE
Condotte da un punto P esterno ad una circonferenza C
una semiretta P A tangente a C in A e una retta che interseca C nei punti B e C si ha P B : P A = P A : P C .
TEOREMA DI ESISTENZA DEI VALORI INTERMEDI
Se una funzione continua f (x) è definita sull’intervallo
[a, b] e t è un numero compreso fra f (a) e f (b), c’è almeno un punto c in [a, b] per cui si ha f (c) = t. In
particolare una funzione continua definita su un intervallo limitato e chiuso assume almeno una volta ogni
valore compreso fra il suo minimo e il suo massimo.
TEOREMA DI PITAGORA
È il più noto teorema di geometria piana: in un triangolo rettangolo il quadrato
costruito sull’ipotenusa è equivalente alla somma dei quadrati costruiti sui cateti. In altri termini, se c è la
misura dell’ipotenusa e a, b le misure dei cateti si ha a2 + b2 = c2 .
Una elegante dimostrazione si può ottenere suddividendo un quadrato di lato a + b in due modi differenti,
come mostra la figura, e uguagliando le aree.
TERNA PITAGORICA
È un insieme di tre interi positivi a, b, c tali che a2 + b2 = c2 .
Se {a, b, c} è una terna pitagorica , allora anche {k a, k b, k c} è una terna pitagorica per ogni k intero positivo.
La formula
a = 2mn,
b = m2 − n 2 ,
c = m2 + n 2
al variare di m, n fra gli interi positiivi con m > n fornisce infinite terne pitagoriche.
Sono terne pitagoriche, ad esempio, {3, 4, 5}, {5, 12, 13}, {8, 15, 17}.